matematicko-fizicki list zagreb

80
MATEMATI ˇ CKO–FIZI ˇ CKI LIST (MFL) za u ˇ cenike i nastavnike. Izlazi u ˇ cetiri broja tokom ˇ skolske godine. Izdaju: HRVATSKO MATEMATI ˇ CKO DRU ˇ STVO i HRVATSKO FIZIKALNO DRU ˇ STVO Pretplata za 2008./ 2009. je 100 kn, pojedini broj 25 kn. Za inozemstvo pretplata je 28 EUR, a pojedini broj 7 EUR. (Uplata se moˇ ze obaviti u kunama ili devizama po te ˇ caju u trenutku pla´ canja.) Adresa lista je: “Matemati ˇ cko–fizi ˇ cki list, Bijeni ˇ cka 32, 10001 Zagreb, tel. (01) 4833-891, fax (01) 4683-535. Uplate na ˇ ziro ra ˇ cun: Hrvatsko fizikalno druˇ stvo, Zagreb, br. 2360000-1101301202 (kn), ZBZ d.d. SWIFT ZABA HRXX 70313-978-3239853 (EUR). Na uplatnici kao svrhu uplate molimo nazna ˇ cite “za MFL”! Molimo Vas da kod svake uplate poˇ saljete (foto-) kopiju uplatnice ili da nas obavijestite telefonom ili elektronskom poˇ stom o uplati. URL: http:/ / web.math.hr/ mfl, e-mail: [email protected] Ure - divaˇ cki odbor: ˇ ZELJKO HANJ ˇ S (Zagreb), glavni i odgovorni urednik, e-mail: [email protected] MATKO MILIN (Zagreb), urednik za fiziku, e-mail: [email protected] ANTE BILU ˇ SI ´ C (Split), IGOR GA ˇ SPARI ´ C, ZDRAVKO KURNIK, VLADIMIR PAAR, MAJA PLANINI ´ C, MIRKO POLONIJO, DUBRAVKA SALOPEK WEBER, SA ˇ SA SINGER, ANA SMONTARA, BO ˇ SKO ˇ SEGO, VLADIMIR VOLENEC, MLADEN VUKOVI ´ C, tajnica SANDRA PO ˇ ZAR (Zagreb), e-mail: [email protected] Izdavaˇ cki savjet: ALEKSA BJELI ˇ S (Zagreb), LIDIJA COLOMBO (Zagreb), BRANIMIR DAKI ´ C (Zagreb), VLADIMIR DEVID ´ E (Za- greb), MARIJAN HUSAK (Varaˇ zdin), MARGITA PA- VLEKOVI ´ C (Osijek), ERNA ˇ SU ˇ STAR (Zagreb), PETAR VRANJKOVI ´ C (Zadar), VLADIS VUJNOVI ´ C (Zagreb), PA ˇ SKO ˇ ZUPANOVI ´ C (Split) List financijski pomaˇ ze Ministarstvo znanosti, obrazovanja i ˇ sporta Republike Hrvatske. Slog i prijelom: Element, Zagreb, Men ˇ ceti´ ceva 2 Tisak: Tiskara Zelina d.d., Sv. Ivan Zelina, Ul. K. Krizmani´ c1 Naklada ovog broja 3000 primjeraka Slika na naslovnici prikazuje detektor CMS na CERN-u u fazi sklapanja. SADR ˇ ZAJ Matematika ˇ Zeljko Hanjˇ s, Soroban – japanski abak . . . . . . . . . 67 Mirjana ˇ Ci ˇ zmeˇ sija, Maja Kozuli´ c, Boˇ sko ˇ Sego Primjena osnovne analize vremenskog niza u poslovanju fondova . . . . . . . . 69 Marko Jurai´ c, Povijest matrica i determinanti . . . . . . . 76 Roko Peˇ si´ c, S razredbenog ispita u Japanu . . . . . . . 80 Petar Vranjkovi´ c, Jedno svojstvo logaritamske funkcije i njegova primjena . . . . . . . . . . . . 82 Fizika Tome Anti ˇ ci´ c i Vuko Brigljevi´ c, LHC: zaˇ sto mu se divimo? . . . . . . . . 86 Ljiljana Sudar, Geometrija i problemi kretanja . . . . . . . 95 Astronomija Dario Hrupec, Teleskop MAGIC – ˇ carobni instrument astroˇ cesti ˇ cne fizike . . . . . . . . . . . . 102 Zabavna matematika . . . . . . . . . . 105 Zadaci i rjeˇ senja A) Zadaci iz matematike . . . . . . . . . 106 B) Zadaci iz fizike . . . . . . . . . . . . 106 C) Rjeˇ senja iz matematike . . . . . . . . . 107 D) Rjeˇ senja iz fizike . . . . . . . . . . . 112 Zanimljivosti Sanja Varoˇ sanec, Matemati ˇ cka konferencija u ˇ cast akademiku Josipu Peˇ cari´ cu . . . . . 117 49. me - dunarodna matemati ˇ cka olimpijada . . 120 Druga srednjoeuropska matemati ˇ cka olimpijada, Olomouc, ˇ Ceˇ ska, 4. – 10. rujna 2008. . . . 123 Me - dunarodno matemati ˇ cko natjecanje “Klokan bez granica” 2008. g. . . . . . . 126 Nove knjige ˇ Zarko Dadi´ c, Egzaktne znanosti u Hrvata u poslijeprosvjetiteljskom razdoblju (1789.–1835.) . . . . . . . . . . . . . . 136 Kvalifikacijski ispiti Razredbeni ispit na FOI-u iz matematike 2008. g. . . . . . . . . . . . 138 Nagradni natje ˇ caj br. 185 . . . . . . . . 144

Upload: miljan-jeremic

Post on 16-Jan-2016

350 views

Category:

Documents


57 download

DESCRIPTION

Zadaci iz matematike i fizike

TRANSCRIPT

Page 1: Matematicko-fizicki list Zagreb

MATEMATICKO–FIZICKI LIST (MFL)za ucenike i nastavnike.Izlazi u cetiri broja tokom skolske godine.

Izdaju:HRVATSKO MATEMATICKO DRUSTVO iHRVATSKO FIZIKALNO DRUSTVOPretplata za 2008./ 2009. je 100 kn, pojedini broj25 kn.Za inozemstvo pretplata je 28 EUR, a pojedini broj7 EUR.(Uplata se moze obaviti u kunama ili devizama potecaju u trenutku placanja.)

Adresa lista je:“Matematicko–fizicki list, Bijenicka 32, 10001Zagreb, tel. (01) 4833-891, fax (01) 4683-535.Uplate na ziro racun:Hrvatsko fizikalno drustvo, Zagreb,br. 2360000-1101301202 (kn),ZBZ d.d. SWIFT ZABA HRXX 70313-978-3239853(EUR).Na uplatnici kao svrhu uplate molimo naznacite“za MFL”!Molimo Vas da kod svake uplate posaljete (foto-)kopiju uplatnice ili da nas obavijestite telefonomili elektronskom postom o uplati.URL: http:/ / web.math.hr/ mfl, e-mail: [email protected]

Ure -divacki odbor:

ZELJKO HANJS (Zagreb), glavni i odgovorni urednik,e-mail: [email protected] MILIN (Zagreb), urednik za fiziku,e-mail: [email protected] BILUSIC (Split), IGOR GASPARIC, ZDRAVKOKURNIK, VLADIMIR PAAR, MAJA PLANINIC, MIRKOPOLONIJO, DUBRAVKA SALOPEK WEBER, SASASINGER, ANA SMONTARA, BOSKO SEGO, VLADIMIRVOLENEC, MLADEN VUKOVIC, tajnica SANDRAPOZAR (Zagreb), e-mail: [email protected]

Izdavacki savjet:

ALEKSA BJELIS (Zagreb), LIDIJA COLOMBO (Zagreb),BRANIMIR DAKIC (Zagreb), VLADIMIR DEVIDE (Za-greb), MARIJAN HUSAK (Varazdin), MARGITA PA-VLEKOVIC (Osijek), ERNA SUSTAR (Zagreb), PETARVRANJKOVIC (Zadar), VLADIS VUJNOVIC (Zagreb),PASKO ZUPANOVIC (Split)

List financijski pomaze Ministarstvo znanosti, obrazovanjai sporta Republike Hrvatske.

Slog i prijelom:Element, Zagreb, Menceticeva 2

Tisak:Tiskara Zelina d.d., Sv. Ivan Zelina, Ul. K. Krizmanic 1Naklada ovog broja 3000 primjeraka

Slika na naslovnici prikazuje detektor CMS na CERN-u ufazi sklapanja.

SADRZAJ

MatematikaZeljko Hanjs,Soroban – japanski abak . . . . . . . . . 67Mirjana Cizmesija, Maja Kozulic, Bosko SegoPrimjena osnovne analize vremenskogniza u poslovanju fondova . . . . . . . . 69Marko Juraic,Povijest matrica i determinanti . . . . . . . 76Roko Pesic,S razredbenog ispita u Japanu . . . . . . . 80Petar Vranjkovic,Jedno svojstvo logaritamske funkcijei njegova primjena . . . . . . . . . . . . 82FizikaTome Anticic i Vuko Brigljevic,LHC: zasto mu se divimo? . . . . . . . . 86Ljiljana Sudar,Geometrija i problemi kretanja . . . . . . . 95AstronomijaDario Hrupec,Teleskop MAGIC – carobni instrumentastrocesticne fizike . . . . . . . . . . . . 102Zabavna matematika . . . . . . . . . . 105Zadaci i rjesenjaA) Zadaci iz matematike . . . . . . . . . 106B) Zadaci iz fizike . . . . . . . . . . . . 106C) Rjesenja iz matematike . . . . . . . . . 107D) Rjesenja iz fizike . . . . . . . . . . . 112ZanimljivostiSanja Varosanec, Matematicka konferencijau cast akademiku Josipu Pecaricu . . . . . 11749. me -dunarodna matematicka olimpijada . . 120Druga srednjoeuropska matematicka olimpijada,Olomouc, Ceska, 4. – 10. rujna 2008. . . . 123Me -dunarodno matematicko natjecanje“Klokan bez granica” 2008. g. . . . . . . 126Nove knjigeZarko Dadic,Egzaktne znanosti u Hrvata uposlijeprosvjetiteljskom razdoblju(1789.–1835.) . . . . . . . . . . . . . . 136Kvalifikacijski ispitiRazredbeni ispit na FOI-u izmatematike 2008. g. . . . . . . . . . . . 138Nagradni natjecaj br. 185 . . . . . . . . 144

Page 2: Matematicko-fizicki list Zagreb

Dragi citatelji!

Pocetkom jeseni ove godine po cijelom je svijetu odjeknula vijest iz CERN-a opustanju u rad Velikog hadronskog sudarivaca, LHC (engl. Large Hadron Collider)za ispitivanje materije. Ocekuju se sudari s najvisim energijama cestica koje se krecuskoro brzinama svjetlosti. Protoni i druge cestice krecu se u tunelu opsega 27 km 100metara ispod povrsine Zemlje. Eksperimenti ukljucuju vise tisuca znanstvenika iz cijelogsvijeta, a trajat ce sljedecih 10 do 20 godina, u nastojanju da se odgonetne sto vise tajnisvemira. U taj pothvat ukljuceno je i desetak hrvatskih fizicara. O ovoj zanimljivoj temiupoznaju nas Tome Anticic i Vuko Brigljevic, visi znanstveni suradnici u Zavodu zaeksperimentalnu fiziku Instituta “Ru -der Boskovic” u Zagrebu.

U prilogu profesorice Ljiljane Sudar iz Leskovca, Geometrija i problemi kretanja,pokazuje kako se mnogi problemi kretanja efikasno i elegantno rjesavaju pomocunestandarne i veoma mocne metode, koja se bazira na primjeni grafova i geometrije.

U proslosti, ne tako davnoj, koristila su se razna racunala u svakodnevnom zivotu.Ovdje se, u prilogu Soroban – japanski abak, samo prisjecamo ruskog, kineskog ijapanskog abaka. Ucenicima i nastavnicima ekonomskih skola namijenjen je clanakPrimjena osnovne analize vremenskog niza u poslovanju fondova, koji su prirediliMirjana Cizmesija, Maja Kozulic i Bosko Sego s Ekonomskog fakulteta u Zagrebu.U prilogu Povijest matrica i determinanti Marka Juraica iz Zagreba dan je ukratkopovijesni prikaz ove grane matematike. Roko Pesic navodi jedan zanimljiv zadatakkoji se pojavio na razredbeom ispitu na jednom sveucilistu u Japanu. Profesor PetarVranjkovic s gimnazije u Zadru opisuje jedno zanimljivo svojstvo logaritamske funkcijei njegovu primjenu u rjesavanju raznih nejednakosti.

Dario Hrupec iz Koprivnice u prilogu Teleskop MAGIC – carobni instrumentastrocesticne fizike opisuje novo, interdisciplinarno, znanstvenoistrazivacko podrucjekoje ukljucuje fiziku cestica, astronomiju, astrofiziku te kozmologiju.

U Trogiru je odrzana matematicka konferencija u cast akademiku Josipu Pecaricu,svjetski poznatom matematicaru iz podrucja matematickih nejednakosti, povodomnjegovog 60-tog ro -dendana. Tu su jos izvjesca za zadacima s 49. me -dunarodnematematicke olimpijade, Druge srednjoeuropske matematicke olimpijade i Me -dunarodnogmatematickog natjecanja “Klokan bez granica” 2008. g.

Akademik Zarko Dadic, autor mnogih knjiga iz fizike i matematike objavio je josjednu, Egzaktne znanosti u Hrvata u poslijeprosvjetiteljskom razdoblju (1789.–1835.).

Na zadnjoj strani omota prisjetili smo se redovitog profesora PMF-a Matematickogodjela, znanstvenika-geometricara, metodicara, autora udzbenika i popularizatoramatematike.

Urednistvo lista

66 Matematicko-fizicki list, LIX 2 (2008. – 2009.)

Page 3: Matematicko-fizicki list Zagreb

Soroban – japanski abak

Zeljko Hanjs, Zagreb

Slika 1. Ruski abak

Abak je naziv za napravu koja je sluzila za racunanje svecim brojevima, a tokom povijesti konstruirane su razlicitenjegove vrste. Spomenut cemo tri najpoznatija abaka: ruski,kineski i japanski. Pravokutnog su oblika, napravljeni oddrveta, sa zicama ili pruticima za kuglice. Kineski i japanskiabak imaju neparan broj prutica, a moze ih biti i preko 30.Ruski abak ima deset zica s po deset kuglica, kao na slici 1.

Engleska rijec abacus etimoloski je izvedena iz grckerijeci αβακιoν , koja oznacava dascice za racunanjepokrivenu pijeskom, a ova potjece od semitske rijeci kojaima znacenje dascicu za racunanje pokrivenoj prasinom ilipijeskom. U grcko i rimsko doba koristili su kuglice ilizrnca od kamena ili metala.

Slika 2. Kineski abak

Kineski abak ima svoje porijeklou rimskom abaku. Datira iz sredine16. stoljeca, imao je sedam kuglica nasvakom pruticu, dvije u gornjem i petu donjem dijelu, kao sto je prikazanona slici 2. Prenesen u Japan negdje u17. stoljecu postao je vrlo popularnompomocnom napravom za racunanje utrgovinama. U Japanu su racionaliziraliovaj abak odbacujuci suvisne kuglice.

Oko godine 1920. japanski abak, koji se tamo nazviva soroban, poprimio je novukonfiguraciju. Napravljen je od drveta, najcesce hrasta, i na svakom pruticu, obicno odbambusa, ima po pet kuglica, jednu u gornjem i cetiri u donjem dijelu ploce. Pokazalose da je to najracionalnije, jer se kod obavljanja operacija na kineskom abaku nikad nemora koristiti krajnja donja niti krajnja gornja kuglica. Blaise Pascal, veliki francuskimatematicar, u 17. stoljecu me -du prvima je konstruirao nekoliko modernijih racunskihstrojeva.

Slika 3. Japanski abak

Japanski abak je jednostavna naprava koja omogucuje brzo obavljanje aritmetickihracunskih operacija, a bazira se na decimalnom sistemu. Svako od 4 zrnca na donjem

Matematicko-fizicki list, LIX 2 (2008. – 2009.) 67

Page 4: Matematicko-fizicki list Zagreb

dijelu prutica abaka ima vrijednost 1, a zrnce na njegovom gornjem dijelu ima vrijednost5. U neutralnom polozaju, kada su donja zrnca pomaknuta do kraja dolje, a gornje dokraja gore, njegova vrijednost je jednaka nuli. Tek pomakom zrnca prema sredini onopoprima odre -denu vrijednost, 1 ili 5.

Prije pojave elektronickih racunala/ kalkulatora soroban je u Japanu bio u sirokojupotrebi. Bio je vrlo koristan u skolama za ucenje osnovnih racunskih operacija.Stovise, koristenjem sorobana razvijale su se i sposobnosti mentalnog racunanja. Tadase predvi -dalo da ce abak u buducnosti imati vrlo vaznu ulogu, ne samo u svakodnevnomzivotu.

Jos prije tridesetak godina soroban je bio u velikoj upotrebi, ali su ga potiskivalikalkulatori i danas se vise gotovo i ne koristi. U Japanu se vise ne proizvode klasicnisorobani i zadrzali su se jos samo kao neizbjezni turisticki suveniri sa samo do desetakprutica, najcesce od plastike. Ipak uz dosta truda moze se tek tu i tamo u trgovininaci i klasicni soroban s 23 prutica. Po velicini sorobani se dijele na male (s 13prutica), srednje (s 15 prutica) i velike (s 23 prutica). Inace ih ima i s po 27, pa caki s 31 pruticem, ali su najcesci bili oni s 21 pruticem jer su uglavnom zadovoljavalisvakodnevne potrebe. U pravilu broj prutica mora biti neparan.

Godine 1954. Takashi Kojima je izdao knjizicu The Japanese Abacus, Its Useand Theory da bi 1996. izaslo vec njezino 36. izdanje. Tamo je na mnogo primjerailustrirano kako se vrse osnovne racunske operacije: zbrajanje, oduzimanje, mnozenje,dijeljenje. Tako -der su se odrzavala natjecanja oko 40-tih godina proslog stoljeca izme -dutadasnjih elektronickih racunala i natjecatelja koji su koristili soroban. Godine 1946.odrzano je u japanu jedno takvo natjecanje, a pobjeda japanskog abaka bila je tadajos uvijek neupitna. Tada su strucnjaci za soroban kod zbrajanja i oduzimanja bili brziod tadasnjih elektronickih racunala. U Japanu je nekad postojao Institut za istrazivanjeabaka.

S vremenom je abak/ soroban postigao nevjerojatnu popularnost u Japanu. Godine1965. oko milijun zainteresiranih ljudi je pristupilo ispitivanju vlastitih sposobnostiracunanja na sorobanu, od kojih je njih cetvrtina polozila ispit (5000 ih je postigloodobrenje prvog, 25 000 drugog i 230 000 treceg stupnja).

Abak je bio ukljucen u nastavni program za osnovne skole kod ucenja elemenataaritmetike, a izucavao se i za potrebe u trgovini. Ukratko, abak je postao vrlo popularnimpredmetom kojeg je imalo skoro svako domacinstvo.

Kako objasniti iznenadan pad popularnosti nekad neizbjeznog i vrlo popularnogabaka, u vrijeme modernizacije tokom dvadesetog stoljeca? Bez ikakve dvojbe glavnoobjasnjenje je u cinjenici da su operacijske metode kod sve modernijih kalkulatorapostale znatno jednostavnije, savrsenije i brze.

Iako su se u poslovima u Japanu sve vise koristila elektronicka racunala, nije biloiznena -dujuce sto se abak u velikoj mjeri jos dugo koristio, kako u malim, tako i uvelikim trgovinama.

Jos sredinom proslog stoljeca strucnjaci za abak/ soroban bili su misljenja da cevrijeme utroseno na ucenje i vjezbavanje operacija na tom racunalu, biti korisno zaucenje aritmetike, a ujedno ce biti ogromna pomoc u poslovima i u svakodnevnomzivotu.

Abak je izuzetno vazan i kao pomoc slijepima. U japanskim skolama za slijepe, svevrste pomagala za racunanje baziraju se na abaku. Cak su se od 1964. g. provodili ispitiracunanja za slijepe osobe.

Obavljanje operacija na sorobanu je potpuno identicno rukovanju s kineskim abakom.U casopisu Matka, 1994. br. 10, opisano je ukratko kako se na kineskom abaku vrsizbrajanje, oduzimanje i mnozenje, a u Matematicko-fizickom listu, 1994./ 95. br. 2/ 178,dani su algoritni za racunanje drugog i treceg korijena.

Ipak i ovo je tek prisjecanje na nekad zlatno doba abaka/ sorobana.

68 Matematicko-fizicki list, LIX 2 (2008. – 2009.)

Page 5: Matematicko-fizicki list Zagreb

Primjena osnovne analize vremenskog niza u poslovanju fondova

Mirjana Cizmesija∗ , Maja Kozulic∗∗ , Bosko Sego∗∗∗ , Zagreb

Uvod

Analiza vremenskih nizova ima siroko podrucje primjene kako u strucnim tako i uznanstvenim istrazivanjima. Analiza dinamike promatrane pojave vazna je za donosenjeposlovnih odluka, a moze posluziti i kao podloga za daljnje slozenije statisticke analize.

Vremenski niz je niz kronoloski ure -denih vrijednosti promatrane pojave, a notira sena sljedeci nacin:

Y : y1, y2, ..., yt, ..., yn t = 1, 2, 3, ..., n, (1)

pri cemu je Y promatrana pojava u vremenu, yt je vrijednost pojave Y u vremenu t(frekvencija vremenskog niza), a n je duljina vremenskog niza.

Prema nacinu promatranja pojave u vremenu razlikuju se intervalni i trenutacnivremenski nizovi. Intervalni niz nastaje promatranjem odre -dene pojave u odre -denimvremenskim intervalima (godina, tromjesecje, mjesec, dan i sl.). Cesto se u ekonomijipromatraju sljedece pojave: godisnja proizvodnja, mjesecni broj prevezenih putnika,dnevni ostvareni promet, i dr. Intervalni vremenski niz ima svojstvo kumulativnostisto znaci da zbroj frekvencija intervalnog niza ima smislenu interpretaciju. Trenutacnivremenski niz nastaje promatranjem pojave u odre -denom trenutku (npr. stanje na tekucemracunu, broj zaposlenih i dr.). Trenutacni niz nema svojstvo kumulativnosti.

Vremenski nizovi mogu biti izvorni i izvedeni. U izvornom nizu frekvencijenastaju izravnim mjerenjem pojave po odabranim intervalima vremena ili u odabranimvremenskim tockama. Izvedeni vremenski niz cine frekvencije koje su nastale primjenombrojcanih operacija nad izvornim vrijednostima jednog ili vise vremenskih nizova.

Graficka analiza vremenskog niza

Veoma korisno sredstvo u analizi vremenskog niza su graficki prikazi. U tablicnimkalkulatorima kao sto su Excel, Lotus i drugi moguce je konstruirati razlicite vrstegrafickih prikaza. Najcesce se vremenski nizovi prikazuju povrsinskim i linijskimgrafikonima. Za prikaz vremenskog niza koji pokazuje sezonske oscilacije pojave svremenom koristi se polarni dijagram. Grafikon mora imati naslov, izvor podataka(ako se koriste sekundarni podaci) i po potrebi napomene i legendu. Ako se frekvencijevremenskog niza nalaze na visokoj brojcanoj razini (daleko od nule) i s malim rasponomvarijacije, prikladno je primijeniti horizontalni prekid mjerila (prekida se mjerilo na osiordinata). Uobicajeno se na osi apscisa nalazi aritmeticko mjerilo za vrijeme, a na osiordinata je aritmeticko mjerilo za frekvencije razmatrane pojave

∗ Docentica na Ekonomskom fakultetu Sveucilista u Zagrebu, Katedra za statistiku.∗∗ Studentica Ekonomskog fakulteta Sveucilista u Zagrebu.∗∗∗ Redoviti profesor na Ekonomskom fakultetu Sveucilista u Zagrebu, Katedra za matematiku.

Matematicko-fizicki list, LIX 2 (2008. – 2009.) 69

Page 6: Matematicko-fizicki list Zagreb

Intervalni niz prikazuje se povrsinskim ili linijskim grafikonom tako da se grafikonkonstruira u prvom kvadrantu koordinatnog sustava.1 Trenutacni vremenski niz prikazujese samo linijskim grafikonom.

Osnovni pokazatelji dinamike vremenskog niza

Iz grafickog prikaza mogu se uociti osnovne karakteristike stanja i razvoja promatranepojave u odre -denom vremenu. Te se promjene mogu kvantificirati izracunom osnovnihpokazatelja dinamike vremenskog niza. Pokazateljima se mjere promjene koje je mogucerazlikovati prema vise kriterija:

— prema razdoblju za koje se pokazatelj izracunava:• pojedinacne promjene,• prosjecne promjene,

— prema mjernoj jedinici u kojoj su promjene izrazene:• apsolutne promjene,• relativne promjene,

— prema razdoblju s kojim se uspore -duje:• promjene u odnosu na razinu pojave u odabranom baznom vremenskom

razdoblju,• promjene razine pojave u uzastopnim vremenskim razdobljima.

Slijedi kratki pregled osnovnih pokazatelja dinamike po navedenim kriterijima.

Pojedinacne, apsolutno izrazene, promjene pojave u uzastopnim razdobljima (prvediferencije)

Ako su Y : y1, y2, ..., yt, ..., yn , t = 1, 2, 3, ..., n , frekvencije vremenskog nizas jednakim intervalima promatranja, pojedinacne promjene u apsolutnom iznosu uuzastopnim razdobljima dane su izrazom:

Δyt = yt − yt−1, t = 2, 3, 4, ..., n. (2)

Navedene promjene tumace se kao povecanje (ako je vrijednost pojave definiraneformulom (2) pozitivan broj) ili smanjenje (ako je rijec o negativnom broju) promatranepojave u tekucem razdoblju u odnosu na prethodno razdoblje. Naravno, ako je Δyt = 0za dano t , onda u trenutku t nije doslo do promjene vrijednosti pojave Y u odnosuna trenutak t − 1. Pri tome su promjene izrazene u istim mjernim jedinicama kao ipromatrana pojava (u apsolutnim mjernim jedinicama).

Pojedinacne, apsolutno izrazene promjene pojave u odnosu na (fiksno) baznorazdoblje

Δy∗t = yt − yt−1, t = 1, 2, 3, 4, ..., n. (3)

Ove se promjene tumace kao porast ili smanjenje promatrane pojave u tekucemrazdoblju u odnosu na neko fiksno, bazno razdoblje. Tako -der su izrazene u istimmjernim jedinicama kao i pojava za koju se izracunavaju.

1 Ako se prikazuje intervalni vremenski niz s nejednakim razdobljima promatranja, potrebno je korigiratifrekvencije niza. Korigiranje frekvencija ne provodi se kod trenutacnog niza jer frekvencije toga niza pokazujustanje pojave u odre -denom trenutku, a ne nad odre -denim razdobljem.

70 Matematicko-fizicki list, LIX 2 (2008. – 2009.)

Page 7: Matematicko-fizicki list Zagreb

Prosjecna promjena pojave (prosjecna prva diferencija)

Δy =

n∑t=2

Δyt

(n − 1)ili Δy =

yn − y1

n − 1. (4)

Buduci da se prosjecna prva diferencija izracunava iz razlike posljednje i prvefrekvencije, a ostale frekvencije vremenskog niza nemaju utjecaj na velicinu prosjeka,ovu je prosjecnu mjeru promjene pojave uputno koristiti ako pojava ne pokazuje znacajnevarijacije u vremenu (izme -du prvog i posljednjeg razdoblja promatranja).

Pojedinacne stope promjene pojave u uzastopnim razdobljima

st =yt − yt−1

yt−1· 100, t = 2, 3, 4, ..., n. (5)

Stopa promjene pojave pokazuje za koliko se (izrazeno relativno) promijenila razinapojave u vremenu t u odnosu na vrijeme t − 1.

Pojedinacne stope promjene pojave u odnosu na (fiksno) bazno razdoblje

s∗t =yt − yb

yb· 100, t = 1, 2, 3, ..., n. (6)

Stopa promjene pojave u odnosu na bazno razdoblje pokazuje za koliko se (izrazenorelativno) promijenila razina pojave u vremenu t u odnosu na bazno razdoblje b .

Ako je potrebno izmjeriti prosjecnu (relativno izrazenu) promjenu pojave u odre -denomrazdoblju, izracunava se prosjecna stopa promjene.

Prosjecna stopa promjene pojave

s = (G − 1) · 100, G =(

n−1

√yn

y1− 1

), (7)

pri cemu je G geometrijska sredina.Prosjecnu stopu promjene tako -der nije uputno koristiti ako vremenski niz pokazuje

izrazito velike oscilacije s vremenom. Time prva i posljednja frekvencija niza nisupouzdan pokazatelj dinamike razvoja pojave unutar cijelog niza.

Prognoza pojave pomocu prosjecne stope promjeneAko pojava nema izrazene varijacije u vremenu i ako se pretpostavi da ce se i u

narednom (kratkorocnom) razdoblju mijenjati prema utvr -denoj prosjecnoj stopi, mozese predvidjeti buduca razina pojave u vremenu n + τ koristeci se formulom

yn+τ = yn · Gτ , (8)pri cemu je:

τ – broj razdoblja nakon posljednjeg za koje se provodi prognoza,yn+τ – prognozirana vrijednost pojave u razdoblju n + τ .

Individualni indeksiU osnovnoj numerickoj analizi vremenskog niza nezaobilazno je koristenje

individualnih indeksa. Ako se pomocu indeksa mjeri dinamika razvoja jedne pojave tadasu to individualni indeksi. Skupni indeksi mjere dinamiku skupine pojava. Individualniindeksi mogu biti verizni (lancani) i bazni. Indeksi su pozitivni brojevi, a mogu bitimanji od sto, veci od sto ili jednaki sto.

Veriznim indeksom se mjeri promjena pojave u tekucem razdoblju u odnosu naprethodno razdoblje (izrazena relativno). To su stope promjene pojave u uzastopnim

Matematicko-fizicki list, LIX 2 (2008. – 2009.) 71

Page 8: Matematicko-fizicki list Zagreb

vremenskim razdobljima povecane za 100. Verizni indeks pokazuje koliko jedinicapojave u vremenu t dolazi na sto jedinica pojave u vremenu t − 1.

Vt =yt

yt−1· 100, t = 2, 3, 4, ..., n, (9)

Vt = st + 100. (10)

Verizni indeksi se mogu graficki prikazati grafikonom jednostavnih stupaca2 ili linijskimgrafikonom.

Bazni indeksi (indeksi na stalnoj bazi) izrazavaju promjenu pojave u tekucemrazdoblju u odnosu na bazno razdoblje (izrazena relativno). To su stope promjene pojaveu odnosu na bazno razdoblje povecane za 100. Bazni indeks pokazuje koliko jedinicapojave u vremenu t dolazi na sto jedinica pojave u baznom vremenu b :

It =yt

yb· 100, t = 1, 2, 3, 4, ..., n, (11)

It = s∗t + 100. (12)Buduci da se pri izracunu baznih indeksa frekvencije niza dijele istim brojem, indeksina stalnoj bazi proporcionalni su velicinama iz kojih su izracunani (frekvencijamavremenskog niza). Kao i verizni indeksi, oni su jednaki sto, manji od sto ili veci od sto.

Primjena osnovne analize vremenskog niza u poslovanju fondova

datum cijena udjelafonda u kn

25.10.2006. 154.8625.11.2006. 154.4725.12.2006. 157.1125.01.2007. 164.7525.02.6007. 177.4825.03.2007. 184.4425.04.2007. 200.7125.05.2007. 206.1225.06.2007. 204.0925.07.2007. 210.7425.08.2007. 206.6725.09.2007. 211.1225.10.2007. 220.6326.11.2007. 198.9224.12.2007. 208.4325.01.2008. 190.1125.02.2008. 185.1325.03.2008. 169.7224.04.2008. 164.85

Tablica 1. Cijene udjela PBZ Equity fonda. Izvor: www.pbzinvest.hr

Na temelju podataka o cijenama udjela PBZ Equity fonda3 provedena je empirijskaanaliza vremenskog niza za razdoblje od 25. listopada 2006. do 24. travnja 2008. godine.Duljina vremenskog niza je 19. Cijene su biljezene gotovo istog dana u mjesecu, stoga

2 Pri tome se na osi apscisa nanosi aritmeticko mjerilo za vrijeme koje upucuju na promjenu u tekucem premaprethodnom razdoblju.

72 Matematicko-fizicki list, LIX 2 (2008. – 2009.)

Page 9: Matematicko-fizicki list Zagreb

je vremenski niz cijena udjela fonda trenutacni vremenski niz (tablica 1).Cijena udjela u fondu formira se ovisno o kretanjima cijena dionica u koje fond ulaze,

ovisi o trzistu efektive, te o kretanju cijena drzavnih obveznica. Najvazniji cilj Fonda jesigurno plasirati prikupljena sredstva, te osigurati stalnu likvidnost uloga imatelja, uz stovecu profitabilnost ulaganja. Osnovna analiza vremenskog niza je od posebne vaznostiu donosenju poslovnih odluka svih onih koji su direktno ili indirektno vezani uz radfonda.

100120140160180200220240

25.1

0.20

06.

25.1

2.20

06.

25.0

2.60

07.

25.0

4.20

07.

25.0

6.20

07.

25.0

8.20

07.

25.1

0.20

07.

24.1

2.20

07.

25.0

2.20

08.

24.0

4.20

08.

cije

na, k

n

Grafikon 1. Cijene udjela PBZ equity fonda. Izvor: www.pbzinvest.hr

datum

cijenaudjelafondau kn

verizniindeksi

bazni indeksi25.04.06=100

prvediferencije

cijeneudjela

pojedinacnestope cijene

udjela

stopepromjeneu odnosuna bazu

b =25.04.2007.t yt Vt It Δyt st s∗t1 2 3 4 5 6 7

25.10.2006. 154.86 − 77.16 − − −22.8425.11.2006. 154.47 99.75 76.96 −0.39 −0.25 −23.0425.12.2006. 157.11 101.71 78.28 2.64 1.71 −21.7225.01.2007. 164.75 104.86 82.08 7.64 4.86 −17.9225.02.6007. 177.48 107.73 88.43 12.73 7.73 −11.5725.03.2007. 184.44 103.92 91.89 6.96 3.92 −8.1125.04.2007. 200.71 108.82 100.00 16.27 8.82 0.0025.05.2007. 206.12 102.70 102.70 5.41 2.69 2.7025.06.2007. 204.09 99.02 101.68 −2.03 −0.98 1.6825.07.2007. 210.74 103.26 105.00 6.65 3.26 5.0025.08.2007. 206.67 98.07 102.97 −4.07 −1.93 2.9725.09.2007. 211.12 102.15 105.19 4.45 2.15 5.1925.10.2007. 220.63 104.50 109.92 9.51 4.50 9.9225.11.2007. 198.92 90.16 99.11 −21.71 −9.84 −0.8924.12.2007. 208.43 104.78 103.85 9.51 4.78 3.8525.01.2008. 190.11 91.21 94.72 −18.32 −8.79 −5.2825.02.2008. 185.13 97.38 92.24 −4.98 −2.62 −7.7625.03.2008. 169.72 91.68 84.56 −15.41 −8.32 −15.4424.04.2008. 164.85 97.13 82.13 −4.87 −2.87 −17.87

Tablica 2. Kretanje cijene udjela PBZ Equity fonda kroz godinu dana

Iz navedenog grafickog prikaza (grafikon 1) moze se uociti da su cijene udjela dolistopada prosle godine pokazale dinamiku ujednacenog rasta, a od tada se, s iznimkomu sijecnju 2008. godine kada su se povecale, kontinuirano smanjuju. Uocenu dinamikukretanja moguce je kvantitativno izraziti osnovnim pokazateljima dinamike kako jeizlozeno u prethodnom dijelu rada (tablica 2). Pri tome je nuzno ukazati na ogranicenjaprimjene nekih od pokazatelja koristenima u analizi navedenog vremenskog niza cijenaudjela.

Matematicko-fizicki list, LIX 2 (2008. – 2009.) 73

Page 10: Matematicko-fizicki list Zagreb

Diferencija uz datum 25.04.2007. prikazuje da je na taj dan cijena udjela u Equityfondu bila veca za 16.25 kn u odnosu na mjesec prije tj. u odnosu na 25.03.2007. No,diferencija uz datum 25.08.2007. prikazuje da je cijena udjela toga dana bila manja za4.07 kn u odnosu na 25.07.2007. Diferencije ostalih dana u mjesecu interpretiraju se naisti nacin.

Pojedinacna stopa promjene cijene udjela Equity fonda na dan 25.05.2007. godineiznosi 2.69. To znaci da je prinos toga dana bio veci za 2.69% u odnosu na 25.04.2007.Dok je 25.06.2007. godine stopa promjene cijene iznosi −0.98. To znaci da je prinostog dana bio manji za 0.98% u odnosu na prethodno razdoblje.

Stopa promjene u odnosu na bazno razdoblje pokazuje za koliko se relativnopromijenila razina pojave u vremenu t u odnosu na bazno razdoblje. Stopa promjene25.11.2006. u odnosu na bazno razdoblje 25.04.2007. iznosila je −23.04, sto znacida je prinos cijena udjela u Fondu tog datuma u odnosu na bazni datum bio nizi za23.04%, dok je 25.10.2007. stopa promjene iznosila 9.92 u odnosu na bazno razdoblje,sto je za 9.92% visa cijena udjela u Fondu u odnosu na bazno razdoblje. Tumacenjestopa promjene u uzastopnim razdobljima jednako je tumacenju veriznih indeksa (tabela2, stupac 3), a stope promjene u odnosu na bazno razdoblje tumace se isto kao i bazniindeksi (tabela 2, stupac 4), odnosno bazni indeks cijene udjela 25.10.2006. iznosi 77.16sto pokazuje da je na 100 kuna cijene udjela u fondu 25.04.2007. dolazilo 77.16 kunacijene udjela 25.10.2006., odnosno da je cijena udjela u fondu bila manja za 22.84%.

U odnosu na bazni datum 25.04.2007. najveci pad cijene udjela u fondu bioje 25.11.2006., dok je najveci porast cijene u odnosu na bazni datum zabiljezen25.10.2007.

Ranije je vec konstatirano, a grafikon 3 to i potvr -duje, da su se cijene udjela dolistopada 2007. godine uglavnom povecavale u odnosu na prethodni mjesec. Stupci nagrafikonu veriznih indeksa koji su iznad osi 100 pokazuju povecanje. Smanjenje u tomrazdoblju zabiljezeno je u studenome 2006., lipnju i kolovozu 2007. godine. Nakon togaje uslijedilo razdoblje kada su verizni indeksi bili manji od 100, stupci na grafikonusu stoga ispod osi 100, sto znaci da su se cijene uzastopno smanjivale u odnosu naprethodni mjesec osim u prosincu kad su se povecale za nesto manje od 5%.

85

95

105

115

11.0

6./1

0.06

.

12.0

6./1

1.06

.

01.0

7./1

2.06

.

02.0

7./0

1.07

.

03.0

7./0

2.07

.

04.0

7./0

3.07

.

05.0

7./0

4.07

.

06.0

7./0

5.07

.

07.0

7./0

6.07

.

08.0

7./0

7.07

.

09.0

7./0

8.07

.

10.0

7./0

9.07

.

11.0

7./1

0.07

.

12.0

7./1

1.07

.

01.0

8./1

2.07

.

02.0

7./0

1.08

.

03.0

8./0

2.08

.

04.0

8./0

3.08

.

inde

ksni

bod

ovi

Grafikon 3. Verizni indeksi cijena udjela PBZ Equity fonda. Izvor: izracun autora

Buduci da je iz grafickog prikaza i iz stopa promjene u uzastopnim razdobljima,odnosno iz veriznih indeksa ocito da se cijena udjela uzastopno povecavala do listopada2007., a od tada se smanjivala, nije uputno racunanje prosjecne prve diferencije nitiprosjecne stope promjene cijene za cijelo razdoblje. Kao sto je ranije navedeno, ovese prosjecne mjere temelje na prvoj i posljednjoj frekvenciji niza koje u ovom slucajunisu dobar reprezentant dinamike cijena u cjelokupnom razdoblju. Stoga je vremenskiniz podijeljen u dva dijela: od listopada 2006. do listopada 2007. i od studenoga 2007.do travnja 2008. godine i izracunate su prosjecne mjere promjene za svaki dio nizaposebno.

74 Matematicko-fizicki list, LIX 2 (2008. – 2009.)

Page 11: Matematicko-fizicki list Zagreb

Tako je za prvo razdoblje (do listopada 2007. godine) prosjecna prva diferencija:

ΔyI =yn − y1

n − 1=

220.63− 154.8613 − 1

= 5.48,

a za razdoblje od studenoga 2007. do travnja 2008. to je:

ΔyII =164.85− 198.92

6 − 1= −6.82

Prosjecna stopa promjene cijene udjela PBZ Equity fonda za oba razdoblja je:

sI =(

(n−1)

√yn

y1− 1

)· 100 =

(12

√220.63154.86

− 1

)· 100 = 2.99

sII =

(5

√164.85198.92

− 1

)· 100 = −3.69

Cijena udjela u fondu u razdoblju od 25. listopada 2006. do 25. listopada 2007.povecavala se prosjecno mjesecno za 5.48 kuna ili gotovo 3%, a nakon toga u razdobljuod studenoga 2007. do travnja 2008. se smanjivala prosjecno mjesecno za 6.82 kn ili za3.7%.

Na temelju uocene dinamike cijena udjela u fondu, nije moguce pretpostaviti dace se cijena mijenjati u narednom razdoblju po posljednjoj utvr -denoj prosjecnoj stopi,stoga nije moguce niti prognoziranje njene razine na temelju posljednje cijene iz travnjai prosjecne stope promjene. Prognoziranje je nuzno provesti slozenijim statistickimprognostickim metodama i modelima koji nisu tema ovoga rada.

Zakljucak

Strategija ulaganja i izbora instrumenata od kojih ce se graditi portfelj Fonda sadrzinesto veci rizik ulaganja, ali s tim nosi i vece prinose u fondu. Fond je namijenjenulagateljima koji zele plasirati dio svoje imovine u domace i strane dionice, ulagati nadulji rok te ostvariti vece stope prinosa. Cilj fonda je ostvarivanje relativno velikogprinosa kroz dulje razdoblje, uz postojanje kratkorocnog rizika. Kontinuirano pracenje ianaliziranje promjena na trzistu kapitala postaje imperativ.

Analiza cijene udjela PBZ Equity fonda u devetnaest mjeseci dala je niz korisnihinformacija. Tendencija rasta cijene udjela u Fondu je sve do listopada 2007. godine bilarastuca, ali se nakon toga uocava poremecaj i kontinuirano smanjenje cijena koje mozebiti jednim dijelom posljedica zasicenosti trzista fondovima ciji se broj s vremenompovecava.

Graficka i numericka analiza vremenskog niza osnovnim pokazateljima dinamike imasiroke, ali uvijek treba imati na umu i ogranicenje, mogucnosti primjene. Numerickipostupci nisu zahtjevni, a daju korisne informacije o dinamici promatrane pojave, stocini izvor informacija za odlucivanje.

Literatura

[1] K. BLACK, Business statistics for contemporary decision making, Willey, New York (2006).[2] S. PIVAC, B. SEGO, Statistika – udzbenik i zbirka zadataka, Alka script, Zagreb (2005).[3] I. SOSIC, Primijenjena statistika, Skolska knjiga, Zagreb (2006).[4] I. SOSIC, Statistika, Skolska knjiga, Zagreb (2006).[5] I. SOSIC, V. SERDAR, Uvod u statistiku, XII. izdanje, Skolska knjiga, Zagreb (2002).[6] www.pbzinvest.hr

Matematicko-fizicki list, LIX 2 (2008. – 2009.) 75

Page 12: Matematicko-fizicki list Zagreb

Povijest matrica i determinanti

Marko Juraic, Zagreb

Poceci koristenja matrica i determinanti sezu u drugo stoljece pr. Kr., no neki tragovigovore da su se koristile cak i u cetvrtom stoljecu pr. Kr. Ipak pravi razvoj i koristenjeje zapocelo tek u 17. st.

Matrice i determinante su nastale proucavanjem sustava linearnih jednadzbi. Babiloncisu proucavali probleme koji su se mogli rijesiti koristenjem takvih jednadzbi, a neki odnjih su sacuvani na glinenim plocicama. Jedna takva plocica potjece iz oko 300. godinapr. Kr. i sadrzi sljedeci problem:

Imamo dva polja cija je ukupna povrsina 1800 m2 . Oba polja su za proizvodnju zita,

s tim da jedno daje23

kg/m2 , a drugo daje12

kg/m2 . Ukoliko je ukupan rod 1100 kg,

kolika je povrsina svakog polja?Kinezi su se, izme -du 200. i 100. godine pr. Kr., vise priblizili matricama nego

Babilonci. Za vrijeme dinastije Han izra -den je tekst pod nazivom Nine Chapters onthe Mathematical Art (Devet poglavlja matematickog umijeca) koji je prvi ikad na -denprimjer koristenja matricnih metoda. Prvi problem je vrlo slican Babilonskom, a glasiovako:

Imamo tri vrste zita, od cega tri mjere prvog, dvije drugog i jedna treceg zajednocine 39 jedinica. Dvije mjere prvog, tri drugog i jedan treceg cine 34 jedinica, a jednamjera prvoga, dvije drugoga i tri trecega cine 26 jedinica. Koliko jedinica zita sadrzijedna mjera za svaki tip zita?

Ovaj problem su rjesavali tako da su postavili jednadzbe, cije koeficijente su posloziliu stupce pridruzene nepoznanicama:

1 2 32 3 23 1 1

26 34 39

Danasnja metoda rjesavanja kaze nam da se koeficijenti pisu u redove, ali to sada nijebitno. U uputama za rjesavanje ovog zadatka navedeno je: pomnozi srednji stupac s 3i oduzmi desni stupac koliko god puta mozes. Zatim se isto napravi i s prvim stupcem(pomnozi s 3 i oduzmi se treci stupac). To daje,

0 0 34 5 28 1 1

39 24 39

Zatim se prvi stupac pomnozi s 5 i od njega se oduzme srednji stupac onoliko putakoliko je to moguce napraviti. Dobivamo,

0 0 30 5 2

36 1 199 24 39

76 Matematicko-fizicki list, LIX 2 (2008. – 2009.)

Page 13: Matematicko-fizicki list Zagreb

Odavde se moze izracunati koliko je jedinica za trecu vrstu zita, zatim za drugu, a ondai za prvu (uvrstavanjem). Ova metoda se naziva Gaussova metoda eliminacije i nije bilapoznata sve do pocetka 19. st.

Godine 1545. Jerome Cardan, u djelu Ars Magna, iznosi pravilo za rjesavanjesustava dviju linearnih jednadzbi koje se naziva majka svih pravila. Ovo pravilo jeistovjetno Cramerovom pravilu za rjesavanje (2, 2) sistema, no Cardan nije napraviozavrsni korak. Nije dosao do definicije determinante, ali je bio na pravom putu.

Mnogi rezultati elementarne matricne teorije su se pojavili mnogo prije nego sumatrice postale predmet matematicke analize, tako da su mnoge zapocete teorije bilezapravo put ka otkrivanju mnogo kompliciranijih izvoda i rezultata za koje se u tovrijeme kada su stvorene nije ni pomisljalo.

Ideja o determinanti se gotovo istovremeno pojavila u Japanu i u Europi, ali prviju je objavio Seki Takakazu (poznat kao Seki Kowa). On je 1683. g. napisao djeloMethod of solving the dissimulated problems (Metoda rjesavanja skrivenih problema)koje sadrzi metode rjesavanja pomocu matrica ispisanih u tablicu i to na isti nacin kakosu to radili u staroj Kini. Bez da je spomenuo i jednu rijec koja bi mogla predstavljati“determinantu”, dao je opcenite nacine izracunavanja determinanti sto se baziralo naprimjerima (determinante matrica reda (2, 2) , (3, 3) , (4, 4) i (5, 5)).

Iste godine se determinanta pojavila i u Europi. Godine 1683. Gottfried WilhelmLeibniz je pisao l’Hospitalu i objasnio mu da sustav jednadzbi

10 + 11x + 12y = 0

20 + 21x + 22y = 0

30 + 31x + 32y = 0ima rjesenje jer je 10 ·21 ·32+11 ·22 ·30+12·20 ·31 = 10 ·22 ·31+11 ·20 ·32+12·21 ·30sto je zapravo uvjet da je determinanta koeficijenata matrice jednaka 0. Leibniz je uovome primjeru koristio brojeve kao koeficijente, a oni zapravo predstavljaju znakovekoje mi danas pisemo kao a11 , a12 itd., odnosno prvi od dva indeksa predstavlja u kojojse jednadzbi nalazi koeficijent, a drugi oznacava nepoznanicu.

Leibniz je bio uvjeren da je kljuc uspjeha u matematici pravilno oznacavanjekoeficijenata, pa je on eksperimentirao upravo s nasumicnim odabirom pisanja. Unjegovim neobjavljenim radovima na -deno je preko 50 razlicitih nacina pisanja sistemakoeficijenata koje je pisao kroz pedesetak godina pocevsi od 1678. Samo na dva mjesta(1700. i 1710. godine) dobiven je rezultat sistema i tada je koristen zapis koji je napisaou svom pismu l’Hospitalu.

Leibniz je za danasnju “determinantu” koristio naziv “rezultanta”, i dokazao jerazlicite rezultate u ovisnosti o determinanti sto poznajemo kao Cramerovo pravilo.Isto tako je znao da se determinanta moze izracunati razvojem po bilo kojem stupcu, stose danas zove Laplaceov razvoj.

Tridesetih godina 18. st. Maclaurin je napisao Treatise of algebra (objavljeno1748. g.) u kojemu je dokazao Cramerovo pravilo za (2, 2) i (3, 3) sisteme i pokazaokako bi se mogao rijesiti (4, 4) sustav. Cramer je odredio opcenito pravilo za (n, n)sisteme u djelu Introduction to the analysis of algebraic curves (Uvod u analizualgebarskih krivulja) (1750. g.), te je u tom djelu izrazio zelju za pronalazenjemjednadzbe ravnine koja je zadana odre -denim brojem tocaka. Pravilo je napisao na krajudjela, ali ono nije bilo dokazano.

Proucavanje determinanata se sada pocelo razvijati.Godine 1764. Bezout je otkrio metode za racunanja determinanti, a isto je ucinio

i Vandermonde 1771. g. Laplace je 1772. g. tvrdio da su Cramerove i Bezoutove

Matematicko-fizicki list, LIX 2 (2008. – 2009.) 77

Page 14: Matematicko-fizicki list Zagreb

metode neprakticne te je napisao diskusije rjesenja linearnih jednadzbi bez racunanjasustava, vec je samo racunao determinante. Iznena -dujuce je da je determinante nazivao“rezultante”, jednako kao i Leibniz, a zapravo nije bio upoznat s njegovim radom.

Lagrange je 1773. g. proucavao identitete determinanti (3, 3) matrica, ali njegovi svikomentari su bili naga -danja i on nije mogao naci vezu izme -du svog rada i rada Laplaceai Vandermondea. No ipak njegovo djelo sadrzi izraz za volumen tetraedra zadanog stockama O(0, 0, 0) , M(x, y, z) , M′(x′, y′, z′) , M′′(x′′, y′′, z′′) :

16[z(x′y′′ − y′x′′) + z′(yx′′ − xy′′) + z′′(xy′ − yx′)].

Izraz “determinanta” je prvi upotrijebio Gauss u djelu Disquisitiones arithmeticae(1801. g.) dok je opisivao kvadraticne forme, a to ime je izabrao zato sto determinantaodre -duje (eng. determines) svojstva kvadratnickog oblika. No njegov koncept nijejednak onome koji mi poznajemo. U istom djelu Gauss je ispisao koeficijente svojekvadraticne forme u oblik pravokutnika. Opisao je mnozenje matrica (koje je smatraokao kompoziciju pa stoga nije dosegao koncept matricne algebre) i izracunavanje inverzamatrice.

“Gaussovu eliminaciju”, koja se prvi put pojavila u djelu Nine Chapters onMathematical Art (Devet poglavlja matematickog umijeca) 200. g. pr. Kr., Gauss jekoristio pri proucavanju putanje asteroida Pallas. Koristenjem tih podataka uzetih izme -du1803. i 1809. g., sastavio je sistem od sest linearnih jednadzbi sa sest nepoznanica.Nasao je sistematsku metodu za rjesavanje takvih jednadzbi sto se upravo i nazivaGaussova metoda eliminacije.

Augustin Louis Cauchy je 1812. g. prvi koristio determinantu u modernom smislu.Cauchyjev rad je najkompletniji rad o determinantama ikad ura -den. On je dokazaoprethodne rezultate i dao nove na temelju minora i adjunkta. Godine 1812. dokazao jeteorem o mnozenju determinanti po prvi put.

Godine 1826. Cauchy je napisao dijagonalizaciju matrice. Isto tako je uveo ideju osimetricnim matricama i pokazao da one imaju realne svojstvene vrijednosti. Poslije jedokazao i da se svaka realna simetricna kvadratna matrica moze dijagonalizirati.

Zanimljivo je da niti Cauchy niti Sturm nisu nasli opcenite relacije za ideje kojesu iznijeli, vec su ih koristili i vidjeli samo u kontekstu onoga cime su se oni bavili.Jacobi (1830. g.), a zatim i Kronecker i Weierstrass (1850. i 1860. g.), bavili suse matricnim rezultatima ali tako -der ne opcenito vec u smislu linearne transformacije.Jacobi je objavio tri rada o determinantama 1841. g. koji su bitni zato sto je po prviputa definicija determinante bila napisana preko algoritma, pa su rezultati bili tocni biloda se radilo samo o brojevima ili funkcijama. Nakon ovih radova ideja determinantipostala je poznatom cijelom svijetu.

Cayley je tako -der 1841. g. objavio prvi engleski clanak o teoriji determinanti. Nasvom papiru je koristio dvije vertikalne linije na lijevoj i desnoj strani redova cime jeoznacavao determinante, a takvo oznacavanje je ostalo i do danas.

Ime “matrica” prvi je koristio Sylvester 1850. g. On je definirao matricu kao pravilnuskupinu vrijednosti i gledao ju kao nesto sto ima ulogu odrediti determinantu vrijednostikoju sadrzava. Nakon povratka iz Amerike u Englesku 1851. g., Sylvester je postaoodvjetnik i upoznao Cayleya, odvjetnika kojeg je matematika tako -der zanimala. Kada jeCayley vidio vaznost matrica, 1853. g. objavio je clanak u kojem se po prvi put pojavioinverz matrice.

Cayley je objavio Memoir on the theory of matrices koja po prvi puta sadrzi apstraktnudefiniciju matrice 1858. g. Pokazao je da su sustavi koeficijenata za kvadraticne forme i

78 Matematicko-fizicki list, LIX 2 (2008. – 2009.)

Page 15: Matematicko-fizicki list Zagreb

linearne transformacije zapravo specijalni slucajevi njegovog opcenitog koncepta. Cayleyje pokazao definicije zbrajanja, mnozenja i inverza matrica. Tako -der je dokazao da, uslucaju (2, 2) matrice, matrica ponistava svoj svojstveni polinom. Hamilton je nakontoga dokazao specijalni slucaj za (4, 4) matrice i stoga se to otkrice danas nazivaCayley-Hamiltonov teorem.

Godine 1870. Jordan je u djelu Treatise on substitutions and algebraic equationsobjavio nesto sto se danas naziva “Jordanova forma matrice”. To je dijagonalnablok-matrica na cijoj se dijagonali nalaze tzv. “Jordanovi blokovi” (kvadratna matricaspecijalnog oblika).

Frobenius je 1878. g. napisao vrlo vazan rad On linear substitutions and bilinearforms (O linearnim supstitucijama i bilinearnim formama) s tim da nije bio upoznats Cayleyvim radom. Nakon toga je, procitavsi Cayleya, dokazao Cayley-Hamiltonovteorem za opceniti slucaj (1896), i izveo definiciju za rang matrice.

Pozitivnost kvadratne matrice je 1884. g. dao Sylvester. Dosao je do zakljucka daje kvadratna forma pozitivna ako i samo ako su glavne minore odgovarajuce matricepozitivne.

Glavni ljudi koji su poceli ozbiljno promovirati teoriju matrica i determinanti bilisu Weierstrass (objavio je rad On determinant theory (O teoriji determinante)) koji jeobjavljen 1903. nakon njegove smrti) i Kronecker (njegovi radovi su tako -der objavljeninakon njegove smrti). Teoriju determinanti su matematicari odmah prihvatili, dok je zateoriju matrica trebalo nesto vise vremena. Rad koji je pridonio potpunom shvacanjuteorije matrica bio je Introduction to higher algebra (Uvod u visu algebru) od Bochera1907. g. Turnball i Aitken su napisali vrlo utjecajne radove 1930. godine, a nakon radaMirskya 1955. (An introduction to linear algebra (Uvod u linearnu algebru)), linearnaalgebra je zauzela mjesto jedne od najvaznijih grana matematike.

� � �

Kalendar natjecanja u matematici

za ucenike osnovnih i srednjih skola 2009. g.

— Skolska natjecanja — 29. sijecnja— Zupanijska natjecanja — 23. veljace— “Klokan bez granica" — 19. ozujka— Drzavno natjecanje — od 29. ozujka do 1. travnja— Mediteransko matematicko natjecanje — travanj ili svibanj— Regionalna natjecanja — 8. svibnja— Me -dunarodna matematicka olimpijada — od 10. do 22. srpnja— Srednjoeuropska matematicka olimpijada — od 24. do 29. rujna

Matematicko-fizicki list, LIX 2 (2008. – 2009.) 79

Page 16: Matematicko-fizicki list Zagreb

S razredbenog ispita u Japanu

Roko Pesic, Zagreb

U jednom americkom casopisu objavljen je sljedeci zadatak koji se pojavio narazredbenom ispitu na jednom sveucilistu u Tokiju, ali bez rjesenja. Zadatak mi sesvi -dao, a nadam se ce i vama biti interesantan.

Slika 1.

Zadatak. Zadana je pravilna cetverostranapiramida i kugla sa sredistem koje lezi u bazipiramide. Ona dira sve njezine bridove, a bridosnovice je duljine a . Na -dite:a) visinu v piramide;b) obujam V dijela koji je zajednicki kugli ipiramidi.

Rjesenje. a) Iz uvjeta zadatka se vidi dasrediste S kugle mora biti u sredistu bazepiramide (baza je kvadrat sa stranicom duljinea) , a to znaci da je polumjer kugle jednakpolovici duljine stranice a ,

R =a2. (0)

Promotrimo pravokutni trokut ASV : |AS|2 + |SV|2 = |AV|2 , gdje je |AS| =a√

22

,

|SV| = v (visina piramide), |AV| = b (bocni brid piramide), pa je

v2 = b2 − a2

2. (1)

Tako -der iz uvjeta zadatka slijedi da je visina |SE| trokuta ASV jednaka polumjeru Rkugle, |SE| = R . Oznacimo li |AE| = x , tada je |EV| = b − x .

S obzirom da su trokuti ASV , ASE i ESV pravokutni, vrijede ove jednakosti:

R2 = x(b − x), (2)(a√

22

)2= bx =⇒ x =

a2

2b. (3)

Uvrstivsi jednakosti (0) i (3) u (2) dobivamo a = b , i to je piramida kojoj su svibridovi jednake duljine (polovica oktaedra). Iz jednakosti (1) tada dobivamo visinupiramide

v =a√

22

.

b) Zajednickom dijelu obujma pripada gornja polukugla umanjena za cetiri kuglinaodsjecka koji ne pripadaju piramidi – po jedan nad svakom pobockom piramide. Tiodsjecci su odre -deni cetirima ravninama u kojima leze pobocke piramide jer njih,(odsjecke) te ravnine odsijecaju od gornje polukugle.

Obujam kuglinog odsjecka je

Vo =16πh(3r2 + h2) , (4)

80 Matematicko-fizicki list, LIX 2 (2008. – 2009.)

Page 17: Matematicko-fizicki list Zagreb

gdje je r polumjer baze, a h visina kuglinog odsjecka. Baza kuglinog odsjecka je krugome -den kruznicom polumjera r . Ta kruznica dira bocne bridove piramide, a kako jeova jednakostranicni trokut stranice duljine a , ta kruznica je upisana u jednakostranicni

trokut pa za nju vrijedi jednakost a = 2r tgα2

, gdje je α sredisnji kut jednakostranicnog

trokuta. Kako je α = 360◦ : 3 tj. α = 120◦ , uvrstavanjem te vrijednosti u prethodnujednakost dobivamo da je a = 2r tg 60◦ , tj. a = 2r

√3 odnosno

r =a

2√

3. (5)

Slika 2. Slika 3. Slika 4.

Za odsjecak kugle polumjera R , visine h i polumjera baze r (vidi sliku 3) vrijedijednakost:

r2 = h(2R − h), (6)sto lako slijedi iz Pitagorinog poucka. Uvrstivsi (0) i (5) u (6), nakon sre -divanja

dobivamo kvadratnu jednadzbu u varijabli h , h2 − ah +a2

12= 0. Njezina rjesenja su:

h = a

(12

+1√6

)≈ 0.9a, (7a)

h = a

(12− 1√

6

)≈ 0.1a. (7b)

Rjesenje (7a) nije ispravno, jer iz (0) bi slijedilo h > R , sto ne moze biti. Rjesenje(7b) zadovoljava uvjet h < R , pa je to rjesenje ispravno.

Uvrstivsi izraze (5) i (7b) u formulu (4) za obujam kuglinog odsjecka konacnodobivamo:

Vo =a3π6

(12− 7

√6

36

). (8)

Zajednicki obujam cemo dobiti ako od obujma polukugle oduzmemo ukupni obujamcetiriju kuglinih odsjecaka:

V =V1

2− 4Vo =

23R3π − 4 · a3π

6

(12− 7

√6

36

).

Nakon sre -divanja dobivamo zajednicki obujam kugle i piramide

V =a3π2

(7√

627

− 12

).

Matematicko-fizicki list, LIX 2 (2008. – 2009.) 81

Page 18: Matematicko-fizicki list Zagreb

Jedno svojstvo logaritamske funkcije i njegova primjena

Petar Vranjkovic, Zadar

U ovom ce clanku biti govora o jednom manje poznatom svojstvu logaritamskefunkcije i njegvoj ucinkovitoj primjeni na rjesavanje nekih logaritamskih nejednadzbi.To su one u kojima su baza logaritamske funkcije i njezin argument funkcije istevarijable. Znamo da se takve nejednadzbe rjesavaju metodom razlikovanja slucajeva.Pokazat cemo da se primjenom tog svojstva spomenute nejednadzbe rjesavaju znatnolakse i jednostavnije.

Po -dimo od definicije logaritamske funkcije. Funkcija f : R+ −→ R odre -denaformulom

f (x) = logb xje definirana za b > 0 i b �= 1.

Primjer 1. Rijesimo nejednadzbulog1+x(2x − 6) ≥ 0 x ∈ R.

Rjesenje. Ovu nejednadzbu cemo rijesiti metodom razlikovanja slucajeva.

1◦ 0 < 1 + x < 1 2◦ 1 + x > 12x − 6 > 0 2x − 6 > 01 + x �= 1 1 + x �= 12x − 6 ≤ 1 2x − 6 ≥ 1

x ∈ ∅ x ≥ 72

Prema tome, rjesenje zadane nejednadzbe je x ≥ 72

.

No, uocimo sljedece nejednadzbe (ostale su u 1◦ i 2◦ ):

1 + x < 1 1 + x > 12x − 6 > 0 2x − 6 > 0

1 + x − 1 < 0 1 + x − 1 > 02x − 6 − 1 ≤ 0 2x − 6 − 1 ≥ 0

(1 + x − 1)(2x − 6 − 1) ≥ 0 (1 + x − 1)(2x − 6 − 1) ≥ 0.

Dakle, u oba slucaja su umnosci istog predznaka. Na tom tragu se pojavila ideja o“novom” svojstvu logaritamske funkcije.

Poucak. Neka je a > 0, a �= 1 i b > 0. (1)Tada vrijedi

loga b ≤ 0 ⇐⇒ (a − 1)(b − 1) ≤ 0 ili loga b � 0 ⇐⇒ (a − 1)(b − 1) ≥ 0 (2)

Dokaz. Provedimo dokaz za slucaj loga b ≤ 0.

Ako a i b zadovoljavaju (1) onda imamo:loga b ≤ 0 ⇐⇒ ((0 < a < 1 ∧ b ≥ 1) ∨ (a > 1 ∧ 0 < b ≤ 1))

82 Matematicko-fizicki list, LIX 2 (2008. – 2009.)

Page 19: Matematicko-fizicki list Zagreb

⇐⇒ ((a − 1 < 0 ∧ b − 1 ≥ 0) ∨ (a − 1 > 0 ∧ b − 1 ≤ 0))⇐⇒ (a − 1)(b − 1) ≤ 0.

Analogno se dokazuje ekvivalentnost nejedankostiloga b ≥ 0 ⇐⇒ (a − 1)(b − 1) ≥ 0.

Vratimo se sada primjeru 1 i rijesimo ga pomocu ovog poucka. Prema (1) imamo1 + x > 0, 2x − 6 > 0, 1 + x �= 1.

Rjesenje ovog sustava je x > 3. (3)Prema (2) imamo

(1 + x − 1)(2x − 6 − 1) ≥ 0,

odnosno x(2x − 7) ≥ 0 tj. 2x2 − 7x ≥ 0.

Rjesenje ove kvadratne nejednadzbe je x ≤ 0 ili x ≥ 72

, a zbog (3), konacno rjesenje

je x ≥ 72.

U slucaju kada je ili baza ili argument logaritamske funkcije konstanta, onda jepostupak brzi i laksi. Evo jednog primjera.

Primjer 2. Rijesimo nejednadzbulog 1

3(4x + 1) < 0, x ∈ R.

Rjesenje. Prema (1) je 4x + 1 > 0, pa je x > −14

, a prema (2) imamo(13− 1

)(4x + 1 − 1) < 0,

odnosno x > 0. Konacno rjesenje je x > 0.

Zadatak 1. Ako je a > 0, b > 0, c > 0 i a �= 1, onda vrijediloga b ≥ loga c ⇐⇒ (a − 1)(b − c) ≥ 0 (4)loga b ≤ loga c ⇐⇒ (a − 1)(b − c) ≤ 0 (4*)

Dokaz. Dovoljno je dokazati prvu tvrdnju.

loga b ≥ loga c ⇐⇒ loga b − loga c ≥ 0

⇐⇒ logabc≥ 0 ⇐⇒ (a − 1)

(bc− 1

)≥ 0 ⇐⇒ (a − 1)(b − c) ≥ 0.

Pri cemu je koristeno (2).

Evo jednog primjera.

Primjer 3. Na -dimo rjesenje nejednadzbelog2x(4x − 1) ≥ 2, x ∈ R.

Rjesenje. Domenu rjesenja cemo dobiti rjesavanjem sustava2x > 0, 2x �= 1, 4x − 1 > 0.

Dobijemo

x ∈⟨

14, +∞

⟩, x �= 1

2. (5)

Matematicko-fizicki list, LIX 2 (2008. – 2009.) 83

Page 20: Matematicko-fizicki list Zagreb

Zadanu nejednadzbu cemo transformirati tako da mozemo primijeniti nejednakost(4). Dobijemo

(2x − 1)(4x − 1 − (2x)2) ≥ 0,

odnosno, nakon sre -divanja (2x − 1)3 ≤ 0.

Odavde imamo 2x − 1 ≤ 0, tj. x ≤ 12

. Uzimajuci u obzir domenu rjesenja, konacno

rjesenje je x ∈⟨

14,12

⟩.

Primjer 4. Rijesimo nejednadzbu

log|x|√

x2 − 9 ≥ 1. (6)

Rjesenje. Odredimo domenu rjesenja:

x �= −1, x �= 0, x �= 1, x2 − 9 > 0.

Ovi uvjeti su zadovoljeni za

x ∈ 〈−∞,−3〉 ∪ 〈 3, +∞〉 . (7)

Nadalje, (6) mozemo zapisati

log|x|√

x2 − 9 ≥ log|x| |x|.Prema (4) imamo

(|x| − 1)(√

x2 − 9 − |x|) ≥ 0. (8)

Dalje, mozemo rjesavati na standardni nacin, ali ovdje cemo koristiti drugu metodu.

U domeni rjesenja sigurno vrijedi√x2 − 9 + |x| ≥ 0. (9)

Pomnozimo li (8) s (9), dobivamo

(|x| − 1)(x2 − 9 − x2) ≥ 0 tj. |x| − 1 ≤ 0.

Rjesenje ove nejednadzbe je −1 ≤ x ≤ 1, a iz (7) imamo konacno rjesenje x ∈ ∅ , tj.nijedan realan broj x ne zadovoljava danu nejednadzbu.

Zadatak 2. Neka je

a > 0, b > 0, c > 0, d > 0, a �= 1, c �= 1. (10)

Tada je

1◦ loga b · logc d ≤ 0 ⇐⇒ (a − 1)(b − 1)(c − 1)(d − 1) ≤ 0 ili (11)

loga b · logc d ≥ 0 ⇐⇒ (a − 1)(b − 1)(c − 1)(d − 1) ≥ 0 (11∗)2◦

loga blogc d

≤ 0 ⇐⇒ (a − 1)(b − 1)(c − 1)(d − 1) ≤ 0, d �= 1 ili (12)

loga blogc d

≥ 0 ⇐⇒ (a − 1)(b − 1)(c − 1)(d − 1) ≥ 0, d �= 1 (12∗)

84 Matematicko-fizicki list, LIX 2 (2008. – 2009.)

Page 21: Matematicko-fizicki list Zagreb

Primjer 5. Rijesimo nejednadzbulogx(1 − x) · log2−x(2x) ≥ 0, x ∈ R. (13)

Rjesenje. Prema (10) i (13) dobivamox > 0, x �= 1, 1 − x > 0, 2 − x > 0, 2 − x �= 1, 2x > 0.

Ovi uvjeti su zadovoljeni zax ∈ 〈 0, 1〉 . (14)

Primjenom (11) nejednadzba (13) je ekvivalentna sljedecoj(x − 1)(1 − x − 1)(2 − x − 1)(2x − 1) ≥ 0, (15)

tj.x(1 − x)2(2x − 1) ≥ 0. (16)

Kako je (1 − x)2 > 0 za sve x ∈ 〈 0, 1〉 , (16) je ekvivalentno s nejednadzbomx(2x − 1) ≥ 0,

cije je rjesenje x ∈ 〈−∞, 0] ∪⟨

12, +∞

⟩, pa iz (14) dobivamo konacno rjesenje

x ∈[12, 1

⟩.

Zadaci za vjezbu

1. log2(4x) > 1, Rj. x > 0.

2. log3x(4x − 1) ≥ log3x(9x2 − 1) , Rj. x ∈fi

13,49

fl.

3. logx 2 · log2(4x) > 1, Rj. x > 1.

4. log2x+4 x2 < 2, Rj. x ∈fi−3

2,−4

3

fl.

5. logx+1|x| − 2|x| + 3

< 0, Rj. x > 2.

6. log 1x

x2

+ logxx4

> 0, Rj. x ∈ 〈 0, 1〉 .

7. log 3+x1−x

„1 + x2 − x

«> 0, Rj. x ∈

fi12, 1

fl.

8. logcos x sin x · logsin x cos x ≤ 0, Rj. nema rjesenja.

9. logsin x 4 · logsin2 x 2 > 0, Rj. kπ < x < π + kπ, x �= π2

+ 2kπ, k ∈ Z .

10.log2+x(1 + x)

log1+x x≥ 0, Rj. x > 1.

Matematicko-fizicki list, LIX 2 (2008. – 2009.) 85

Page 22: Matematicko-fizicki list Zagreb

LHC: zasto mu se divimo?

Tome Anticic i Vuko Brigljevic1 , Zagreb

Brzina je vazna

Fizika cestica istrazuje najmanje dijelove prirode i stanje svemira duboko u proslosti,samo nekoliko trenutaka nakon Velikog praska. Jedan od glavnih nacina za dobivanjelaboratorijskih uvjeta za stvaranje novih, tezih cestica i za ispitivanje ponasanja prirode navisokim temperaturama je sudaranjem visokoenergetskih cestica. Einsteinova jednadzbakoja povezuje masu i energiju objasnjava nam kako je to moguce:

E =1√

1 − v2

c2

· m · c2,

gdje su m , v i E masa, brzina i energija cestice, a c brzina svjetlosti. S malorazmisljanja moze se doci do zakljucka da priblizavanjem brzine v brzini svjetlosti c ,energija cestice raste prema beskonacnosti. Dakle, s vecim brzinama stvaraju se veceenergije, a time i uvjeti za proizvodnju cestica s vecom masom i proucavanje fizike navisim temperaturama.

Slika 1. Shematski prikaz LHC-a

1 Dr. Tome Anticic i dr. Vuko Brigljevic su visi znanstveni suradnici u Zavodu za eksperimentalnu fiziku Instituta“Ru -der Boskovic“ u Zagrebu.

86 Matematicko-fizicki list, LIX 2 (2008. – 2009.)

Page 23: Matematicko-fizicki list Zagreb

Puno komaraca u Velikom hadronskom sudarivacu

Najmocnije oru -de danasnjice za takvo ispitivanje materije je Veliki hadronski sudarivac(engl. Large Hadron Collider, skraceno LHC), smjesten oko 100 m ispod povrsinena svicarsko-francuskoj granici. Izgradnja LHC-a dovrsena je upravo ove godine, au proljece 2009. ocekuju se sudari na najvisim energijama za koje je konstruiran.Pridjev “veliki” je posve na mjestu: LHC ubrzava protone (i druge cestice) u kruznomtunelu s opsegom od cak 27 km, i kao takav je daleko najveci sudarivac na svijetu.Veliki je i zato sto, dijelom i zbog svojih impresivnih dimenzija, stvara najenergetskijelaboratorijske sudare. Naime, kroz LHC mogu kruziti dva snopa protona kroz odvojenecijevi u suprotnim smjerovima s energijama od cak 7 TeV ( 1 TeV = 1012 eV, a1 eV = 1.6 · 10−19 J, energija koju jedan elektron dobije kada je na potencijalu odjednog volta). Izrazeno brzinama, svaki proton ce se kretati brzinom jednakom cak0.999999991 brzine svjetlosti. Dakle, svake sekunde ce protoni u snopu cak 11 000 putaobici cijeli prsten LHC-a. Izrazeno svakodnevnim primjerom, ta energija je otprilikejednaka kinetickoj energiji leteceg komarca. Na prvi pogled to se i ne doima jakoimpresivno, ali treba imati na umu da je na LHC-u ta energija koncentrirana u prostorbilijun puta manji od dimenzije tog istog komarca.

Slika 2. Slikoviti prikaz stvaranja novih cestica (razno voce) iz energije

Jos impresivnija je energija po snopu. U svakom od njih je oko 3000 skupina cestica,koji se sastoje od cak 100 milijardi protona. To znaci da ce svaki snop (od dva) naLHC-u imati ukupnu energiju od oko 3.5 · 108 J, ekvivalentno 400 tonskom vlaku kojiputuje brzinom od 150 km/ sat!

Puno energije u Velikom hadronskom sudarivacu

LHC je dizajniran kao sudarivac, sto znaci da se ubrzane cestice sudaraju jedna udrugu, za razliku od inzinjerski puno jednostavnijeg i jeftinijeg rjesenja usmjeravanjasnopa na nepokretnu metu. Razlog tome je razlicit iznos kolicine energije u sustavucentra mase, to jest korisne kolicine energije za stvaranje novih cestica. U frontalnom

Matematicko-fizicki list, LIX 2 (2008. – 2009.) 87

Page 24: Matematicko-fizicki list Zagreb

sudaru cestica iz suprotnih pravaca energija se zbraja, tako da se dobiva 14 TeV-ana LHC-u za protonske snopove (snopovi teskih jezgara, poput olova, imat ce i vecuenergiju), dok se kod nepokretne mete korisna energija mijenja kao korijen od energijeudarajuce cestice, to jest za velike energije E ≈ √

2 · mproton · Esnop , pa se umjesto14 TeV-a dobiva tricavih 0.114 TeV-a.

Slika 3. Slicnosti u operaciji televizora i sudarivaca

Mocni magneti

Kretanje cestice kroz tunel kontrolira se pomocu dvije vrste magneta: dipola ikvadrupola. Dipolni magneti imaju, kao sto im ime i kaze, dva magnetska pola, izaduzeni su za drzanje cestica na kruznoj putanji. Naime, magnetsko polje ne povecavabrzinu cestice, vec je skrece, tako da cestica koja ide pod pravim kutom u odnosu napolje, putuje kruznom putanjom. Sto je veca energija koja se zeli postici, to dipolnimagneti (a ima ih 1232 na LHC-u) moraju proizvoditi jace magnetsko polje da bi moglicestice zadrzati unutar cijevi sudarivaca. Za energije koje se zele postici na LHC-u,magnetsko dipolno polje iznosi cak 8.3 T (tesla), sto zahtijeva supravodljive magnete ihla -denje pomocu helija u superfluidnom stanju (koji ima iznimno dobru vodljivost zatoplinu, ali je strahovito tezak za manipuliranje) na temperaturi od samo 1.9 K (kelvina),sto je hladnije cak i od dalekog Svemira (2.7 K). Ukupno ce u svakom trenutku magnetibiti hla -deni s cak 120 tona helija!

Da je LHC manji, magnetsko polje bi trebalo biti jos vece za uspostavu iste energijesnopa, sto je i razlog potrebe tunela sto veceg promjera:

p ≈ 0.3 · B · R,

gdje je p impuls cestice u GeV/ c2 (1 GeV = 109 eV) , B je polje u teslama, a Rpromjer u metrima.

Kvadrupolni magneti, s cetiri naizmjenicna pola, su zaduzeni za fokusiranje snopa.Zbog sprecavanja nepozeljnih sudara, snop se krece u cijevima u kojima je uspostavljenvakuum deset puta bolji od vakuuma na Mjesecu (Mjesec nema atmosferu).

88 Matematicko-fizicki list, LIX 2 (2008. – 2009.)

Page 25: Matematicko-fizicki list Zagreb

Slika 4. Pogled na dipolne magnete unutar LHC-tunela

Gubici energije u Velikom hadronskom sudarivacu

Uz magnete, LHC ima i elektromagnetske komore koje sluze najprije za ubrzavanjecestica i onda za drzanje istih na konstatnoj energiji kompenzirajuci gubitke energije.Operacija sudarivaca je ustvari na puno nacina slicna televizoru (onom starinskog tipa skatodnim cijevima), kao sto je objasnjeno na slici 3. Samo nekoliko cestica je prikladnoza ubrzavanje u sudarivacima. One moraju biti i stabilne i nabijene, sto izbor svodina protone, jezgre tezih elemenata ili elektrone. Spomenuti gubitak energije eliminiraelektrone kao cestice prikladne za LHC. Rijec je o takozvanom sinkrotonskom zracenju,kada nabijena cestica uslijed kretanja u kruznoj putanji gubi energiju emitirajuci fotone:

Esinkroton∞ 1R

(Esnop

m

)4.

Ovaj gubitak energije je ocito iznimno osjetljiv na masu cestice, i za teze cestice kaosto su protoni je puno manji (po jednom krugu rotacije) nego za oko 2000 puta lakseelektrone. Ako protoni energije 7 TeV na LHC-u gube po krugu 7 · 103 eV, izracunajtekoliko bi elektron izgubio sinkrotonskim zracenjem! Odgovor ce vam nedvosmislenoobjasniti zasto se elektroni ne rabe na LHC-u. Izbor dakle ostaje samo na protonima (itezim ionima) kao jedinim prikladnim cesticama.

Slika 5. Prosireni prikaz impresivnog CMS (od engl. Compact Muon Solenoid) detektora.Sicusna mrlja ispred CMS-a je ustvari prikaz covjeka u pravoj skali. CMS je napravljen,

izme -du ostalog, od cak 10 000 tona zeljeza, sto je vise nego u Eiffelovom tornju. Detektor je15 m visok, 21 m dugacak i ima masu od 12 500 tona.

Matematicko-fizicki list, LIX 2 (2008. – 2009.) 89

Page 26: Matematicko-fizicki list Zagreb

Protoni se klasificiraju kao hadroni, vrsta cestica karakterizirana kvarkovskom strukturom(vidi prvi okvir). Tako je konacno objasnjen i pridjev “hadronski” u LHC-u. Manaprotonskih sudara je upravo postojanje njihove strukture, zbog cega je samo dio energijekoristan za stvaranje novih cestica. U principu bi se na LHC-u mogli sudarati protonii antiprotoni, sto bi imalo jos vecu korisnu energiju. Me -dutim, protone je puno laksestvoriti i time je stvoreni broj sudara u sekundi (velicina proporcionalna tzv. luminozitetu)iznimno veci.

Okvir 1. Standardni model elementarnih cesticaElementarne cestice (fermioni)Postoje tri generacije elementarnih cestica. Cestice druge i trece generacije su u svemu slicneonima iz prve generacije osim po tome sto su teze. Tako -der su nestabilne i nakon kratkogvremena se raspadaju u cestice prve generacije. Materijalni svijet oko nas se sastoji iskljucivood cestica prve generacije, dok se one ostalih generacija stvaraju u prirodnim ili umjetnimvisokoenergetskim procesima i brzo raspadaju.

leptoni kvarkovi

elektricninaboj

−1 0 +2

3− 1

3generacija

elektrongradi atomzajedno sjezgrom

elektronski neutrinojako slabo me -dudjelujei moze proci krozcijelu zemlju a dase ne zaustavi,milijarde njih prolazekroz nas svakesekunde; ima jakomalu masu

gornji (Up)sastavni dioprotona ineutronazajedno sadown kvarkom

donji (Down)sastavni dioprotona ineutrona

1

miontezi ro -dakelektrona

mionski neutrinoslican elektronskomneutrinu, stvara sezajedno s mionimau nekim raspadima

carobni(Charm)tezi ro -dakup kvarka

strani (Strange)tezi ro -dakdownkvarka

2

taujos tezi,jakonestabilan

tau-neutrinootkriven tek 2000.godine kaozadnji od svihfermiona

vrsni (Top)najteza odsvih poznatihcestica, tezakkao atomzlata

dubinski (Bottom)jos tezi ro -dak downkvarka, vazan zaproucavanje asimetrijematerija-antimaterija

3

Kvarkovi se nikada ne nalaze sami. Uvijek su vezani u hadrone. Jedini stabilni hadroni suproton i neutron (i neutron zapravo samo ukoliko se nalazi u atomskoj jezgri, izvan nje seraspada nakon nekoliko minuta). Proton se sastoji od dva up-kvarka i jednog down-kvarka, aneutron od dva down-kvarka i jednog up-kvarka.Svi leptoni i kvarkovi su fermioni: njihov spin, to jest intrinsicna kutna kolicina gibanja, jevisekratnik Planckove konstante podijeljene s 2. Fermioni zadovoljavaju Paulijev princip: dvaidenticna fermiona ne mogu se nalaziti istodobno u istom stanju. Taj zahtjev je neophodanuvjet za bogatstvo struktura koje vidimo u svijetu, od atomske jezgre do kompleksnih DNKmolekula zivota.Za svaki fermion, kvark ili lepton, postoji i odgovarajuca anticestica, jednake mase, jednakogtrajanja zivota i spina, ali sa suprotnim elektricnim nabojem. Tijekom velikog praska nastajalesu jednake kolicine cestica i anticestica. Me -dutim, anticestice su potpuno nestale iz nasegsvemira, a razlog za to jos nije utvr -den.

90 Matematicko-fizicki list, LIX 2 (2008. – 2009.)

Page 27: Matematicko-fizicki list Zagreb

Divovski detektori za otkrivanje mikrosvijeta

Na vise mjesta na LHC-tunelu protoni iz suprotnih pravaca se fokusiraju unutardesetak mikrona i sudaraju. Oko tih lokacija izgra -deni su masivni detektorski sustavikoji detaljno analiziraju stvorene cestice i iz toga pokusavaju razumjeti fiziku kojase pri sudaru dogodila. Detektori su koncipirani slojevito: cestice koje proizlaze iztocke sudara prolaze kroz razne slojeve. Svaki od njih mjeri specificnu karakteristikuprolazecih cestica: kolicinu gibanja, elektricni naboj, energiju ili brzinu. Razne vrstecestica ce ostaviti razlicite “potpise” u detektoru. Dok ce neke kao elektroni ili fotoni bitizaustavljene vec u prvim slojevima detektora, druge ce, poput miona, proci kroz gotovocijeli detektor i jedini ostaviti trag u golemim mionskim komorama u vanjskom slojudetektora (vidi sliku 6). Svaki od tih divovskih eksperimenta je rezultat kolaboracijevise tisuca znanstvenika i tehnicara.

Slika 6. Presjek CMS detektora s raznim slojevima detektora.Prikazan je i odziv detektora na razne vrste cestica.

Carobne rijeci: Higgs, dodatne dimenzije, supersimetrija, tamna tvar,. . .

LHC je stvoren jer nase znanje Svemira nije konacno. Tzv. Standardni model cesticai sila ujedinjuje nase trenutno znanje fizike cestica, vidi okvir 1 i 2. Standardni modelje testiran na brojnim eksperimentima i bio je iznimno uspjesan u predvi -danju novihcestica i njihovih svojstava. Me -dutim, puno nerijesenih problema ostaje, i LHC jeupravo dizajniran da postigne energiju gdje se neki odgovori moraju pojaviti. Prvoveliko otvoreno pitanje je pitanje porijekla mase, koje je od kljucne vaznosti zasamu konzistentnost standardnog modela. U verziji u kojoj postoje samo vec poznatecestice, masa svih cestica bi treba biti jednaka nuli, sto ocito ne odgovara cinjenicama.Standardni model se moze nadograditi uvo -denjem Higgsovog polja, s cijom interakcijomcestice dobivaju masu (jeste li se kada upitali: zasto tijela imaju masu?). To poljepodrazumijeva i najmanje jednu jos neotkrivenu cesticu, tzv. Higgsov bozon. Teorijapredvi -da sve njegove karakteristike osim jedne, njegove mase. Me -dutim, iz ranijihmjerenja i teorijskih razloga znamo da mu masa mora biti veca od otprilike 120 i manjaod 200 vodikovih atoma. Ako postoji i ako mu je masa u tom podrucju, Higgsov bozonce nastati u proton proton sudarima na LHC-u i eksperimenti ce ga otkriti. OtkriceHiggosvog bozona je jedan od prvenstvenih ciljeva dva najveca LHC-eksperimenta,CMS-a i ATLAS-a.

Matematicko-fizicki list, LIX 2 (2008. – 2009.) 91

Page 28: Matematicko-fizicki list Zagreb

Me -dutim, otkrice Higgsovog bozona ne moze biti kraj price. Naime, standardnimodel cak i s Higgsovim poljem ne daje kompletnu sliku svih fundamentalnih sila, tesadrzi i puno proizvoljnih parametara, i ostavlja niz otvorenih pitanja: zasto se prirodaponavlja po tri puta (vidi okvir 1)? Krije li se dublji princip reda iza tri generacijaelementarnih cestica? Zasto su mase elementarnih cestica toliko razlicite jedna od druge(top kvark je vise od 300 tisuca puta tezi od elektrona)? Kako ukljuciti gravitaciju usliku? Jedna moguca nadogradnja tog modela je tzv. teorija supersimetrije, koja postuliratemeljnu simetriju izme -du bozona i fermiona, i koja predvi -da da svaka postojeca cesticaima i svog “superpartnera”. Ako je ta teorija tocna, LHC ce najvjerojatnije naci najlaksusupersimetricnu cesticu, koja je ujedno i glavni kandidat za objasnjenje tamne tvari, tj.23% gustoce energije svemira koja ne potjece od nama poznatih cestica (odgovornih zatek 4%). Postoje jos egzoticnije stvari koje ce mozda biti otkrivene, kao sto su dodatnedimenzije prostora, ili odstupanje zakona gravitacije od 1/r2 ovisnosti na jako malimskalama, sto bi moglo dovesti do jako zanimljivog fenomena produkcije mini crnih rupana LHC-u.

Okvir 2. Standardni Model elementarnih cesticaElementarne sile (bozoni)Sile su odgovorne za me -dudjelovanje izme -du elementarnih cestica. Bez njih ne bi bilostrukturirane tvari, reakcija, raspada. Danas poznajemo cetiri osnovne sile. Sile se prenoseputem izmjene cestica nosioca sila.

elektromagnetska slaba jaka gravitacija

osjetljive cestice kvarkovi i nabijenileptoni

svi kvarkovii leptoni

kvarkovi sve cestices masom

vezane pojave drzi elektrone okoatomskih jezgri,spaja atome umolekule, izasluzna jeza svojstvaplinova,tekucina, ikrutih tvari

uzrokradioaktivnosti

veze kvarkove uprotonu, neutronu idrugim hadronima,drzi atomsku jezgruna okupu i pri tomeje jaca od ogromneelektricne odbojnesile izme -duprotona

prva poznatasila, ali namikroskalijos uvijeknajlosijeshvacena;ne uklapa seu teorijskiopis standardnogmodela

nositelj sile foton W+ , W− , i Z 8 gluona graviton? (postulat)

Elektromagnetska i slaba sila su prepoznate kao razlicite manifestacije jednog istog principai sjedinjene u elektroslabu silu. U okviru standardnog modela opisane su elektroslaba i jakasila. Ukljucenje gravitacije u tu sliku ostaje jedan od velikih izazova fizike na pocetku 21.stoljeca.

Nosioci sila su bozoni, sto znaci da je njihov spin (vidi okvir 1) cjelobrojan (0,1,2,. . . )visekratnik Planckove konstante.

Potraga za novom fizikom: trazenje igle u puno plastova sijena

Kako ce fizicari naci Higgsov bozon, supersimetricne cestice ili neki drugi znak novefizike u LHC sudarima? Kao prvo treba napomenuti da je rijec o vrlo rijetkim procesimakoje ce fizicari trebati izdvojiti iz ogromne kolicine podataka. U srcu CMS i ATLASdetektora ce se svakih 25 nanosekundi krizati snopovi protona i u svakom krizanju cedoci do nekoliko proton-proton sudara koji ce ostaviti tragove u detektoru, dakle 40

92 Matematicko-fizicki list, LIX 2 (2008. – 2009.)

Page 29: Matematicko-fizicki list Zagreb

milijuna puta svake sekunde. Ocekuje se da ce stopa proizvodnje Higgsovog bozona,ovisno o teorijskom modelu, biti reda velicine jedan na dan (po svakom eksperimentu),sto znaci da se trazi jedan od 1013 doga -daja! Trazenje igle u plastu sijena u usporedbiizgleda djecjom igrom: ako je volumen tipicne iglice 5 mm3 , a volumen plasta sijena 50m3 , trazenje nove fizike u LHC sudarima odgovara trazenju jedne igle u tisucu plastovasijena! Pri tome je prvi izazov uopce sacuvati podatke zanimljivih sudara. Svaki sudarce u detektoru generirati otprilike 1 MB informacije, a 40 milijuna njih 40 TB svakesekunde! Takvu kolicinu podataka nije moguce pohraniti i od njih ce se samo 100 do 200najzanimljivijih sacuvati za daljnju analizu. Odbaceni doga -daji su zauvijek izgubljeni istoga fizicari moraju biti sasvim sigurni da smo sacuvali doga -daje koji sadrze tragoveHiggsovog bozona ili drugih oznaka jos nepoznate fizike. Za izbor najzanimljivijihdoga -daja je odgovoran veliki racunalni grozd sastavljen od nekoliko tisuca racunala kojiu nekoliko milisekundi mora analizirati svaki sudar trazeci oznake zanimljivih doga -daja.Tu fizicarima zakoni fizike ipak dolaze u pomoc i bitno im olaksavaju posao. U veciniproton-proton sudara ce se dva protona tek okrznuti i samo lagano rasprsiti, te ceprodukti tih sudara uglavnom letjeti jako blizu osi samog snopa. S druge strane, ako seu sudaru stvori tezi objekt poput Higgsovog bozona ili supersimetricne cestice, od kojihznamo iz rezultata ranijih eksperimenata da ukoliko postoje moraju biti tezi od otprilike100 atoma vodika, raspasti ce se u puno lakse vidljive ili nevidljive cestice. Iz vecspomenute relacije izme -du mase i energije slijedi da ce se masa teskog objekta u raspadupretvoriti u veliku kolicinu gibanja laganih produkata raspada. A zato jer je u principusvaki pravac leta produkata raspada moguc, iz takvih doga -daja se ocekuju cesto cestices velikim kutom rasprsenja prema pravcu snopa, ili velikom transverzalnom kolicinomgibanja u odnosu na pravac snopa. Taj jednostavan argument je kljuc izbora zanimljivihdoga -daja: traze se sudari s cesticama velike transverzalne energije ili kolicine gibanja.

Slika 7. Unutrasnja struktura protona: proton je sastavljen od dva donja i jednog gornjegkvarka. Ali unutar protona mozemo naci i gluone koji vezu kvarkove, a i dodatne parove

kvark-antikvark koji se mogu na trenutak stvoriti i ponovo nestati.

Kad su podaci sudara jednom snimljeni, postaju objekt duge i cesto kompliciraneanalize fizicara koji ce krenuti u potragu za oznakama nove fizike. Trazene nove cestice,npr. Higgsov bozon, se obicno nece mjeriti direktno u detektoru, jer su jako nestabilne.Raspasti ce se vrlo brzo nakon stvaranja i produkti njihovog raspada, obicno vec poznatecestice, su oni koji ce ostaviti vidljiv trag u raznim slojevima detektora. Ako je npr.masa Higgsovog bozona jednaka masi 150 do 200 atoma vodika, cesto ce se raspastiu 2 elektrona i 2 miona. Elektroni i mioni (mion je tezi ro -dak elektrona,vidi okvir 1)ostavljaju karakteristican trag u detektoru (vidi sliku 6). Iz mjerenja njihovih impulsamoze se jednostavno odrediti masa Higgsovog bozona. Pri tome se koristi samo zakonocuvanja energije i impulsa i relativisticka relacija izme -du mase, energije i impulsajedne cestice: E2 = p2c2 + m2c4 (ovo je jednadzba s pocetka clanka u drugacijemobliku). Mala komplikacija je takozvana pozadina: bit ce puno drugih doga -daja s 2elektrona i 2 miona u konacnom stanju, zapravo ce ih obicno biti za nekoliko redovavelicine vise od doga -daja s Higgsovim bozonom (signal). Tu se otvara prostor zamastu i kreativnost fizicara koji ce kroz lukavi odabir zahtjeva uspjeti odbaciti skorosve pozadinske doga -daje i sacuvati one s Higgs-signalom. Katkad ce se za tu proceduru

Matematicko-fizicki list, LIX 2 (2008. – 2009.) 93

Page 30: Matematicko-fizicki list Zagreb

koristiti slozenim alatima poput neuronskih mreza. Nova fizika se moze otkriti iako sestvaraju nove cestice koje nevidljivo pobjegnu iz detektora bez da ostave ikakav trag,ali mogu sa sobom nositi znacajnu kolicinu energije i kolicinu gibanja. To bi npr. bioslucaj za cesticu tamne tvari, koja samo vrlo slabo djeluje s normalnom materijom.Koristeci se ponovo zakonima ocuvanja kolicine gibanja i energije i cinjenicom dadetektor potpuno okruzuje tocku sudara moze se izmjeriti sveukupni impuls i energiju iprimijetiti da nesto nedostaje. Iz ostatka detektora moze se odrediti energija, a mozda imasa i druge osobine “bjegunca”. Mozemo dakle otkriti i nesto sto uopce nismo vidjeli,zakoni ocuvanja su stvarno vazni u fizici!

Konacno je vrijedno spomenuti jos jednu karakteristiku sudara proton-proton kojadodatno otezava mjerenja na LHC-u: okruzenje tih sudara je jako prljavo! Da bismorazumjeli o cemu je rijec moramo se sjetiti da proton nije jednostavan i elementaranobjekt. Sastoji se od tri kvarka, ali zapravo je i slozeniji od toga (vidi okvir 1). Uprotonu mozemo naci kvarkove, ali i gluone, pa cak i antikvarkove. Kad se u sudaruproton-proton stvara Higgsov bozon, ili par top-kvarkova (za njih znamo sigurno dapostoje!), to je produkt elementarne reakcije izme -du dvije elementarne cestice (kvark,antikvark ili gluon) iz dva protona. Ali tu su i ostali sastavni dijelovi protona, ostalikvarkovi i gluoni, koji nisu sudjelovali u “glavnom” procesu, u kojem igraju na jedannacin samo ulogu gledatelja. Nakon tog glavnog procesa, i kvarkovi-gledatelji ce letjetidalje i zakomplicirati konacno stanje s dodatnim cesticama. Nalazenje tragova iz raspadaneke nove cestice me -du svim tim cesticama (njih cesto vise od 100) predstavlja dodatniizazov u fascinantnom intelektualnom pothvatu potrage za novom fizikom

Osim sudara proton-proton, sudarima iona olova LHC ce tako -der proucavati i stanjematerije koja je postojala nekoliko mikrosekundi nakon velikog praska, tzv. kvarkovsko-gluonske plazme. U tom stanju kvarkovi i gluoni su se slobodno kretali u iznimno vrucoji gustoj juhi, za razliku od danas kada ih se samo moze naci u cesticama kao sto suprotoni ili neutroni. U sudarima olovo-olovo stvorena temperatura bit ce vise od 100 000puta veca nego ona u sredistu Sunca. Eksperiment optimiziran za ova istrazivanja jeALICE. Pomocu cetvrtog eksperimenta na LHC-u koji se zove LHC-b, pokusat ce seodgonetnuti tajne asimetrije tvari u svemiru, to jest naci odgovor na pitanje zasto nemajednake kolicine materije i antimaterije u svemiru.

Eksperimenti na CERN-u ukljucuju i po vise tisuca znanstvenika iz cijelog svijeta(vecinom fizicara) koji ce imati pune ruke posla sljedecih 10 do 20 godina da pokusajuiz dobivenih podataka odgonetnuti sto vise tajni Svemira. I desetak hrvatskih fizicaraje ukljuceno u taj pothvat. Mozete ih kontaktirati ako mozda i vi zelite biti dio ovognajveceg znanstvenog projekta u povijesti. . .

� � �

Nadopuna. U clanku J. Siftara Abelova nagrada 2008. g. – John G. Thompson iJacques Tits u br. 1/ 233 ispustene su slike dobitnika nagrade, pa ih sada donosimo.

John G. Thompson Jacques Tits

94 Matematicko-fizicki list, LIX 2 (2008. – 2009.)

Page 31: Matematicko-fizicki list Zagreb

Geometrija i problemi kretanja

Ljiljana Sudar, Leskovac

Neki fizicki problemi mogu se rijesiti na vise nacina, primjenom razlicitih podrucjamatematike. Problemi kretanja se, npr. najcesce rjesavaju algebarski. Taj nacin rjesavanjazahtijeva za dano kretanje poznavanje eksplicitne ovisnosti brzine i prije -denog puta odvremena.

Me -dutim, i bez poznavanja te eksplicitne ovisnosti moguce je i efikasnije i elegantnijerijesiti mnoge probleme kretanja pomocu nestandardne, veoma mocne metode, koja sebazira na primjeni grafova i geometrije.

Ukoliko se pri kretanju intenzitet brzine za isti vremenski interval uvijek mijenjaza istu vrijednost (tj. ubrzanje je konstantno), samo na osnovu grafova i elementarnogpoznavanja geometrije mogu se postaviti relacije iz kojih se dobiva rjesenje problema.

Slika 1. Grafovi ovisnosti brzine o vremenu

Ako se intenzitet brzine za svaki mali vremenski interval Δt uvijek povecava za istuvrijednost Δv (tj. ubrzanje je konstantno), graf brzine je pravac koji zaklapa ostar kut

α s pozitivnim dijelom t -osi (sl. 1a). S grafa se vidi: tgα =ΔvΔt

=v2 − v1

t2 − t1= a ,

gdje je a intenzitet ubrzanja. Ako se kroz tocku K koja je srediste duzine DCpovuce pravac paralelan t -osi, pravokutni trokuti DD1K i KC1C su identicni, paimaju jednake povrsine. Zato ce osjencana povrsina trapeza ABCD (u oznaci P) bitijednaka povrsini pravokutnika ABC1D1 (PABC1D1) osnovice Δt = t2 − t1 i visine v′ tj.P = PABC1D1 = v′Δt , pa kako je v′Δt elementarni prije -deni put ΔS za vrijeme Δt , bitce P = ΔS tj. povrsina ispod odgovarajuceg dijela grafa brzine predstavlja elementarniprije -deni put.

Ako se intenzitet brzine za svaki mali vremenski interval Δt uvijek smanjuje zaistu vrijednost Δv (tj. ubrzanje je konstantno), graf brzine je pravac koji zatvaratupi kut α s pozitivnim dijelom t -osi (sl. 1b). S grafa je ocigledno α = π − β i

tgα = tg(π − β) = − tgβ = −ΔvΔt

= −a , gdje je a intenzitet ubrzanja, pa slijedi

tgβ = a , gdje je β kut koji pravac zatvara s negativnim dijelom t -osi. Analognose, kao u prethodnom slucaju, moze pokazati da je elementarni prije -deni put tijela zavrijeme Δt (ΔS) osjencana povrsina trapeza ABCD (P) s osnovicama AD i BC i

visinom Δt = t2 − t1 , tj. ΔS = P =|AD| + |BC|

2Δt .

Matematicko-fizicki list, LIX 2 (2008. – 2009.) 95

Page 32: Matematicko-fizicki list Zagreb

Ako se intenzitet brzine tijekom vremena ne mijenja, pravac je paralelan s t -osi(sl. 1c). Prije -deni put tijela za vrijeme Δt (u oznaci ΔS) je, ocigledno, osjencanapovrsina pravokutnika ABCD (P) osnovice Δt i visine v , tj. ΔS = P = Δt · v .

Zbroj elementarnih prije -denih putova je ukupan prije -deni put tijela i predstavljapovrsinu ispod odgovarajuceg dijela grafa brzine.

Metoda rjesavanja problema kretanja pomocu grafa, primjenom geometrije zahtijeva,osim dobrog razumijevanja grafickog prikaza problema kretanja, i to imajuci u vidu:

1) da je povrsina ispod grafa brzine prije -deni put tijela i

2) da je nagib pravca, kojom se prikazuje ovisnost brzine tijela o vremenu, definiranubrzanjem tijela koje se krece.

Slijede primjeri koji ilustriraju ovu metodu.

Primjer 1. Prvu polovinu puta tijelo prije -de za dvostruko dulje vrijeme nego drugupolovinu. Srednja brzina duz cijelog puta je 6 m/ s. Kolika je srednja brzina na prvoj, akolika na drugoj polovini puta? [2]

Rjesenje.

Dan je graf brzine tijela u ovisnosti o vreme-

nu. Drugu polovinu puta s2 =s2

tijelo prelazi

brzinom v2 za vrijeme t . Prvu polovinu puta

s1 =s2

tijelo prelazi brzinom v1 za vrijeme 2t

(tj. dva puta dulje vrijeme nego drugu polovinuputa), pa je ukupno vrijeme kretanja tijela 3t icitav put

s = s1 + s2. (1)

Put s1 jednak je povrsini pravokutnika OABC ,

osnovice |OA| = 2t i visine |OC| = v1 , (u oznaci POABC ), tj.

s1 =s2

= POABC = v1 · 2t. (2)

Put s2 jednak je povrsini pravokutnika ADEF , osnovice |AD| = t i visine |AF| = v2 ,(u oznaci PADEF) , tj.

s2 =s2

= PADEF = v2 · t. (3)

Citav put s tijelo bi preslo i da se sve vrijeme (3t) kretalo srednjom brzinom vsr = 6(u m/ s), pa je s grafa ocigledno da bi prije -deni put s u tom slucaju bio jednak povrsinipravokutnika ODGH , osnovice |OD| = 3t i visine OH = vsr , (u oznaci PODGH) , tj.

s = PODGH = vsr · 3t = 18t. (4)

Iz relacija (2) i (3) dobivamo2v1 = v2. (5)

Zamjena (2), (3) i (4) u (1), uz uvjet (5) daje v1 = 4.5 i v2 = 9 (u m/ s).

Primjer 2. Ivan i Luka hodaju s istog mjesta u suprotnim smjerovima oko nogometnogigralista koje je dugacko 110 m, a siroko 70 m. Luka hoda brzinom 5 m/ s, a Ivanbrzinom 4 m/ s. Koliko metara ce svaki od njih prevaliti kad se prvi put sretnu? Nakonkoliko vremena ce to biti? [3]

96 Matematicko-fizicki list, LIX 2 (2008. – 2009.)

Page 33: Matematicko-fizicki list Zagreb

Rjesenje.

Na sl. a) su dani grafovi brzina Luke (L) i Ivana (I) u ovisnosti o vremenu.Do trenutka (t) kad se prvi put sretnu Luka je presao put (sL) jednak povrsini

pravokutnika OACD , osnovice |OA| = t i visine |OD| = 5 (u m/ s), (u oznaci POACD ),pa je

sL = POACD = 5t. (1)Ivan je do prvog susreta presao put jednak povrsini pravokutnika OABE , osnovice|OA| = t i visine |OE| = 4 (u m/ s), (u oznaci POABE ), pa je

sI = POABE = 4t. (2)U trenutku susreta (t) je zbroj prije -denih putova Luke i Ivana jednak opsegu igralista(O) duzine a = 110 m i sirine b = 70 m (vidi sl. b), tj.

sL + sI = O. (3)Kako je opseg igralista O = 2a + 2b = 360 (u m), kao i relacija (1), (2) i (3) dobivase: 9t = 360, tj. t = 40 (u s) sto znaci da je vrijeme prvog susreta Luke i Ivana 40 snakon istovremenog polaska s istog mjesta (na sl. b) to mjesto je vrh M pravokutnika;Luka iz njega krece oko nogometnog igralista u smjeru kretanja kazaljki na satu, a Ivanu suprotnom smjeru, na sto ukazuju strelice). Na osnovu relacije (1), Luka je do prvogsusreta presao put sL = 5t = 200 (u m), a na osnovu relacije (2) Ivan je presao putsI = 4t = 160 (u m).

Primjer 3. Padobranac skace iz aviona te nakon 3 s otvara padobran. Nakon otvaranjapadobrana on naglo (trenutno) usporava te nastavlja padati brzinom od 5.4 m/ s. Desetsekundi nakon njegovog skakanja iz aviona skace drugi padobranac. Nakon kolikovremena on mora otvoriti padobran da bi zajedno stigli do tla? Pretpostavite da je otporzraka prije otvaranja padobrana zanemariv. [3]

Rjesenje.

Grafovi I i II daju ovisnost brzina od vremena oba padobranca. Nulti trenutakvremena je onaj kada skace prvi padobranac. Prve tri sekunde on slobodno pada tj.krece se s ubrzanjem a = g , a zatim konstantnom brzinom 5.4 (u m/ s). Nakon desetsekundi iskace i drugi. On slobodno pada tj. krece se s ubrzanjem a = g sve do trenutka

Matematicko-fizicki list, LIX 2 (2008. – 2009.) 97

Page 34: Matematicko-fizicki list Zagreb

t kada sustize prvog padobranca, otvara padobran pa njegova brzina trenutno postaje 5.4(u m/ s) (vidi graf). Od trenutka t oba padobranca se krecu brzinom 5.4 (u m/ s) svedok istovremeno ne stignu do tla.

Zbog istog ubrzanja padobranaca dok slobodno padaju (a1 = a2 = g) , nagib duzinaOB i CE prema pozitivnom dijelu t -osi je isti, pa iz pravokutnih trokuta ΔOAB iΔCDE imamo

v1

3=

v2

t − 10= g. (1)

Prije -deni put prvog padobranca, s1 , je u trenutku sustizanja (t) jednak zbroju povrsinatrokuta ΔOAB , osnovice |OA| = 3 (u s) i visine |AB| = v1 , i pravokutnika ADFG ,osnovice |AD| = t − 3 i visine |DF| = 5.4 (u m/ s), tj.

s1 =3v1

2+ 5.4(t − 3). (2)

Prije -deni put drugog padobranca, s2 , je u trenutku sustizanja (t) jednak povrsini trokutaΔCDE , osnovice |CD| = t − 10 i visine |DE| = v2 , tj.

s2 =v2(t − 10)

2. (3)

Prije -deni putovi padobranaca su jednaki u trenutku sustizanja (t) tj.s1 = s2 (4)

pa je prema relacijama (2) i (3):

3v1

2+ 5.4(t − 3) =

v2(t − 10)2

. (5)

Zamjenom relacije (1) u (5) dolazi se do kvadratne jednadzbe:

t2 − t(20 +

10.8g

)+ 91 +

30.24g

= 0.

Ako se uzme g = 9.81 (u m/ s2 ), kvadratna jednadzbe glasi:

t2 − 21.1t + 94.08 = 0.

Njena resenja su: t1 = 14.7 i t2 = 6.4 (u s). Drugi padobranac otvara padobran nakont − 10 sekundi (vidi sliku), pa je ocigledno da fizicki smisao ima samo drugo rjesenjeza koje je t − 10 = 4.7 (u s).

Primjer 4. Kuriri iz mjesta A i B krecu jedan drugome u susret, pri cemu se svakikrece jednoliko, ali razlicitom brzinom u odnosu na onog drugog. Posto su se sreli, dabi stigli u nasuprotna mjesta, jednome je potrebno jos 16, a drugome 9 sati. Kolikoje potrebno vremena jednom, a koliko drugom, da prije -du citav put izme -du A i B?(Zadatak Lewisa Carrolla, engleskog matematicara i pisca knjige za djecu “Alisa uzemlji cudesa”.) [4]

Rjesenje.

98 Matematicko-fizicki list, LIX 2 (2008. – 2009.)

Page 35: Matematicko-fizicki list Zagreb

Dan je graf brzina oba kurira u ovisnosti o vremenu. Ne smanjujuci opcenitostrjesavanja mozemo pretpostaviti da je brzina vA kurira iz mjesta A veca od brzine vBkurira iz mjesta B tj. vA > vB . Nulti trenutak vremena je onaj kada se kuriri pocinjukretati jedan drugome ususret, a t je trenutak njihovog susreta. U trenutku susreta (t)zbroj prije -denih putova kurira jednak je udaljenosti d mjesta A i B (vidi sl. b)) tj.

sA + sB = d. (1)

Kurir iz A je do susreta presao put sA , jednak povrsini pravokutnika OKMN ,osnovice |OK| = t i visine |ON| = vA , (u oznaci POKMN ), pa je

sA = POKMN = vAt. (2)Kurir iz B je do susreta presao put sB , jednak povrsini pravokutnika OKLP , osnovice|OK| = t i visine |OP| = vB , (u oznaci POKLP ), pa je

sB = POKLP = vBt. (3)Zamjenom relacija (2) i (3) u (1) dobiva se

vAt + vBt = d. (4)Kako je vA > vB kurir iz A je manje udaljen od mjesta B nego kurir iz B od mjesta A ,pa ce po uvjetu zadatka njemu trebati 9 sati da stigne u B , dok ce kuriru iz B trebati16 sati da stigne u A . S grafa je ocigledno da kurir iz A stize u B u trenutku t + 9,presavsi put jednak udaljenosti d mjesta A od mjesta B , sto na grafu odgovara povrsinipravokutnika OQRN, osnovice |OQ| = t + 9 i visine |ON| = vA , (u oznaci POQRN) , tj.

d = POQRN = vA(t + 9). (5)Kurir iz B stize u A u trenutku t + 16 presavsi put jednak udaljenosti d mjesta B odmjesta A , sto na grafu odgovara povrsini pravokutnika OSTP , osnovice |OS| = t + 16i visine |OP| = vB , (u oznaci POSTP ) tj.

d = POSTP = vB(t + 16). (6)

Iz relacija (5) i (6) mozemo izraziti brzine: vA =d

t + 9i vB =

dt + 16

.

Njihovom zamjenom u relaciju (4) dolazi se do kvadratne jednadzbe:d

t + 9t +

dt + 16

t = d tj. t2 = 144 odnosno t = 12 (u h).

Kurir iz A krecuci se brzinom vA preci ce citav put od A do B za vrijeme:t + 9 = 21 (u h)

a kurir iz B krecuci se brzinom vB preci ce citav put od B do A za vrijeme:t + 16 = 28 (u h).

Primjer 5. Tijelo je baceno brzinom 20 m/ s vertikalno uvis. Na visini 15 m nalazise horizontalna ploca od koje se tijelo savrseno elasticno odbije. Nakon koliko vremena(od trenutka izbacaja) se tijelo vrati natrag? (g = 9.81 ms−2) . [5]

Rjesenje: Graf predstavlja ovisnost brzinetijela od vremena. Nulti trenutak vremena jetrenutak izbacaja tijela vertikalno uvis. Brzinatijela se tokom kretanja navise smanjuje i utrenutku t1 , kada tijelo ima brzinu v1 , udara uhorizontalnu plocu. Prema uvjetu zadatka udartijela o plocu, koja je na udaljenosti d = 15 mod mjesta izbacaja tijela, savrseno je elastican

Matematicko-fizicki list, LIX 2 (2008. – 2009.) 99

Page 36: Matematicko-fizicki list Zagreb

pa brzina v1 mijenja samo smjer dok njezin intenzitet ostaje isti. Tijelo se nakon udarao plocu krece nanize i u trenutku t2 vraca na mjesto izbacaja. Prije -deni put tijelavertikalno uvis do udara u plocu (s1) jednak je udaljenosti ploce od mjesta izbacaja (d)tj. s1 = d = 15 (u m).

Na grafu tom putu odgovara povrsina pravokutnog trapeza OABC , s osnovicama|OC| = 20 (u m/ s) i |AB| = v1 i visinom |OA| = t1 , (u oznaci POABC) , pa je

s1 = POABC =20 + v1

2t1 = 15 (u m). (1)

Na grafu prije -denog puta tijela vertikalno dolje do mjesta izbacaja (s2) odgovarapovrsina pravokutnog trapeza ADEB , s osnovicama |AB| = v1 i |DE| = v2 i visinom|AD| = t2 − t1 = Δt , (u oznaci PADEB) , pa je

s2 = PADEB =v2 + v1

2Δt = 15 (u m). (2)

Ubrzanje tijela prema gore i prema dolje je gravitacijsko (g = 9.81 m/ s2) i odre -dujenagib duzine BC prema negativnom dijelu t -osi i nagib duzine BE prema t -osi (vidisliku) pa je: FBC = KBE i vrijedi

g =20 − v1

t1=

v2 − v1

Δt. (3)

Ako oko duzine AB rotiramo pravokutni trapez ADEB za 180◦ , on ce prijeci u osjencanitrapez AD1E1B s osnovicama |AB| = v1 i |D1E1| = v2 i visinom |AD1| = |AD| = Δti imat ce, naravno, istu povrsinu kao i trapez ADEB . Sa slike je ocigledno razlikapovrsina pravokutnog trapeza OABC i osjencanog trapeza AD1E1B tako -der povrsinapravokutnog trapeza OD1E1C s osnovicama |OC| = 20 (u m/ s) i |D1E1| = |DE| = v2i visinom |OD1| = t1 − Δt tj.

POABC − PADEB =20 + v2

2(t1 − Δt). (4)

Zamjenom relacija (1) i (2) u (4) dobiva se:

0 =20 + v2

2(t1 − Δt) odakle je t1 = Δt (5)

pa iz (3) dobivamo v2 = 20 (u m/ s).Zato graf brzine tijela u ovisnosti od vremena, zapravo, izgleda ovako.

S grafa i iz relacije (5) se dobivat2 = t1 + Δt = 2t1. (6)

S grafa su ocigledne i relacije:

g =20 − v1

t1i (7)

POABC =20 + v1

2t1 = 15. (8)

Iz relacija (7) i (8) se dobiva kvadratna jednadzba: gt21 − 40t1 + 30 = 0 cija surjesenja: t′1 = 0.99 (u s) i t′′1 = 3.09 (u s). Rjesenje t′′1 = 3.09 nema fizicki smisaojer se iz (7) za njega dobiva negativna vrijednost za velicinu brzine, sto je nemoguce(v1 = 20 − gt1 = −10.3) . Na osnovu rjesenja t′1 = 0.99 (u s) i relacije (6) dobiva seproteklo vrijeme od trenutka izbacaja do povratka tijela

t2 = 2t′1 = 2 · 0.99 = 1.98 (u s)Ova nestandardna metoda rjesavanja problema kretanja moze se veoma uspjesnoprimijeniti i na sve vrste hitaca (vertikalan, kosi, horizontalan), kao i na slaganjekretanja, pa se naizgled jako teski zadaci mogu lako i brzo rijesiti.

100 Matematicko-fizicki list, LIX 2 (2008. – 2009.)

Page 37: Matematicko-fizicki list Zagreb

Zadaci za vjezbu

1. Tokom seste sekunde, krecuci se jednoliko usporeno, motociklista prije -de put od 3 m.Ubrzanje kojim se pri tome kretao ima intenzitet od 2 m/ s2 . Kolika mu je pocetna brzina? (Rj.14 m/ s.) [2]

2. Balon se krece vertikalno uvis stalnom brzinom 5 m/ s. Poslije 20 s od balona se odvojikuglica.

a) Kolika je najveca visina do koje ce kuglica dospjeti? (Rj. 101.27 m.)

b) Poslije kojeg vremena ce kuglica pasti na Zemlju ako se ono mjeri od trenutka odvajanjaod balona? (Rj. 5.05 s.)

c) Kolika je srednja brzina kuglice na citavom putu? (Rj. 8.08 m/ s.) [2]

3. Ako se pocetna brzina tijela poveca 1.2 puta, pri nepromijenjenom usporenju, zaustavniput se poveca za 22 m. Koliki je bio zaustavni put prije navedene promjene? (Rj. 50 m.) [2]

4. Cestica, koja izlijece iz nekog izvora, najprije prelazi 200 m stalnom brzinom v , a zatimusporava s 2 m/ s2 . Pri kojoj je vrijednosti brzine v minimalno vrijeme kretanja cestice od

izlijetanja iz izvora do zaustavljanja? (Rj. 20 m/ s; tmin = 2

rlg

). [6]

5. Tri kuglice se puste slobodno padati, svaka sljedeca sa zakasnjenjem od jedne sekunde.Kolike su njihove me -dusobne udaljenosti pet sekundi nakon pustanja prve? (Rj. 44.145 m;34.335 m; 78.48 m.) [1], [2]

Literatura

[1] LJILJANA SUDAR, Geometrija pomaze fizici, Zbornik sa XXVI republickog seminara onastavi fizike, Drustvo fizicara Srbije, Vrnjacka Banja 2008., str.123–130.

[2] JEVREM JANJIC, MIROSLAV PAVLOV, BRANKO RADIVOJEVIC, Fizika za I razred srednjegobrazovanja i vaspitanja, Naucna knjiga, Beograd 1987. str. 129–132.

[3] Zadaci i rjesenja – rjesenja iz fizike, zadatak OS–259., zadatak 1357., MFL 1/ 229, god.LVIII, Zagreb 2007.–2008., str. 45, 46.

[4] MILAN SARIC, BOGOLJUB MARINKOVIC, Razliciti nacini resavanja zadataka II, problemikretanja, Materijali za mlade matematicare sv. 83 (10), Arhimedes–R. Kasanin, Beograd–Beli Manastir 1995., str. 16.

[5] Kvalifikacijski ispiti, zadatak F–25., MFL 2/ 218, god. LV, Zagreb 2004.–2005., str. 126.

[6] NATASA CALUKOVIC, MILAN RASPOPOVIC, Fizika 1M, Krug, Beograd 2001., str. 9.

� � �

Bangov teorem

Sve strane tetraedra imaju isti opseg ako i samo ako suone me -dusobno sukladne.

Sve strane tetraedra imaju jednake povrsine ako i samoako su one me -dusobno sukladne.

Matematicko-fizicki list, LIX 2 (2008. – 2009.) 101

Page 38: Matematicko-fizicki list Zagreb

Teleskop MAGIC – carobni instrument astrocesticne fizike

Dario Hrupec1 , Koprivnica

Astrocesticna fizika

Astrocesticna je fizika novo, interdisciplinarno, znanstvenoistrazivacko podrucje kojeukljucuje fiziku cestica, astronomiju i astrofiziku te kozmologiju (dijelom tako -der,nuklearnu fiziku i gravitaciju). Ona istrazuje, izme -du ostalog, daleke i egzoticne objektepoput pulsara2 i supermasivnih crnih rupa. U blizini takvih objekata postoje polja(elektricno, magnetsko i gravitacijsko) ekstremne jakosti koja uzrokuju ubrzavanjecestica do vrlo visokih energija te emisiju razlicitih valova (elektromagnetskih igravitacijskih). Cestice i valove, koji do -du do Zemlje i ondje budu opazeni, nazivamoprenositeljima (engl. messenger). Istovremeno opazanje razlicitih prenositelja koji dolazeiz istog kozmickog objekta, sto je danas u astronomiji trend, nazivamo kombiniranimpristupom (engl. multimessenger approach) [1].

Gama-astronomija

Jedna od najuspjesnijih grana astrocesticne fizike danas je gama-astronomija. Onaistrazuje kozmicke gama-zrake, odnosno elektromagnetsko zracenje najvisih energija kojedolazi iz svemira [2]. Nize energijsko podrucje kozmickih gama-zraka (do par desetakaGeV3 ) dostupno je, neposredno, samo detektorima na satelitima. Trenutno je u orbitinekoliko satelita koji opazaju kozmicke gama-zrake, no najveci i najperspektivniji me -dunjima je nedavno lansirani satelit Fermi (bivsi GLAST) [3]. Vise energijsko podrucjekozmickih gama-zraka (od par desetaka GeV do par desetaka TeV) dostupno je,posredno, detektorima na Zemlji. Visokoenergijske gama-zrake (kao i visokoenergijskenabijene cestice, takozvane kozmicke zrake) izazivaju u atmosferi velike pljuskovesekundarnih cestica. Nabijene sekundarne cestice u pljusku, cija je brzina veca odbrzine svjetlosti4 u zraku, emitiraju posebnu vrstu svjetlosti – Cerenkovljevo zracenje.Cerenkovljevo zracenje iz pljuska sekundarnih cestica u atmosferi moguce je opaziti

1 Autor je visi asistent u Institutu “Ru -der Boskovic” u Zagrebu, e-mail: dario.hrupecirb.hr2 Pulsar je brzorotirajuca neuronska zvijezda, kozmicki objekt ogromne gustoce, sredisnji ostatak eksplozijesupernove.3 GeV je gigaelektronvolt, 109 eV, a TeV teraelektronvolt, 1012 eV.4 Brzina svjetlosti u vakuumu, c , granicna je brzina u prirodi. Me -dutim, kroz prozirnu tvar (npr. staklo, vodu

ili zrak) svjetlost putuje sporije, brzinom v =c

n, gdje je n indeks loma tvari. Cestica se kroz tvar moze gibati

brzinom koja je veca od v , ali jos uvijek manja od c .

102 Matematicko-fizicki list, LIX 2 (2008. – 2009.)

Page 39: Matematicko-fizicki list Zagreb

posebnom vrstom teleskopa – Cerenkovljevim teleskopom. Danas postoji nekolikosustava Cerenkovljevih teleskopa, H.E.S.S. u Namibiji [4], MAGIC na Kanarima [5],VERITAS u Arizoni te CANGAROO III u Australiji. Cerenkovljevi teleskopi su iznimnouspjesni znanstveni instrumenti koji gotovo svakodnevno donose nova otkrica. Vodecuulogu u zemaljskoj gama-astronomiji danas drze dva europska5 sustava teleskopa,MAGIC i H.E.S.S. Kolaboracije MAGIC i H.E.S.S. tako -der zajednicki rade na buducemvelikom sustavu, od par desetaka do mozda i stotinu, Cerenkovljevih teleskopa CTA(Cherenkov Telescope Array) koji bi trebao proraditi za pet do deset godina [6].

Slika 1. Teleskop MAGIC na kanarskom otoku La Palmi. Izvor: Robert Wagner, MPI, Munchen.

Uvod u teleskop MAGIC

Teleskop MAGIC (Major Atmospheric Gamma Imaging Cherenkov) najveci jeCerenkovljev teleskop, promjera 17 m, i trenutno jedan od znanstveno najproduktivnijihinstrumenata iz podrucja astrocesticne fizike. Smjesten je na vrhu kanarskog otoka LaPalma, na visini 2200 m, unutar opservatorija Roque de los Muchachos. Teleskop jedovrsen krajem 2003. godine i opaza punim kapacitetom od 2004. godine. Pri samomje zavrsetku drugi teleskop, MAGIC II, koji bi trebao proraditi pocetkom 2009. godine.Oba ce teleskopa moci raditi samostalno ili u paru. Kolaboracija MAGIC broji oko 150clanova, cak iz 17 institucija. Od ove godine clanica kolaboracije MAGIC je i hrvatskagrupa (Croatian MAGIC Consortium) od sest fizicara s Instituta “Ru -der Boskovic”,Sveucilista u Splitu i Sveucilista u Rijeci. Za razliku od optickih teleskopa, Cerenkovljevteleskop nema cjeloviti reflektor nego segmentirani, sastavljen od mnostva manjih zrcala,kako bi se relativno jeftino dobila sto veca sabirna povrsina. Teleskop MAGIC imaukupnu povrsinu reflektora od 236 m2 . Nadalje, kamera Cerenkovljevog teleskopa nesastoji se od ure -daja CCD (Charge-Coupled Device), kao kod vecine optickih teleskopa,nego od fotomultiplikatora6 . To su, zasad, jedini prikladni fotosenzori za biljezenje vrlokratkotrajnih bljeskova Cerenkovljeve svjetlosti. Kamera teleskopa MAGIC II sadrzavatce, po prvi put u povijesti Cerenkovljevih teleskopa, napredne fotosenzore tipa HPD(Hybrid Photo-Detector). U razvoju fotosenzora HPD vaznu su ulogu odigrali i hrvatskifizicari, posebice prof. dr. Daniel Ferenc sa Sveucilista u Kaliforniji, Davis.

5 Oba su sustava zemljopisno smjestena na africkom kontinentu, zbog klime, no vode ih europske kolaboracije.6 Fotomultiplikator je iznimno osjetljiv detektor svjetlosti koji moze opaziti cak pojedinacne cestice svjetlosti,fotone. Posebne elektrode u cijevi, takozvane dinode, djeluju kao pojacivac, ili multiplikator, signala.

Matematicko-fizicki list, LIX 2 (2008. – 2009.) 103

Page 40: Matematicko-fizicki list Zagreb

Ciljevi i rezultati teleskopa MAGIC

Popis ciljeva teleskopa MAGIC iznimno je opsezan. U prvom redu tu suizvangalakticki objekti i pojave: aktivne galakticke jezge i provale gama-zracenja(engl. gamma ray burst, GRB), zatim galakticki objekti: pulsari, dvojni sustavikompaktnih kozmickih objekata7 te ostaci supernova. Vezano uz kozmologiju, me -duciljevima su primjerice izvangalakticka pozadinska svjetlost te, danas jako aktualna,tamna tvar.

Slika 2. Umjetnicki prikaz aktivne galakticke jezgre. Vidi se akrecijski disk i dva relativistickamlaza. U samom sredistu je supermasivna crna rupa. Izvor: NASA.

Popis dosad ostvarenih rezultata je velik i izvan opsega ovog teksta. Me -du vaznijimotkricima, koja su nedavno objavljena u uglednom casopisu Science, mogu se spomenuti:otkrice visokoenergijskih gama-zraka iz udaljenog kvazara 3C2798 te otkrice pulsnoggama-zracenja iz pulsara Crab9 .

Literatura

[1] D. HRUPEC, Kombinirani pristup u astronomiji, MFL LVI 4 (2005/ 2006)[2] D. HRUPEC, Gama-astronomija – posljednji elektromagnetski prozor u svemir, MFL LVI

1 (2005/ 2006)[3] http://fermi.gsfc.nasa.gov/[4] http://www.mpi-hd.mpg.de/hfm/HESS/[5] http://magic.mppmu.mpg.de/[6] http://www.cta-observatory.org/

7 Kompaktni kozmicki objekti su objekti vrlo velike gustoce, bijeli patuljci, neutronske zvijezde ili crne rupe,koji nastaju nakon smrti obicnih zvijezda. Nase Sunce ima relativno malu masu pa ce, kad potrosi sve svojenuklearno gorivo, zavrsiti kao bijeli patuljak za otprilike pet milijardi godina.8 Objavljeno u casopisu Science, 27. lipnja 2008.9 Objavljeno u ScienceXpress, 16. listopada 2008.

104 Matematicko-fizicki list, LIX 2 (2008. – 2009.)

Page 41: Matematicko-fizicki list Zagreb

Profesor Strogi bio je toga dana iznimnodobrog raspolozenja. Poceo je veselo:

— Proslo je vec sedam godina ovogastoljeca, a i osma je pri kraju. Danas cemoih malo srediti.

— Strogi nas godinama “sre -duje”.Bojim se da ce nas i ovaj puta punosrediti! — culo se otpozadi.

— Cuo sam vas! Ali danas nemamvremena za prodiku. Treba raditi. Evo, naploci pisem sve brojke po velicini. Vasje zadatak da osam puta izme -du nekihbrojki stavite znakove racunskih radnji,a po potrebi i zagrade, tako da dobijeteizraze kojima su redom vrijednosti 2001,2002, 2003, 2004, 2005, 2006, 2007, 2008.Posebno cu ocijeniti ona rjesenja koja nisuna mome popisu rjesenja. Rjesavanje vammogu olaksati ove jednakosti: 91 · 22 =2002, 667 · 3 = 2001, 654 · 3 = 1962. Naposao!

Sela A , B i C cine vrhove pravilnogtrokuta.U selu A zivi 30 skolaraca, u seluB ih je 20, dok je u selu C njihov brojnajmanji i iznosi 10. Svi skolarci isli su uskolu u prilicno udaljenom mjestu i zato suzitelji tih triju sela odlucili sagraditi skoluu svojoj blizini.

Gdje treba sagraditi tu skolu da zbrojudaljenosti, koje svi skolarci moraju prijeciod svojih sela do skole, bude najmanji?

Brat i sestra Veselic odlucili su odmahrijesiti domacu zadacu kako bi imali viseslobodnog vremena za odmor. Ali napocetku nije sve krenulo najbolje.

— Nas profesor Dobric ovaj put jepretjerao — ljutio se za svojim radnim

stolicem brat Branko. — Dati za domacuzadacu da se na -du zadnje znamenkeovakvih potencija 550 , 8800 , 99000 ! Zaron misli da cu ja citav vikend mnozitisilne petice, osmice i devetke?

Za drugim stolicem sestra Snjezana sesmjeskala:

— Da, da, na velikim si mukama! Samosi zaboravio uputu naseg dobrog profesora.Na moras stvarno mnoziti bazu po bazuda bi doznao kojom znamenkom zavrsavasvaka od tih potencija.

Sto je Branko zaboravio?

Ako sredista susjednih strana kocke po-vezemo duzinama, dobiva se geometrijskotijelo koje ima 6 vrhova.

Evo pitanja:1) Koje je to tijelo?2) Koliki je obujam toga tijela ako je

duljina brida kocke 1?

Na crtezu je krug razdijeljen sa 4 pravcana 9 dijelova. Pokazite da je moguceodabrati 4 pravca koji krug dijele na manji,odnosno veci broj dijelova od 9. Svakipravac treba pritom obodnu kruznicu sjeciu dvije tocke.

Koji se najmanji, a koji najveci broj di-jelova moze dobiti na taj nacin? Pokusajterijesiti zadatak ako je broj pravaca 5 ili 6.

Zdravko Kurnik, Zagreb

Matematicko-fizicki list, LIX 2 (2008. – 2009.) 105

Page 42: Matematicko-fizicki list Zagreb

Redakcija, iz tehnickih razloga, daje ovoupozorenje:

Krajnji rok za primanje rjesenja iz ovogbroja je 28. veljace 2009. Rjesenja (i imenarjesavatelja) bit ce objavljena u br. 4/ 236.

Ujedno molimo da pripazite na upute rjesa-vateljima koje su na str. 143.

A) Zadaci iz matematike

U zadatku 3131. pojavila se greska panavodimo ispravnu formulaciju.

3131. Izracunaj sumu

114 + 12 + 1

+2

24 + 22 + 1

+ . . . +2008

20084 + 20082 + 1.

3133. Ako su p i p2 + 8 prosti brojevidokazi da je i p2 + 4 prost broj.

3134. Za dani prirodan broj n pokazi da je64n+1 − 63n − 64 djeljivo s 3969. Opcenito,dokazi da je za svake prirodne brojeve n i abroj (a + 1)n+1 − an − (a + 1) djeljiv s a2 .

3135. Dokazi da su za svaki prirodan brojn ≥ 5 prvih n decimala broja

(n +p

n2 + 1)n

jednake nuli.

3136. Na -di sva pozitivna rjesenja jednazbe

xq

1 − y2 + yp

2 − z2 + zp

3 − x2 = 3.

3137. Omjer duljina stranica paralelogramaje p : q , a omjer duljina njegovih dijagonalaje m : n . Odredi kutove paralelograma.

3138. Neka je P proizvoljna tocka unutarsiljastokutnog trokuta ABC i k opisana mukruznica. Pravci AP , BP i CP po drugi putsijeku k u tockama A1 , B1 i C1 . Oznacimos A2 , B2 i C2 ortogonalne projekcije tocke Pna pravce BC , CA i AB , tim redom. Dokazida su trokuti A1B1C1 i A2B2C2 slicni.

3139. Tri kruznice polumjera p , q i r sasredistima A , B i C , tim redom, dodiruju se

izvana u tockama D , E i F . Dokazi da jeomjer povrsina trokuta DEF i ABC jednak

2pqr(p + q)(q + r)(r + p)

.

3140. U ravnini je zadan konveksni cetvero-kut ABCD . S njegove unutarnje strane nacrtanisu jednakostranicni trokuti ABQ i CDN , a snjegove vanjske strane jednakostranicni trokutiBCM i ADP . Dokazi da je cetverokut MQPNparalelogram.

3141. Ako za kutove α , β i γ trokutavrijedi

sin2 α + sin2 β + sin2 γ = 2,

dokazi da je on pravokutan.

3142. Za bilo koje tocke A , B , C i D uprostoru dokazi nejednakost

|AC|2+|BD|2+|AD|2+|BC|2 ≥ |AB|2+|CD|2.3143. U koordinatnoj ravnini dane su tocke

A(−a, 0) i B(2a, 0) , a > 0. Sto je skup svihtocaka P(x, y) takvih da je <)PBA = 2<)BAP?

Koje tocke na osi x pripadaju tom skupu?

3144. Odredi produktnY

k=1

1 − tg2 2kπ

2n + 1

!.

3145. Bridovi AB , AC i AD tetraedraABCD su u parovima okomiti i njihove duljinesu redom a , b i c . Odredi polumjer sfereopisane tom tetraedru.

3146. U nekom gradu ima 10 000 bicikala,svaki je oznacen brojem od 1 do 10 000 takoda nikoja dva od njih nemaju isti broj. Kolikaje vjerojatnost da je svaka znamenka brojaslucajno izabranog bicikla razlicita od 8?

B) Zadaci iz fizike

OS – 286. Kroz bakrenu zavojnicu tecestruja jakosti 0.4 A kad je napon na njenimkrajevima 5.4 V. Nakon mjerenja ucenicisu odmotali zicu s kalema i izvagali je.Masa joj je iznosila 11 g. Koliki su bilipromjer i duljina bakrene zice? Gustoca bakraiznosi 8900 kg/ m3 , a elektricna otpornost0.017 · 10−6 Ω m.

106 Matematicko-fizicki list, LIX 2 (2008. – 2009.)

Page 43: Matematicko-fizicki list Zagreb

OS – 287. Nekad su se kotaci za zapreznakola pravili stavljanjem zeljeznih obruca nadrvene kotace. Na kotac promjera 80 cmstavljao se zeljezni obruc kojem je promjer bio4 mm manji od promjera kotaca. Za kolikostupnjeva je trebalo zagrijati obruc da bi semogao staviti na kotac? Koeficijet linearnogsirenja zeljeza iznosi 1.2 · 10−5 K−1 .

OS – 288. Markov otac zeli prije zimenapuniti cisternu s uljem za lozenje. Njihovacisterna je pravokutnog oblika, duljine 2 metra,siroka 1 metar i visoka 1.5 metara. Napunjenaje uljem do visine 40 cm. Koliko litara uljaMarkov otac mora naruciti da bi napuniocisternu do vrha? Koliko ce se nakon punjenjapovecati hidrostatski tlak na dnu cisterne?Gustoca ulja za lozenje je 850 kg/ m3 .

OS – 289. Jedan djecak vuce sanjkena kojima sjedi drugi djecak mase 42 kg.Sila vucenja iznosi 30 N. Masa sanjki je6 kilograma. Maksimalna misicna sila djecakakoji vuce sanjke iznosi 64 N. Hoce li on mocivuci sanjke ako na njih sjedne treci djecakmase 50 kg?

1406. Na kojoj je visini ubrzanje slobodnogpada jednako 25% od ubrzanja slobodnog padana povrsini Zemlje?

1407. Metak mase m = 6 g ispaljen jehorizontalno u blok mase M = 2.8 kg kojimiruje na horizontalnoj podlozi koeficijentatrenja μ = 0.3. Metak se zabije u blok i ostajeu njemu, a blok se pomakne za l = 0.65 m izaustavi se. Na -di pocetnu brzinu metka!

1408. Na horizontalnoj podlozi lezi kuglamase M = 2 kg spojena preko opruge konstanteelasticnosti k = 500 N/ m s cvrstim zidom.Metak mase m = 20 g i brzine v1 = 600 m/ szabije se u kuglu i ostaje u njoj.

a) Koliko ce se stisnuti opruga?

b) Odredi period titranja sistema!

1409. Elektron se krece brzinom 0.8c .Kolika je masa elektrona i ukupna energija(m0 = 9.1 · 10−31 kg) .

1410. Tanki homogeni stap mase m1 = 2 kgi duljine l = 2 m ucvrscen je o jedan krajpomocu zgloba, a o drugi je privezan naoprugu tako da je ravnotezni polozaj stapaokomit. Opruga ima zanemarivu masu, anjezina konstanta elasticnosti je k = 6.1 N/ m.Izracunaj period malih oscilacija stapa.

1411. Izracunaj doskok skijasa skakaca kojipolijece brzinom od 20 m/ s pod kutom 15◦prema horizontalnoj ravnini, uz pretpostavkuda se otpor zraka zanemaruje i da se padina nakoju skijas skace moze aproksimirati kosinomkoja s horizontalnom ravninom zatvara kut od45◦ . (Doskok je udaljenost od tocke polijetanjado tocke u kojoj skijas dodirne padinu.)

1412. U valjkastu posudu polumjerar = 10 cm ulije se 1 litra vode. U posuduspustimo staklenu kocku (ρ = 4000 kg/ m3) .Koliku je silu potrebno upotrijebiti da se kockapodigne s dna posude, ako je njezina masam = 0.75 kg?

C) Rjesenja iz matematike

U rjesenju zadatka 3093. pojavila se greskapa donosimo ispravno rjesenje.

3093. Ako su a, b i c pozitivni realnibrojevi, dokazi nejednakost

b + ca2 + bc

+c + a

b2 + ca+

a + bc2 + ab

≤ 1a

+1b

+1c.

Rjesenje.

1a+

1b+

1c− b+c

a2+bc− c+a

b2+ca− a+b

c2+ab� 0.

· abc(a2 + bc)(b2 + ca)(c2 + ab)

⇐⇒ . . . ⇐⇒

Matematicko-fizicki list, LIX 2 (2008. – 2009.) 107

Page 44: Matematicko-fizicki list Zagreb

a4b4+b4c4+c4a4−a4b2c2−b4c2a2−c4a2b2

abc(a2+bc)(b2+ca)(c2+ab)�0.

( ∗ )Koristeci nejednakost

x2 + y2 + z2 � xy + yz + zx

⇐⇒ 12[(x − y)2 + (y − z)2 + (z − x)2] � 0

imamo za x = a2b2 , y = b2c2 , z = c2a2 :

(a2b2)2 + (b2c2)2 + (c2a2)2

� a2b2 · b2c2 + b2c2 · c2a2 + c2a2 · a2b2

⇐⇒ a4b4 + b4c4 + c4a4

− a4b2c2 − b4c2a2 − c4a2b2 � 0.

Zato je ispravna nejednakost (∗) .

Jednakost vrijedi ako i samo ako jea = b = c .

Edin Ajanovic (3),Prva bosnjacka gimnazija, Sarajevo, BiH

3105. Dokazi da niz brojeva an =3n + 4, n ∈ N ne sadrzi nijedan kvadratprirodnog broja.

Rjesenje. Pretpostavimo suprotno, tj. da zaneki n vrijedi 3n + 4 = m2 , m ∈ N . Buducida 3 � |m2 , onda ni 3 � |m ,

3n = m2 − 4,

3n = (m + 2)(m − 2).

Jer je 3 prost broj, oba faktora moraju bitidjeljiva s 3 ili jedan od njih mora biti jednak1.

Kad bi oba broja bila djeljiva s 3, tad bii njihova razlika bila djeljiva s 3, no njihovaje razlika jednaka 4, stoga barem jedan od tihbrojeva nije djeljiv s 3. Dakle, preostaje jedinoda je jedan od tih faktora jednak 1.

Za m + 2 = 1 je m = −1 sto nije moguce,jer je m prirodan broj.

Za m − 2 = 1 je m = 3 sto nije moguce,jer m nije djeljiv s 3.

Dakle, niz (an) ne sadrzi nijedan kvadratprirodnog broja.

Kristijan Kvaternik (1),V. gimnazija, Zagreb

3106. Neka su a, b, c realni brojevi takvida je abc �= 0 . Ako je

x =bc

+cb, y =

ca

+ac, z =

ab

+ba,

dokazix2 + y2 + z2 − xyz = 4.

Rjesenje.

x2 + y2 + z2 − xyz = 4,

x2 + y2 + z2 = 4 + xyz,„bc

+cb

«2

+„

ca

+ac

«2

+„

ab

+ba

«2

= 4 +„

bc

+cb

«„ca

+ac

«„ab

+ba

«,

b2

c2 + 2 +c2

b2 +c2

a2 + 2 +a2

c2 +a2

b2 + 2 +b2

a2

= 4 +„

ba

+abc2 +

c2

ab+

ab

«„ab

+ba

«,

b2

c2 +c2

b2 +c2

a2 +a2

c2 +a2

b2 +b2

a2 + 6

= 4 + 1 +b2

a2 +a2

c2 +b2

c2 +c2

b2 +c2

a2 +a2

b2 + 1.

Na kraju i lijevo i desno imamo iste clanovekoji se svi me -dusobno eliminiraju.

David Devald (1),SS Izidora Krsnjavoga, Nasice

3107. Na -di sva cjelobrojna rjesenja jed-nadzbe

(x−2005)(x−2006)(x−2007)(x−2008) = 3024.

Rjesenje. Zamjenom x−2005 = a jednadz-ba postaje

a(a − 1)(a − 2)(a − 3) = 3024,

(a2 − 3a)(a2 − 3a + 2) = 3024.

Supstitucijom a2 − 3a = b dobivamo

b(b + 2) = 3024,

b2 + 2b − 3024 = 0.

Rjesenja ove jednadzbe su b1 = 54 ib2 = −56.

Za b1 = 54 je

a2 − 3a = 54,

108 Matematicko-fizicki list, LIX 2 (2008. – 2009.)

Page 45: Matematicko-fizicki list Zagreb

a2 − 3a − 54 = 0,

a1 = 9, a2 = −6,

x1 = 2014, x2 = 1999.

Za b2 rjesenje je

a2 − 3a = −56,

a2 − 3a + 56 = 0.

Jednadzba nema realnih rjesenja.

Dakle, x ∈ {1999, 2014}Kristijan Kavternik (1), Zagreb

3108. Za pozitivne realne brojeve x , y , zdokazi nejednakosti:

a)q

x2 + xy + y2 ≥√

32

(x + y) ;

b)px2+xy+y2

xy+

py2+yz+z2

yz+

√z2+zx+x2

zx

≥√

3

„1x

+1y

+1z

«.

Rjesenje. a)p

x2 + xy + y2 �√

32

(x + y) ,

⇐⇒ x2 + xy + y2 � 34(x2 + 2xy + y2),

⇐⇒ x2 − 2xy + y2 � 0,

⇐⇒ (x − y)2 � 0.

b) Iskoristit cemo dokazanu nejednakost iza): q

x2 + xy + y2 �√

32

(x + y).

Podijelimo tu nejednakost s xy :px2 + xy + y2

xy�

√3

2x + yxy

,px2 + xy + y2

xy�

√3

2

„1x

+1y

«.

Analogne nejednakosti dobivamo za parove yi z , odnosno z i x :p

y2 + yz + z2

yz�

√3

2

„1y

+1z

«,

√z2 + zx + x2

zx�

√3

2

„1z

+1x

«.

Zbrajanjem tih triju nejednakosti dobivamozadanu nejednakost:p

x2+xy+y2

xy+

py2+yz+z2

yz+

√z2+zx+x2

zx

�√

32

„1x

+1y

+1y

+1z

+1z

+1x

«

�√

3

„1x

+1y

+1z

«.

David Devald (1), Nasice

3109. U decimalnom zapisu broja (5 +√26)n odredi prvih n znamenaka poslije

decimalne tocke.

Rjesenje. Neka je α = 5 +√

26 , β =5−√

26 . Brojevi α i β su rjesenja koordinatnejednadzbe

x2 − 10x − 1 = 0.

Ako jeαn = an + bn

√26,

tada jeβn = an − bn

√26,

pa jeαn + βn = 2an = An

prirodan broj. Odatle slijedi

An − αn = βn i

|An − αn| = |βn| =1

(5 +√

26)n<

110n .

Prema tome, razlika izme -du αn i An po

apsolutnoj je vrijednosti manja od1

10n . Kako

je β < 0 za parno n prvih n znamenaka poslijedecimalne tocke su devetke, a za neparno nprvih n znamenaka poslije decimalne tocke sunule.

Ur.

3110. Odredi cjelobrojno rjesenje jednadzbe

� 3√1 + � 3√2 + . . . + � 3p

x3 − 1 = 400.

(�x je oznaka za najveci cijeli broj koji nijeveci od x .)

Rjesenje. Kako je 3√8 = 3√23 = 2, prvih

7 clanova ima vrijednost 1, pa je njihov zbroj

Matematicko-fizicki list, LIX 2 (2008. – 2009.) 109

Page 46: Matematicko-fizicki list Zagreb

7 · 1 = 7. Zatim, 3√27 = 3√33 = 3 pa svi

clanovi od 8. do 26. imaju vrijednost 2. Njihov

zbroj je 19 · 2 = 38. 3√64 = 3√43 = 4, pa svi

clanovi od 27. do 63. imaju vrijednost 3. Njihov

zbroj je 37·3 = 111. Dalje 3√125 = 3√53 = 5,

pa svi clanovi od 64. do 124. imaju vrijednost4. Njihov zbroj je 61 · 4 = 244. Zbrojimo7 + 38 + 111 + 244 = 400. Zakljucimo da jezadnji clan jednak � 3√124 = � 3√x3 − 1 izcega slijedi x3 = 125, odnosno x = 5.

David Devald (1), Nasice

3111. Ako je zbroj cijelih brojeva a, b, cjednak 0, dokazi da je 2(a4 + b4 + c4) kvadratcijelog broja.

Rjesenje. Kako je a + b + c = 0 imamo

a2 + b2 + c2 = −2(ab + bc + ca).

Tada je

(a2 + b2 + c2)2 = 4(ab + bc + ca)2,

a4 + b4 + c4 + 2(a2b2 + b2c2 + c2a2)

= 4(a2b2 + b2c2 + c2a2) + 8abc(a + b + c),

tj. a4 + b4 + c4 = 2(a2b2 + b2c2 + c2a2).Sada je

2(a4 + b4 + c4)

= a4 + b4 + c4 + 2(a2b2 + b2c2 + c2a2)

= (a2 + b2 + c2)2.

Edin Ajanovic (3), Sarajevo, BIH

3112. Tocka D je noziste visine spusteneiz vrha A na krak BC jednakokracnog trokutaABC . Ako je |AC| + |CD| = 2(|AB| + |BD|) ,koliki su njegovi kutovi?

Rjesenje.

Uz oznake kao na slici dani uvjet mozemozapisati u obliku 2b − x = 2(a + x) . Izslicnosti �ABD i �BCE slijedi a : x =

2b : a , odnosno x =a2

2b. Uvrstavanjem

x u pocetnu jednadzbu dobivamo kvadratnujednadzbu 4b2 − 4ab − 3a2 = 0. Za pozitivno

rjesenje dobivamoba

=32

. Zbog cos β =a2b

=13

slijedi β = 70◦32′ , a α = 90◦−β =

19◦28′ .

Vanja Ubovic (2),Gimnazija P. Preradovica, Virovitica

3113. Ako su α , β , γ kutovi siljastokutnogtrokuta, dokazi da vrijedi nejednakost

sinα+sin β+sin γ ≤ 2(cosα+cos β+cos γ ).

Rjesenje.

Po poucku o sinusima vrijedi a = 2R sinα ,b = 2R sin β , c = 2R sin γ . Za a′ = |AH|imamo

|AC′| = a′ · cos <)HAC1 = a′ cos <)A1AB

= a′ cos

„π2− β

«= a′ sin β tj.

a′ =|AC′|sin β

=b cosαsin β

=2R sin β · cosα

sin β= 2R cosα.

Analogno se dobije

b′ = 2R cos β , c′ = 2R cos γ .

Iz nejednakosti o trokutu vrijedi

a′ + b′ > c,

b′ + c′ > a,

110 Matematicko-fizicki list, LIX 2 (2008. – 2009.)

Page 47: Matematicko-fizicki list Zagreb

c′ + a′ > b,

odakle je

2(a′ + b′ + c′) > a + b + c, tj.

2(cosα + cos β + cos γ > sinα + sin β + sin γ .

Dakle, vrijedi stroga nejednakost.

Ur.

3114. Kompleksni brojevi zadovoljavaju oveuvjete:

z1 + z2 = −i − 1,

z1 · z2 = −i.

Ne racunajuci z1 i z2 , odredite z1 · z2 .

Rjesenje. Primijetimo da je

z1 · z2 =z1z2

· |z2|2.

Iz (z1 + z2)2 = 2i = −2z1 · z2 , dobivamo da je

z12 + 4z1z2 + z2

2 = 0

ili u ekvivalentnom obliku„z1z2

«2

+ 4

„z1z2

«+ 1 = 0.

Vidimo da jez1z2

realno i negativno. Zato

je z1 · z2 tako -der realno i negativno. Kakoje |z1 · z2| = |z1 · z2| = 1, konacno dobijemoz1 · z2 = −1.

Ur.

3115. Dokazi da za α ∈“0,π2

”vrijedi

nejednakost„1 +

1sinα

«„1 +

1cosα

«≥ 3 + 2

√2.

Prvo rjesenje.

α ∈fi

0,π2

fl=⇒ sinα , cosα > 0,

„1 +

1sinα

«„1 +

1cosα

«

= 1 +„

1sinα

+1

cosα

«+

1sinα cosα

� 1 +2√

sinα cosα+

1sinα cosα

=„

1 +1√

sinα cosα

«2

=„

1 +√

2√sin 2α

«2

� (1 +√

2)2 = 3 + 2√

2.

Ur.

Drugo rjesenje. Koristit cemo Huygensovunejednakost: Ako su realni brojevi xi > 0,i = 1, 2, . . . , n , tada vrijedi nejednakost

(1+x1)(1+x2) . . . (1+xn) �„

1+ n√x1x2 . . . xn

«n

.

Iz dane nejednakosti slijedi„1+

1sinα

«„1+

1cosα

«�„

1+1√

sinα cosα

«2

A−G�„

1+2

sinα cosα

«2

� (1+√

2)2 = 3+2√

2.

( ∗ )Radi poznate nejednakosti

0 < sinα + cosα � 2

√2

2=

√2,

odnosno1

sinα + cosα� 1√

2,

slijedi (∗) . Jednakost vrijedi ako i samo ako

je sinα = cosα =√

22

, tj. ako i samo ako je

α = 45◦ .

Vanja Ubovic (2), Virovitica

3116. Koliko se najvise lovaca mozepostaviti na sahovsku plocu n × n tako dase nikoja dva ne napadaju.

Rjesenje. Promotrimo dijagonale sahovskeploce n · n .

(Tih dijagonala ima 2n − 1.)

Na svakoj od tih dijagonala smije stajatinajvise jedan lovac. No na dijagonali 1 i 2n−1

Matematicko-fizicki list, LIX 2 (2008. – 2009.) 111

Page 48: Matematicko-fizicki list Zagreb

ne mogu istovremeno stajati dva lovca. (To supolja na suprotnim kutovima ploce.)

Zakljucak: najvise mozemo staviti (2n −1) − 1 = 2n − 2 lovaca.

No postoji razmjestaj 2n − 2 lovaca uzzadane uvjete:

Ur.

3117. Na -di vjerojatnost da slucajno izabra-ni dvoznamenkasti broj bude djeljiv barem ilis 3 ili sa 7.

Rjesenje. Neka je A doga -daj u kojem jeizabran broj djeljiv s 3, a B doga -daj ukojem je izabran broj djeljiv sa 7. Treba nacivjerojatnost p(A + B) , pri cemu se doga -daji Ai B ne iskljucuju. Dvoznamenkastih brojeva11, 12, . . . , 99 ima 90. Od njih je 30 djeljivos 3 (povoljni slucajevi za doga -daj A ), 13 sa7 (povoljni slucajevi za doga -daj B ) i 4 sudjeljiva i s 3 i sa 7 (povoljni slucajevi za AB ).Zato je

p(A + B) = p(A) + p(B) − p(AB)

=3090

+1390

− 490

=1330

= 0.43.

Edin Ajanovic (3), Sarajevo

3118. Na -di sumu reda∞Xn=0

2n

a2n + 1,

gdje je a > 1 .

Rjesenje. Za a > 1 imamo

2n

a2n + 1=

2n(a2n − 1)a2n+1 − 1

=2n(a2n + 1) − 2n + 1

a2n+1−1

=2n

a2n − 1− 2n + 1

a2n+1 − 1,

pa je

Sk =kX

n<0

2n

a2n + 1

=kX

n=0

„2n

a2n − 1− 2n + 1

a2n+1 − 1

«

=1

a − 1− 2k+1

a2k+1 − 1,

S = limk→∞

Sk = limk→∞

„1

a − 1− 2k+1

a2k+1 − 1

«

=1

a − 1.

jer je limk→∞

2k + 1

a2k+1 − 1= 0.

Edin Ajanovic (3), Sarajevo

D) Rjesenja iz fizike

OS – 278. Lukina baka stanuje na cetvrtomkatu. Visinska razlika izme -du katova je 3 metra.Luka je izmjerio da lift putuje do cetvrtog kata12 sekundi. Snaga elektromotora koji pokrecekabinu lifta je 6 kW. Kolika je korisnost motoraako je masa kabine, zajedno s ljudima u njoj,500 kg?

Rjesenje.

h = 4 · 3 m = 12 m

t = 12 s

Pu = 6 kW

m = 500 kg

η =?

G = m · g = 500 kg · 10 N/ kg

G = 5000 N

W = G · h = 5000 N · 12 m

W = 60 000 J

Pk =Wt

=60 000 J

12 sPk = 5000 W = 5 kW

112 Matematicko-fizicki list, LIX 2 (2008. – 2009.)

Page 49: Matematicko-fizicki list Zagreb

η =Pk

Pu=

5 kW6 kW

η = 0.83 = 83%

Arijan Golub (8),OS Augusta Cesarca, Krapina

OS – 279. Napravi njihalo od konca iutega proizvoljne mase. Odredi period takvognjihala za duljine konca 10, 20, 30, 40i 50 cm. Nacrtaj dijagram koji prikazujeovisnost perioda njihala o duljini konca. Je lita ovisnost linearna?

Rjesenje.

l (cm) N t (s) T =tN

(s)

50 10 14.3 1.4340 10 12.6 1.2630 10 11.1 1.1120 10 9.53 0.9510 10 6.8 0.68

Ovisnost perioda o duljini niti nije linearna.

Ana Paic (7),OS Fausta Vrancica, Sibenik

OS – 280. Stolica ima cetiri nogekvadratnog presjeka. Njihova je debljina 2 cm.Masa stolice je 4 kilograma. Kolika bi trebalabiti masa covjeka koji bi sjeo na tu stolicu datlak pod njenim nogama iznosi 0.5 MPa?

Rjesenje.

n = 4

a = 2 cm

ms = 4 kg

p = 0.5 MPa = 500 000 Pa

mc =?

A1 = a · a = 2 cm · 2 cm

A1 = 4 cm2

A = 4 · A1

A = 16 cm2 = 0.0016 m2

G = F = p · AG = 500 000 Pa · 0.0016 m2

G = 800 N

Gs = ms · g = 4 kg · 10 N/ kg

Gs = 40 N

Gc = G − Gs

Gc = 760 N

mc =Gc

g=

760 N10 N/ kg

mc = 76 kg

Juraj Krsnik (8),OS Augusta Cesarca, Krapina

OS – 281. Keopsova piramida je najstarijeod sedam starih svjetskih cuda. Sagra -denaje oko 2560 g.p.n.e. To je najveca inajteza gra -devina ikad sagra -dena na Zemlji.Sagra -dena je od 2 300 000 kamenih blokovaprosjecne mase 2.5 tona. Sad je visoka135.75 metara, a duljina stranica osnoviceiznosi 229 metara. Izracunaj koliki postotak uukupnom volumenu piramide otpada na kamen,a koji na prostorije i hodnike. Gustoca kamenakrecnjaka od kojeg su isklesani blokovi iznosi2600 kg/m3 .

Rjesenje.

m = 2.5 t · 2 300 000

= 5 750 000 t = 5 750 000 000 kg

h = 135.75 m

a = 229 m

ρ = 2600 kg/ m3

Volumen piramide:

Vp = S · h3

Vp = a · a · h3

Vp = 229 · 229 · 135.753

Matematicko-fizicki list, LIX 2 (2008. – 2009.) 113

Page 50: Matematicko-fizicki list Zagreb

Vp = 2 372 955 m3

Volumen kamena:

ρ =mV

V1 =mρ

V1 =2500 kg

2600 kg/ m3

V1 = 0.96 m3

Vk = 0.96 · 2 300 000

Vk = 2 208 000 m3

Vk

Vp=

2 208 000 m3

2 372 955 m3 = 0.93 = 93%

93% otpada na kamen, a 7% na prostorije ihodnike.

Ana Paic (7),OS Fausta Vrancica, Sibenik

1392. Konac namotan na kalem unutarnjegradijusa r i vanjskog R lezi na horizontalnojpovrsini i moze se kotrljati bez klizanja.Kolikom brzinom i u kojem smjeru ce se kalemgibati ako se kraj konca povlaci brzinom vkao na slici?

Rjesenje. Kalem se kotrlja bez klizanja, pamora postojati sila trenja Ftr koja djeluje usuprotnom smjeru od brzine v (s hvatistem utocki dodira s podlogom). Buduci da se kalemne ubrzava, konac se povlaci silom F istogiznosa (dakako, suprotnog smjera). Momentsile trenja bit ce veci od momenta sile F , pace kalem rotirati u smjeru kazaljke na satu. Zavrijeme t kraj konca ce se pomaknuti za v · t ,centar kalema ce se pomaknuti za v′t = ω ·R · t(gdje je ω kutna brzina rotacije kalema), akonac ce se dodatno “zamotati” za duljinuω · r · t . Stoga vrijedi:

v · t − ω · R · t = −ω · r · t,v = ω · R − ω · r,

ω =v

R − r,

v′ = Rω =vR

R − r.

Ur.

1393. Dva tijela masa m1 i m2 spojenasu krutom sipkom duljine l i postavljenana horizontalnu podlogu. Koeficijenti trenjaizme -du tijela i podloge su μ1 i μ2 . Sipkaje zanemarive mase dok su tijela zanemarivomalenih dimenzija u odnosu na duljinu sipke.U sredistu sipke djeluje sila iznosa F okomitona nju i paralelno s horizontalnom podlogomkao na slici. Koliki mora biti iznos te sile dabi sipka ostala okomita na smjer sile?

Rjesenje. Sile trenja na mase m1 i m2djeluju u smjeru suprotnom od sile F . Tezistesistema nalazi se u tocki koja je za

l1 =lm2

m1 + m2

udaljena od m1 ; udaljenost tezista od m2 je

l2 =lm1

m1 + m2.

Bez gubljenja opcenitosti, mozemo pretposta-viti da je m1 > m2 (teziste se tada nalazilijevo od centra, tj. polovice sipke). Ukupanmoment sile oko tezista je:

M = Ftr1l1 − Ftr2l2 + F

„l2− l1

«

= μ1m1glm2

m1 + m2− μ2m2g

lm1

m1 + m2

+ F

„l2− lm2

m1 + m2

«

= glm1m2

m1+m2(μ1−μ2) + F

„l2− lm2

m1+m2

«.

Ako zelimo da sipka ostane okomita na smjersile, moment sile mora biti jednak nuli (u

114 Matematicko-fizicki list, LIX 2 (2008. – 2009.)

Page 51: Matematicko-fizicki list Zagreb

protivnom ce doci do rotacije sipke):

M = 0 =⇒

glm1m2

m1 + m2(μ1 − μ2) = −F

„l2− lm2

m1 + m2

«,

F = glm1m2

m1 + m2(μ2 − μ1) :

„l2− lm2

m1 + m2

«

= g2lm1m2

l (m1 + m2) − 2lm2(μ2 − μ1).

Ur.

1394. Dijete mase m njise se na njihaljciduljine l s kutom maksimalnog otklona odravnoteznog polozaja α . Dijete vice i timeproizvodi zvuk frekvencije f . Koji rasponfrekvencija cuje roditelj koji stoji isprednjihaljke daleko u odnosu na svoju visinu?

Rjesenje. Maksimalnu brzinu njihanja dijetepostize u trenutku kada je u najnizem polozajujer se tada sva potencijalna gravitacijskaenergija pretvara u kineticku:

mgl − mgl cosα =mv2

2,

v =p

2gl(1 − cosα).

Minimalnu frekvenciju racunamo pomocu Dop-plerovog izraza u situaciji kada se dijete tombrzinom udaljava od roditelja, a maksimalnukada mu se priblizava:

f ′ = fvzvuka

vzvuka ± v

= fvzvuka

vzvuka ±p

2gl(1 − cosα).

Ur.

1395. Dokazi da je za sfericna zrcalaumnozak udaljenosti predmeta i slike od zaristauvijek jednak kvadratu zarisne duljine.

Rjesenje. Oznacimo li s a i b udaljenostipredmeta i slike od tjemena zrcala, s x i yudaljenosti predmeta i slike od fokusa, vrijedi:

a = x + f , b = y + f .

Iz jednadzbe sfernog zrcala:1f

=1a

+1b

imamo:

f (a + b) = ab,

f (x + y + 2f ) = (x + f )(y + f ),

f x + f y + 2f 2 = xy + f x + f y + f 2,

f 2 = xy,

sto je i trebalo pokazati.

Kristijan Kvaternik (1),V. gimnazija, Zagreb

1396. Udaljenost izme -du dvije identicnemetalne kuglice na kojima su razlicite kolicinenaboja je 2 cm. Radijusi kuglica su mnogomanji od njihove me -dusobne udaljenosti.Privlacna sila izme -du kuglica je 4 · 10−5 N.Nakon sto su kuglice spojene vodljivom zicomi zatim odspojene, one se me -dusobno odbijajusilom od 2.25·10−5 N. Odredite pocetne nabojena kuglicama.

Rjesenje.

F1 = −4 · 10−5 N

F2 = 2.25 · 10−5 N

r = 0.02 m

Q1, Q2 =?

Bez smanjenja opcenitosti moze se pretpostavitida je kuglica naboja Q1 prije dodira nabijenapozitivno, a kuglica s nabojem Q2 negativno.Tada je:

F1 =1

4πε0Q1Q2

r2 ,

Q1Q2 = 4πε0r2F1.

Pri spajanju kuglica doslo je do prelaskaodre -denog pozitivnog naboja Q s kuglice snabojem Q1 na kuglicu s nabojem Q2 , tako daje naboj na kuglicama nakon spajanja jednak(kuglice su identicne; dakle, imaju jednakikapacitet).

F2 =1

4πε0(Q1 − Q)(Q2 + Q)

r2 ,

Q1 − Q = Q2 + Q.

Ove tri jednadzbe cine sistem s tri nepoznanice(Q,Q1, Q2 ). Eliminiranjem Q2 dobivamo:

(Q1 − Q)2 = 4πε0r2F2,

Q1(Q1 − 2Q) = 4πε0r2F1.

Oduzimanjem ove dvije jednadzbe dobivamo:

Q2 = 4πε0r2(F2 − F1),

Matematicko-fizicki list, LIX 2 (2008. – 2009.) 115

Page 52: Matematicko-fizicki list Zagreb

Q = 1.67 · 10−9 C,

Q1 =p

4πε0r2F2 + Q = 2.67 · 10−9 C,

Q2 = Q1 − 2Q = −0.67 · 10−9 C.

Ur.

1397. Elektricna zarulja od 110 V i 60 Wspojena je na bateriju elektromotorne sile od120 V. Unutarnji otpor baterije je 60 Ω . Hoceli zarulja svijetliti maksimalnim intenzitetom uovakvom spoju?

Rjesenje.

U = 110 V,

P = 60 W,

E = 120 V,

r = 60 Ω.

Otpor zarulje racunamo iz njenih karakteristika:

P = UI =U2

R=⇒ R =

U2

P= 201.7 Ω.

Struja koja ce teci kroz zarulju kada je spojimona bateriju je:

I =E

R + r= 0.46 A.

Na zarulji ce se razvijati snaga

P = I2R = 42.7 W,

sto je manje od nominalne snage. Dakle, zaruljanece svijetliti maksimalnim intenzitetom.

Ur.

1398. Dvije identicne metalne kugle zagri-javaju se tako da im se polumjer poveca zaisti iznos. Jedna od njih lezi na stolu, a drugaje objesena o nit. Obje kugle su toplinski izo-lirane od stola i niti. Kojoj kugli je dovedenovise topline i zasto?

Rjesenje. Kugli koja lezi na stolu pripovecavanju polumjera raste i gravitacijskapotencijalna energija (jer joj se teziste podize)pa je ukupna toplina koju joj treba dovestiveca!

Ur.

Rjesenja zabavne matematike

Tanja je nabrojila 61 pravokutni trokut, aPerica 40 paralelograma.

Prema rijecima profesora Kubusa lakozakljucujemo sljedece:

Svaki matematicar dopisuje se na istomjeziku s barem tri druga. Od ove trojice ili sedva dopisuju na tome istom jeziku, ili se svatrojica dopisuju drugim jezikom.

Svaki matematicar dopisuje se na istomjeziku s barem sest drugih. Od ove sestoriceili se dva dopisuju na tom istom jeziku, ili sesva sestorica dopisuju na ostala dva jezika, atada se zakljucivanje izvodi kao u prethodnomslucaju.

Dakle, postoje najmanje tri matematicarakoji se me -dusobno dopisuju na istom jeziku.

Broj osvojenih bodova je 190. Ako jea broj bodova posljednjeg na tablici i rrazlika bodova svaka dva susjedna igraca,onda su igraci osvojili redom a , a + r ,a + 2r ,. . . , a + 19r bodova. Zbroj tih bodovaje 20a+190r , pa mora biti 20a+190r = 190.Jedini par prirodnih brojeva koji zadovoljavaovu jednakost je a = 0, r = 1. Pobjednik jeosvojio 19 bodova, a posljednji na tablici 0bodova.

Ucenici su otkrili da se broj 666 666, nakonnalazenja njegova rastava na proste faktore uobliku 666 666 = 2 · 3 · 3 · 7 · 11 · 13 · 37, mozena tri nacina prikazati kao umnozak dvajutroznamenkastih brojeva:

666 666 = 693 · 962 = 777 · 858 = 814 · 819.

116 Matematicko-fizicki list, LIX 2 (2008. – 2009.)

Page 53: Matematicko-fizicki list Zagreb

Matematicka konferencija u cast akademiku Josipu Pecaricu

Sanja Varosanec1 , Zagreb

Sudionici matematickih natjecanja cesto se susrecu s jednom posebnom vrstomzadataka – s matematickim nejednakostima. Nejednakost trokuta, Cauchy-Schwarz-Bunjakowskijeva nejednakost, Jensenova, Holderova, Minkowskijeva nejednakost,nejednakost izme -du aritmeticke i geometrijske sredine, te nejednakosti izme -du drugihsredina, samo su neke od osnovnih nejednakosti koje uz puno maste i blistavih idejakoristimo pri rjesavanju takvih zadataka. No, nejednakosti se ozbiljno proucavaju iu matematickoj znanstvenoj zajednici. Siroj je javnosti mozda manje poznato da uZagrebu zivi i djeluje jedno od najvecih svjetskih matematickih imena iz podrucjamatematickih nejednakosti – akademik Josip Pecaric. Cinjenica da je u 2008. godiniprofesor Pecaric navrsio 60. godina bila je povodom za organizaciju me -dunarodnekonferencije Mathematical Inequalities and Application 2008 (MIA 2008). Konferencijacija je organizacija pocela dvije godine prije odrzana je od 8. do 14. lipnja 2008.u Trogiru uz sudjelovanje 135 matematicara iz 27 zemalja – od sjevera Europe pado Australije. Malo je koja matematicka konferencija u Hrvatskoj privukla toliki brojsudionika iz cijeloga svijeta.

June 8 -14, 2008, Trogir, Croatia Conference in honour of Professor Josip Pečarić on the occasion of his 60th birthday

Mathematical Inequalities and Applications 2008

Faculty of Natural Sciences, Mathematics and Kinesiology, University of Split Faculty of Textile Technology, University of Zagreb Croatian Mathematical Society

INFORMATION: http://mia2008.ele-math.com CONTACT: [email protected] DEADLINE FOR ABSTRACTS: May 1, 2008

Organizing Committee

Aleksandra ČižmešijaNeven ElezovićMilica Klaričić BakulaMarko MatićSanja Varošanec

Scientific Committee

Lars-Erik Persson (Chairman)

Shoshana AbramovichRavi P. AgarwalTsuyoshi Ando Drumi BainovCatherine BandleRichard A. BrualdiPeter S. BullenVictor BurenkovWing-Sum CheungYeol Je ChoNeven ElezovićCarla GiordanoRoland GlowinskiPankaj JainAnna KaminskaAlois KufnerAndrea LaforgiaChi-Kwong Li Lech MaligrandaMarko MatićRam N. MohapatraEdward NeumanConstantin NiculescuLudmila Y. NikolovaRyskul OınarovIngram OlkinBohumir OpicCharles E. M. PearceIvan PerićAndrás PrékopaSaburou SaitohGord SinnamonHari M. SrivastavaVladimir D. StepanovGeorge P. H. StyanNenad UjevićSanja Varošanec

Peter S. Bullen Victor BurenkovSever S. DragomirRoland GlowinskiFrank HansenAlois KufnerLech MaligrandaRoy Mathias

Marko MatićIngram OlkinZsolt PálesIvan PerićLars-Erik PerssonAndrás PrékopaGord SinnamonVladimir D. Stepanov

Plenary Speakers

foto: Jelena Popić

Okosnicu konferencije cinilo je 15 jednosatnih predavanja pozvanih znanstvenika –P. S. Bullen (Kanada) osvrnuo se na najzanimljivija Pecariceva otkrica, V. Burenkov(Velika Britanija) iznio je najnovije rezultate o multiplikativnim visedimenzionalnimnejednakostima u tezinskim Lebesgueovim prostorima, S. S. Dragomir (Australija) dao je

1 Dr. sc. Sanja Varosanec je redovita profesorica PMF-Matematickog odjela Sveucilista u Zagrebu.

Matematicko-fizicki list, LIX 2 (2008. – 2009.) 117

Page 54: Matematicko-fizicki list Zagreb

pregled aproksimacija Riemann-Stieltjesovog integrala, R. Glowinski (USA) diskutiraoje rjesenja neglatkog problema svojstvenih vrijednosti, o operatorskim nejednakostima sprimjenama predavanje je odrzao F. Hansen (Danska), a o nejednakostima Hardyjevogtipa A. Kufner iz Ceske. Norme izme -du kvazi-Banachovih Lp -Lq prostora bilesu tema predavanja L. Maligrande (Svedska), a M. Matic (Split) pozabavio seEulerovim tipovima nejednakosti. Profesor I. Olkin (USA) u svom je predavanjunapravio zanimljivo povezivanje nejednakosti izme -du momenata, sredina, matricnih iprobabilistickih nejednakosti, a o nejednakostima za konveksne funkcije viseg reda culismo od A. Prekope (USA). Sredine i to generalizirane kvazi-aritmeticke bile su temapredavanja Zs. Palesa (Ma -darska), a mjesovite sredine bile su tema predavanja I. Perica(Zagreb). Predsjednik znanstvenog odbora konferencije Lars-Erik Person (Svedska)odrzao je predavanje o Hardyju i njegovom uplivu na teoriju nejednakosti, dok jeG. Sinnamon prezentirao rezultate o tezinskim Fourierovim nejednakostima, a V. D.Stepanov (Rusija) o nejednakostima za integralne operatore.

Ostali sudionici konferencije svoje su rezultate izlozili u vidu kratkih 20-minutnihpriopcenja. Zbog velikog broja prijavljenih priopcenja (njih 107) rad je bio organiziranu tri paralelne sekcije, a sve se odvijalo svakodnevno od 9 sati ujutro pa doveceri, uz pauzu za rucak. Predavanja su obuhvatila gotovo sve aspekte matematickihnejednakosti. Izlagalo se o analitickim nejednakostima, o geometrijskim, integralnim,operatorskim i matricnim, o varijacijskim, o nejednakostima u apstraktnim strukturama, oprimjenama nejednakosti u metodi konacnih elemenata, u ocjenama numerickih formula,pri rjesavanju funkcijskih jednadzbi, nelinearnih sustava diferencijalnih jednadzbi, teo problemima konveksnog programiranja. A naravno, odrzati konferenciju u Trogirui ne obici neke znamenitosti u okolici bilo bi nedopustivo. Sudionici konferencijeposjetili su Nacionalni park Krka, te obisli Split i Trogir. Na stranici konferencije(http://mia2008.ele-math.com) nalaze se detalji o konferenciji, a tamo je osimsluzbenih informacija moguce naci i niz slika koje docaravaju atmosferu.

Buduci da je konferencija MIA 2008 organizirana u cast 60. ro -dendana profesoraPecarica, razumljivo je da su se predavaci u velikoj mjeri dotakli i rezultata radaprofesora Pecarica sto je potpuno i razumljivo buduci da je on jedan od vodecihstrucnjaka u podrucju matematickih nejednakosti i jedan od najproduktivnijih svjetskihmatematicara s vise od 700 objavljenih ili prihvacenih za tisak znanstvenih radova, sa 70-ak radova na konferencijama i u knjigama, 60-ak iz elektrotehnike, geofizike, geologije,gra -devinarstva, fizike i povijesti, 14 matematickih monografija te dva visokoskolska

118 Matematicko-fizicki list, LIX 2 (2008. – 2009.)

Page 55: Matematicko-fizicki list Zagreb

udzbenika. Bio je deseti matematicar u svijetu koji je na MathSciNetu imao preko 500radova. Tako, na primjer, ISI Web of Knowledge u bazi SCIE prikazuje da se radoviJ. Pecarica citiraju 542 puta u 310 radova u casopisima te baze, a u monografiji P.S. Bullena Handbook of Means and Their Inequalities, Mathematics and Applications(Kluwer, 2003) Pecaric je najcitiraniji autor (7 knjiga i 105 clanaka). Ovi brojevicitiranosti kazuju koliki je odjek pojedinih radova u znanstvenoj zajednici, tj. koliko seputa drugi matematicari pozivaju na te radove. Njegovim je imenom nazvano nekolikoidentiteta i nejednakosti, a o primjenama Mond-Pecariceve metode na operatore uHilbertovim prostorima objavljena je monografija, dok se druga upravo priprema. Zanjegov je rad karakteristicna suradnja – preko 150 suradnika iz 26 zemalja, a podnjegovim je vodstvom izra -deno ili je u izradi 30-tak magistarskih i doktorskih radnji.Poznato je da svega cetrdesetak matematicara u svijetu ima preko sto suradnika.

Josip je Pecaric ro -den 2. rujna 1948. u Kotoru. Tamo je zavrsio osnovnoskolskoi srednjoskolsko obrazovanje, a 1972. je diplomirao na Elektrotehnickom fakultetu uBeogradu s radom iz podrucja nuklearne fizike. Na istom je fakultetu i magistrirao, a1982. obranio je doktorsku disertaciju iz podrucja matematike s naslovom Jensenovei srodne nejednakosti. U komisiji su bili profesori Boas i Ky Fan. Dio Boasovogizvjesataja navodimo u originalu:

Let me say to begin with that I would consider Pecaric’s thesis acceptable inany american university, and a better thesis than most of thesis I have seen.

Pecaric has, in the first place, systematized a large amount of material thatappeared in a haphazard way over the years and had become rather chaotic. Thissynthesis is a valuable piece of work in itself.

In the second place, Pecaric has shown great ingenuity in finding simpler proofsof some inequalities and appropriate generalizations of others. Consequently hehas made many inequalities more understandable. He has also been able to unifysome inequalities that originally seemed to be quite unrelated.

Finally, he has made many original contributions to the field.

Do 1987. godine radio je u Geomag-netskom institutu u Grockoj te na Gra -de-vinskom fakultetu u Beogradu, a onda sobitelji preseljava u Zagreb te se zapos-ljava na Tekstilno-tehnoloskom fakultetuSveucilista u Zagrebu gdje i danas radikao redoviti profesor. Od tada do danasnekoliko je puta kao gostujuci profesor bo-ravio na sveucilistima u Australiji, Italiji,Pakistanu i Svedskoj. Godine 1998. pok-renuo je me -dunarodni znanstveni casopisMathematical Inequalities and Applicati-ons (izdavac Element, Zagreb) koji je vec

nakon dvije godine izlazenja uvrsten na Scientific Citation Index Expanded (SCIE) listu,a prije par godina pokrenuo jos dva casopisa Journal of Mathematical Inequalities iOperators and Matrices. Urednistva ovih triju casopisa cini 90-ak uglednih matematicara.Osim toga objavio je 21 publicisticku knjigu – od knjige Srpski mit o Jasenovcu pa doZa hrvatske vrednote i preko 200 clanaka u kojima progovara i poucava o povijesti isadasnjosti Hrvatske zastupajuci ideje samostalne Hrvatske.

Za svoj doprinos u znanosti dobio je Drzavnu nagradu za znanost za 1996. godinu,te je odlikovan ordenom Reda Danice hrvatske s likom R. Boskovica. Od 2000. godineprofesor Pecaric redoviti je clan Hrvatske akademije znanosti i umjetnosti.

Matematicko-fizicki list, LIX 2 (2008. – 2009.) 119

Page 56: Matematicko-fizicki list Zagreb

49. me -dunarodna matematicka olimpijada

Ovogodisnja Me -dunarodna matematicka olimpijada (IMO) odrzana je u Madridu, usrpnju 2008. Sudjelovalo je 535 natjecatelja iz 97 drzava. Hrvatsku su predstavljali NinaKamcev (XV. gimnazija, Zagreb), Adrian Satja Kurdija, (V. gimnazija, Zagreb), InesMarusic (V. gimnazija, Zagreb), Sara Muhvic (III. gimnazija, Osijek), Melkior Ornik(XV. gimnazija, Zagreb) i Goran Zuzic (V. gimnazija, Zagreb). Voditelji ekipe bili suZeljko Hanjs i Mea Bombardelli s Matematickog odjela PMF-a u Zagrebu.

Ucenici su krenuli u Madrid 14. srpnja zajedno s voditeljicom Meom Bombardelli,dok je voditelj ekipe Zeljko Hanjs otputovao ranije u Spanjolsku i u gradu La Granjasudjelovao u izboru zadataka za natjecanje. Tamo nas je docekalo jako spanjolsko suncesto nas nije sprijecilo da vec prvi dan s vodicem po -demo u setnju gradom.

Natjecatelji su bili smjesteni u nekoliko studentskih domova. Na istom katu s namasu bile smjestene i ekipe iz Makedonije i Vijetnama. S Makedoncima smo otpocetkaostvarili veoma dobru komunikaciju, dok je s Vijetnamcima bilo malo teze jer njihovengleski nije bio ni upola tako dobar kao njihovo znanje matematike. Sljedeci dan, nasvecanom otvaranju, upoznali smo se i s ostalim ekipama iz nase regije – Bosancima,Crnogorcima, Slovencima i Srbima, s kojima smo se i kasnije druzili.

Na otvaranju smo, osim tradicionalnih govora o vaznosti matematike, imali prilikevidjeti i nekoliko cirkuskih nastupa. Sutradan je bio prvi dan natjecanja, na kojem suzadaci, po misljenju mnogih sudionika, bili laksi nego inace, ali je ipak bilo teskonakon prvog dana osigurati bilo kakvu medalju. Nakon drugog dana natjecanja poceloje spekuliranje o bodovima koje smo postepeno saznavali tijekom sljedeca dva dana,dok smo isli na izlete promatrajuci razne ljepote Spanjolske, kao sto su Segovia, SanLorenzo de Escorial ili Toledo.

Treci dan nakon natjecanja na Internetu su bili objavljeni cjelokupni rezultati: AdrianSatja Kurdija, Goran Zuzic i Melkior Ornik osvojili su broncane medalje (prag je bio 15bodova, a oni su imali redom 20, 19 i 17), dok su Nina Kamcev i Sara Muhvic dobilepohvalu za tocno rijesen jedan zadatak (i to na razlicitim zadacima!). Po ukupnombroju osvojenih bodova Hrvatska je bila 38., sto je najbolji uspjeh u zadnje tri godine.Kao i obicno, (po broju osvojenih bodova) pobijedili su Kinezi s pet zlatnih medaljai jednom srebrnom, dok je druga bila ekipa Rusije, ciji je svaki predstavnik osvojilozlatnu medalju.

Uvijek se moze bolje – Nini (koja je nakon drugog dana rjesavanja zadatakaotputovala u Vijetnam na Me -dunarodnu olimpijadu iz fizike, gdje je osvojila broncanumedalju) je nedostajao tek bod za broncu, a Adrianu dva boda za srebro. Ipak, sve tomoze se nadomjestiti sljedece godine, kada ce se odrzavati jubilarna 50. IMO u Bremenuu Njemackoj.

Trojica ucenika ostvarila su maksimalan broj bodova – dvojica iz Kine i jedan izSAD-a (ali kineskog porijekla!). Oni su na svecanosti zatvaranja dobili pljesak veci caki od princa od Asturije, koji je dosao da bi dodjeljivao medalje i svojom prisutnoscuuvelicao svecanost.

Nakon zatvaranja isli smo na zajednicki banket i druzenje na Univerzitetu Carlos III.Od raznih ekipa dobili smo sitne poklone za uspomenu, a i mi smo drugima dijeliliolovke. Mnogima je bilo tesko oprostiti se od prijatelja i simpatija koje su stekli na ovojolimpijadi, ali sve je proslo u prijateljskoj atmosferi. Bilo je lijepo vratiti se u Hrvatskui nadati se jos boljem uspjehu na sljedecoj Olimpijadi.

Adrijan Satja Kurdija i Melkior Ornik

120 Matematicko-fizicki list, LIX 2 (2008. – 2009.)

Page 57: Matematicko-fizicki list Zagreb

Zadaci

Prvi dan

Spanjolska, srijeda, 16. srpnja 2008.

1. Tocka H je ortocentar siljastokutnog trokuta ABC . Kruznica sa sredistem upolovistu tranice BC prolazi kroz H i sijece pravac BC u A1 i A2 . Slicno, kruznica sasredistem u polovistu od CA prolazi kroz H i sijece pravac CA u B1 i B2 ; kruznica sasredistem u polovistu od AB prolazi kroz H i sijece pravac AB u C1 i C2 . Dokazi dasve tocke A1 , A2 , B1 , B2 , C1 , C2 leze na kruznici.

(Rusija)

2. a) Dokazi da vrijedi:

x2

(x − 1)2+

y2

(y − 1)2+

z2

(z − 1)2≥ 1

za sve realne brojeve x , y , z , od kojih je svaki razlicit od 1, i zadovoljavaju uvjetxyz = 1.

b) Dokazi da jednakost vrijedi za beskonacno mnogo trojki racionalnih rojeva x , y ,z , od kojih je svaki razlicit od 1, i zadovoljavaju uvjet xyz = 1.

(Austrija)

3. Dokazi da postoji beskonacno mnogo prirodnih brojeva n takvih da je n2 + 1djeljivo prostim brojem koji je veci od 2n +

√2n .

(Litva)

Drugi dan

Spanjolska, srijeda, 17. srpnja 2008.

4. Na -di sve funkcije f : (0,∞) → (0,∞) ( f je funkcija sa skupa pozitivnh ealnihbrojeva u skup pozitivnh realnih brojeva) takvih da vrijedi

(f (w))2 + (f (x))2

f (y2) + f (z2)=

w2 + x2

y2 + z2,

za sve pozitivne realne brojeve w , x , y , z , koji zadovljavaju uvjet wx = yz .(Republika Koreja)

5. Neka su n i k prirodni brojevi takvi da je k ≥ n i k − n paran broj. Dano je 2nzarulja oznacenih s 1, 2 , . . . , 2n i svaka od njih moze biti ili upaljena ili ugasena. Napocetku su sve zarulje ugasene. Promatraj nizove koraka: u svakom koraku tocno jednazarulja promijeni svoje stanje (ako je bila upaljena, ugasi se, a ako je bila ugasena, upalise). Neka je N broj takvih nizova od kojih svaki ima k koraka i na kraju su sve zaruljeod 1 do n upaljene,dok su sve zarulje od n + 1 do 2n ugasene. Neka je M broj nizovaod kojih svaki ima k koraka, i na kraju su sve zarulje od 1 do n upaljene,dok su svezarulje od n + 1 do 2n ugasene, i nijedna od zarulja n + 1 do 2n nije se niti palila niti

gasila. Odredi omjerNM

.

(Francuska)

Matematicko-fizicki list, LIX 2 (2008. – 2009.) 121

Page 58: Matematicko-fizicki list Zagreb

6. Neka je ABCD konveksan cetverokut takav da je |BA| �= |BC| . Oznacimo kruzniceupisane rokutima ABC i ADC s ω1 i ω2 , tim redom. Pretpostavi da postoji kruznicaω koja dodiruje polupravac BA u tocki iza A i polupravac BC u tocki iza C , a istotako dodiruje pravce AD i CD . Dokazi da se zajednicke vanjske tangente kruznica ω1i ω2 sijeku na kruznici ω .

(Rusija)

Vrijeme za rad svakog dana: 4 sata i 30 minutaSvaki zadatak vrijedi 7 bodova

Rang–lista

nagrade broj nagrade broj

I II III poh. bod. I II III poh. bod.

Kina 5 1 217 Bosna i Hercegovina 3 1 68Rusija 6 199 Slovenija 2 1 68SAD 4 2 190 Svicarska 1 1 2 68Juzna Koreja 4 2 188 Svedska 1 3 67Iran 1 5 181 Danska 2 1 66Tajland 2 3 1 175 Kostarika 2 3 65Sjeverna Koreja 2 4 173 Malezija 1 4 65Turska 3 1 2 170 Austrija 1 4 63Tajvan 2 4 168 Norveska 1 1 62Ma -darska 2 3 1 165 Belgija 1 1 1 61Japan 2 3 1 163 Makedonija 2 61Vijetnam 2 2 2 159 Luksemburg (5) 2 2 60Poljska 2 3 1 157 Ta -dikistan 1 3 60Bugarska 2 1 3 154 Litva 1 2 58Ukrajina 2 2 2 153 Makau 2 1 58Brazil 5 1 152 Maroko 1 2 58Peru 1 3 2 141 Armenija 4 56Rumunjska 4 2 141 Portugal 2 1 55Australija 5 1 140 Albanija 1 1 53Njemacka 1 2 3 139 Cile (3) 1 1 1 49Srbija 1 3 2 139 Irska 2 45Kanada 2 4 135 Cipar 1 1 42Ujedinjeno Kraljevstvo 4 2 134 Novi Zeland 3 42Italija 3 3 132 Estonija 1 1 41Kazahstan 1 2 3 128 Finska 1 1 40Bjelorusija 3 2 125 Banglades (4) 1 33Izrael 1 1 2 2 120 Island (5) 1 31Hong Kong 3 1 1 107 Salvador (4) 3 31Mongolija 2 1 2 105 Sri Lanka 1 29Francuska 1 4 1 104 Trinidad i Tobago 1 28Indija 5 1 103 Kirgistan 5 1 28Singapur 1 3 1 98 Kuba (1) 1 27Nizozemska 2 2 94 Ekvador 1 26Uzbekistan 4 94 Kambodza 1 25Litva 1 2 3 92 Crna Gora (3) 2 24Indonezija 1 2 2 88 Paragvaj (4) 1 24Meksiko 1 1 4 87 Filipini 1 23Hrvatska 3 2 86 Urugvaj (5) 1 22Argentina 1 3 85 Tunis (4) 1 20Ceska 1 1 3 85 Honduras (2) 2 17Grcka 2 4 85 Gvatemala (4) 1 16Gruzija 5 1 84 Lihtenstajn (2) 1 16Spanjolska 3 3 82 Venecuela (2) 1 16Juznoafricka Republika 1 4 79 Portoriko (3) 9Kolumbija 2 1 77 Saudijska Arabija 8Slovacka 3 1 76 Bolivija (5) 5Turkmenistan 4 1 76 Ujedinjeni Arapski Emirati (4) 5Azerbajdzan 3 74 Kuvajt (5) 3Moldavija 1 2 74

122 Matematicko-fizicki list, LIX 2 (2008. – 2009.)

Page 59: Matematicko-fizicki list Zagreb

Druga srednjoeuropska matematicka olimpijada,Olomouc, Ceska, 4. – 10. rujna 2008.

Ilko Brnetic, Zagreb

Prosle smo godine s velikim zadovoljstvom docekali ra -danje novog regionalnognatjecanja u kojem sudjelujemo – Srednjoeuropske matematicke olimpijade. Ove smogodine imali priliku potvrditi uspjesan nastavak jedne lijepe price.

Ekipu za natjecanje smo birali u dvije faze i uz dosta zajednickih priprema. NaDrzavnom natjecanju smo uz odabir sestero ucenika za Me -dunarodnu matematickuolimpijadu, odabrali i siri krug od dvanaest potencijalnih kandidata za nastup naSrednjoeuropskoj matematickoj olimpijadi u Olomoucu. Nakon dijela dopisnih i kratkihzajednickih priprema u dva navrata odrzanih u Zagrebu, odrzano je i izborno dvodnevnonatjecanje. U ekipu su se konacno plasirali: Ivana Antolis (II. razred), Ivo Bozic (II.razred), Matko Ljulj (I. razred) i Irma Telarovic (III. razred) iz XV. gimnazije u Zagrebu,Grgur Valentic (I. razred) iz V. gimnazije u Zagrebu i Borna Cicvaric (II. razred)iz Gimnazije Andrije Mohorovicica u Rijeci. Moramo naglasiti da ostali kandidatinisu mnogo bodovno zaostali u tom zanimljivom natjecanju, kao i to da sudionicime -dunarodne matematicke olimpijade i maturanti, po pravilu, nisu imali pravo nastupana ovom natjecanju. Nakon izbora pripreme su nastavljene do pocetnog tjedna mjesecasrpnja, a nakon ljetnog odmora, clanovi ekipe okupili su se u Zagrebu nekoliko danaprije polaska na put.

Put u Olomouc, jedno od vodecih sredista pokrajine Moravske, vodio nas je prekoSlovenije i Austrije. Zahvaljujuci nenadanim popravcima na pruzi izmedju Maribora iGraza, umjesto predvi -dena dva presjedanja, imali smo ih cak pet, no, nakon 11 sativoznje stigli smo na odrediste. Nakon toga slijedilo je pet i pol dana intenzivnog rada,natjecanja, druzenja,. . .

Natjecanje i smjestaj bili su organizirani u Sveucilistu Jan Palacky. Ove godinenatjecalo se svih devet drzava koje su dogovorom i inicirale ovo natjecanje. Upravo jesudjelovanje Ma -darske i Njemacke, koje prosle godine nisu sudjelovale u natjecanju,pridonijelo znacajnom jacanju kvalitete natjecanja koje se sastoji od dva dijela:pojedinacnog i ekipnog.

U pojedinacnom dijelu sudjelovalo je 52 natjecatelja, po 6 iz svake drzave sudionice,samo iz Slovenije ih je sudjelovalo cetvero. Nasi su ucenici osvojili tri broncane medalje(Ivo Bozic, Matko Ljulj i Grgur Valentic) i jednu pohvalu (Borna Cicvaric). Ivo Bozicbio je nadomak srebrne medalje, nedostajao mu je tek bod, a Borna Cicvaric je jediniod nasih ucenika rijesio najtezi, geometrijski zadatak. U ekipnom dijelu natjecanja nasisu ucenici bili tek osmi, no uz neveliki zaostatak za bolje plasiranim ekipama.

Rezultatski gledano, najuspjesnije su bile tri objektivno najjace ekipe, Ma -darska,Poljska i Njemacka, dok su sve ostale, izuzev Slovenije, imale vrlo slicne rezultate. Mismo s time vrlo zadovoljni, jer smo imali najmla -du ekipu na natjecanju.

Proveli smo vrlo ugodne dane u Ceskoj; domacini su priredili bogat program izleta,tako da smo uz upoznavanja vrlo lijepog Olomouca, grada bogate povijesti i kulture,upoznali Kromeriz, grad sa spomenicima pod zastitom UNESCO-a, poznat ponajprijepo dvorcu i lijepim vrtovima, lijepu spilju Javoricko, te dvorac Bouzov.

Sljedece godine, natjecanje ce se odrzati u Poljskoj.

Matematicko-fizicki list, LIX 2 (2008. – 2009.) 123

Page 60: Matematicko-fizicki list Zagreb

Na kraju navodimo i zadatke s natjecanja (uz napomenu da je vrijeme rjesavanjasvakog dana bilo 5 sati), kao i rezultate.

Zadaci s pojedinacnog natjecanja

1. Neka je (an)∞n=1 niz prirodnih brojeva takav da je an < an+1 , ∀n � 1. Zasvaku cetvorku indeksa (i, j, k, m) , gdje je 1 ≤ i < j ≤ k < m i i + m = j + k , vrijedinejednakost ai + am > aj + ak . Odredi najmanju mogucu vrijednost broja a2008 .

2. Promatramo sahovsku plocu dimenzija n × n , gdje je n > 1 prirodan broj. Nakoliko nacina se moze postaviti 2n − 2 jednakih kamencica na tu sahovsku plocu (narazlicita polja) tako da nikoja dva od njih ne leze na istoj dijagonali sahovske ploce?(Dva su kamencica na istoj dijagonali sahovske ploce ako su sredista tih polja paralelnajednoj od dijagonala n × n kvadrata.)

3. Neka je ABC jednakokracan trokut u kojem je |AC| = |BC| . Njegova upisanakruznica dodiruje AB i BC u tockama D i E redom. Pravac (razlicit od AE ) prolazikroz A i sijece upisanu kruznicu u F i G . Pravac AB sijece pravce EF i EG redom uK i L . Dokazi da je |DK| = |DL| .

4. Odredi sve cijele brojeve k takve da su za sve cjelobrojne n , brojevi 4n + 1 ikn + 1 relativno prosti.

Zadaci s ekipnog natjecanja

1. Na -dite sve funkcije f : R → R takve da vrijedi

xf (x + xy) = xf (x) + f (x2)f (y)

za sve realne brojeve x i y .

2. Na ploci je napisano n ≥ 2 prirodnih brojeva. U svakom koraku biramo parbrojeva s ploce i zamijenimo svaki od njih njihovom sumom. Odredite sve vrijednostin za koje mozemo, nakon konacnog broja koraka, dobiti n -torku s jednakim brojevima.

3. Dan je siljastokutni trokut ABC . Neka je E tocka sa suprotne strane pravca ACu odnosu na tocku B , i neka je D tocka unutar duzine AE . Pretpostavimo da vrijedi<)ADB = <)CDE , <)BAD = <)ECD i <)ACB = <)EBA . Dokazite da tocke B , C i Eleze na istom pravcu.

4. Neka je n prirodan broj. Ako je suma svih pozitivnih djelitelja broja n potencijabroja 2, onda je i broj tih djelitelja tako -der potencija broja 2. Dokazite!

124 Matematicko-fizicki list, LIX 2 (2008. – 2009.)

Page 61: Matematicko-fizicki list Zagreb

Rezultati pojedinacnog i ekipnog natjecanja

drzava zlatnamedalja

srebrnamedalja

broncanamedalja

Ma -darska 3 3 0Poljska 1 4 1Njemacka 1 3 1Ceska 0 1 1Hrvatska 0 0 3Svicarska 0 0 3Austrija 0 0 3Slovacka 0 0 2Slovenija 0 0 0

poredak drzava brojbodova

1. Poljska 321. Ma -darska 321. Njemacka 324. Austrija 265. Slovacka 256. Svicarska 247. Ceska 228. Hrvatska 219. Slovenija 9

� � �

Matematicko-fizicki list, LIX 2 (2008. – 2009.) 125

Page 62: Matematicko-fizicki list Zagreb

Me -dunarodno matematicko natjecanje“Klokan bez granica” 2008. g.

Me -dunarodno matematicko natjecanje “Klokan bez granica” odr-zalo se deseti put pod pokroviteljstvom Ministarstva prosvjete isporta i Hrvatskog matematickog drustva 20. travnja ove godi-ne. S priblizno istim zadacima natjecalo se 5 000 000 ucenikau 42 zemlje svijeta, sto ovo natjecanje cini najvecim skolskimnatjecanjem u svijetu. Prema odjecima koji su stigli do nas,vjerujemo da je natjecanje postiglo svoju svrhu i zainteresiraloucenike za rjesavanje zadataka iz matematike. U Hrvatskoj su se

natjecali ucenici u 320 osnovnih i 120 srednjih skola u svim zupanija u sest kategorija:Leptirici, Ecolier, Benjamin, Cadet, Junior, Student. Ukupno se natjecalo 29 806ucenika.

Sudjelovalo je 6743 ucenika II. i III. razreda osnovne skole (L), 8365 ucenika IV. iV. razreda osnovne skole (E), 6166 ucenika VI. i VII. razreda osnovne skole (B), 4526ucenika VIII. razreda osnovne i I. razreda srednje skole (C). 2935 ucenika II. i III.razreda srednje skole (J) i 1071 ucenik IV. razreda srednje skole (S).

Najveci broj bodova koji se mogao ostvariti bio je 72 u kategoriji Leptirici, te 120bodova za sve ostale kategorije. Prilikom dolaska na natjecanje svaki je ucenik dobiopoklon, a 10% najboljih natjecatelja dobili su nagrade. Bodovni prag najboljih 10%natjecatelja bio je: za “Leptirice” – 50 bodova; “Ecolier” – 87.50 bodova; “Benjamin”– 77.50 bod; “Cadet” – 55.00 bodova; “Junior” – 60.25 bodova; “Junior” – matematickiprogram 75.00 bodova; “Student” 48.75 bodova; “Student” – matematicki program 66.25bodova.

Sljedeci zadaci mogu vas upoznati s proslogodisnjem natjecanjem i korisno posluzitiza pripreme za novo natjecanje koje ce se odrzati 19. ozujka 2009. godine.

Koordinator natjecanja “Klokan bez granica”, Neda Lukac, prof.

Zadaci za ucenike 8. razreda osnovne skole i 1. razreda srednje skole (Cadet)

Pitanja za 3 boda:1. U gusarskoj skoli, svaki ucenik mora sasiti svoju crno-bijelu zastavu. Pri tome

mora biti ispunjen uvjet da crni dio cini tri petine zastave. Koliko od prikazanih zastavaispunjava taj uvjet?

A. nijedna B. jedna C. dvije D. tri E. cetiri

2. U razrednom odjeljenju ima 9 djecaka i 13 djevojcica. Polovina djece u odjeljenjuima prehladu. Koliko djevojcica najmanje ima prehladu?

A. 0 B. 1 C. 2 D. 3 E. 4

126 Matematicko-fizicki list, LIX 2 (2008. – 2009.)

Page 63: Matematicko-fizicki list Zagreb

3. 6 klokana pojedu 6 vreca sijena u 6 minuta. Koliko klokana ce pojesti 100 vrecasijena za 100 minuta?

A. 100 B. 60 C. 6 D. 10 E. 600

4. Brojeve 2, 3, 4 i jos jedan nepoznati prirodni broj treba upisatiu kvadratice na slici. Zbroj brojeva u prvom retku mora biti 9, a zbrojbrojeva u drugom retku 6. Nepoznati broj je

A. 5 B. 6 C. 7 D. 8 E. 4

4cm

5. Trokut i kvadrat na slici imaju jednake opsege. Koliki jeopseg cijelog lika (peterokuta)?

A. 12 cm B. 24 cm C. 28 cm D. 32 cmE. ovisno o duljinama stranica trokuta

6. Cvjecarka Rina ima 24 bijele, 42 crvene i 36 zutih ruza. Koliko najvise jednakihkitica ona moze sloziti ako zeli upotrijebiti sve ruze?

A. 4 B. 6 C. 8 D. 10 E. 12

7. Koliko se kvadrata moze nacrtati spajanjem tocaka na slici?

A. 2 B. 3 C. 4 D. 5 E. 6

124 108

8. Tri pravca prolaze istom tockom. Velicine dvajukutova su 108◦ i 124◦ , kao sto se vidi na slici. Kolikostupnjeva ima kut osjencan sivom bojom?

A. 52◦ B. 53◦ C. 54◦ D. 55◦ E. 56◦

Pitanja za 4 boda:

9. Na kocki su odsijeceni vrhovi, kao na slici. Koliko bridovaima “nova” figura na slici?

A. 26 B. 30 C. 36 D. 40 E. neki drugi odgovor

10. Danijel ima 9 novcica (svaki vrijednosti 2 lipe), a njegova sestra Ana 8 novcicavrijednosti 5 lipa svaki. Koliko najmanje novcica trebaju razmijeniti me -dusobno Danijeli Ana da bi imali jednake novcane iznose?

A. 4 B. 5 C. 8 D. 12 E. nije moguca takva razmjena

11. Tom i Jerry razrezali su 2 sukladna pravokutnika. Tom je dobio 2 pravokutnika,svaki opsega 40 cm, a Jerry 2 pravokutnika, svaki opsega 50 cm. Koliki je bio opsegpocetnih pravokutnika?

A. 40 cm B. 50 cm C. 60 cm D. 80 cm E. 90 cm

Matematicko-fizicki list, LIX 2 (2008. – 2009.) 127

Page 64: Matematicko-fizicki list Zagreb

12. Jedna strana kocke razrezana je po njenim dijagonalama (kao naslici). Koja od sljedecih mreza nije mreza kocke na slici?

1 2 3 4 5

A. 1 i 3 B. 1 i 5 C. 3 i 4 D. 3 i 5 E. 2 i 4

13. Na pravcu su istaknute tocke A , B , C i D . Poznate su sljedece duljine:|AB| = 13, |BC| = 11, |CD| = 14 i |DA| = 12. Kolika je udaljenost dviju najudaljeni-jih tocaka?

A. 14 B. 38 C. 50 D. 25 E. drugi odgovor

P

R

Q14. U pravokutnik su upisane 4 kruzniceduljine polumjera 6 cm, kao na slici. Tocka P jevrh pravokutnika, a tocke Q i R diralista kruznicai stranica pravokutnika. Kolika je povrsina trokutaPQR?

A. 27 cm2 B. 45 cm2 C. 54 cm2 D. 108 cm2 E. 180 cm2

15. U kutiji se nalazi 7 karata. Na kartama su napisani brojevi od 1 do 7 (na svakojkarti tocno jedan broj). Matija uzima iz kutije nasumce 3 karte, a Luka 2 karte (2 sukarte ostale u kutiji ). Tada Matija kaze Luki: “Znam da je zbroj brojeva na tvojimkartama paran”. Zbroj brojeva Matijinih karata iznosi:

A. 10 B. 12 C. 6 D. 9 E. 15

16. Francuski matematicar August de Morgan imao je x godina u godini x2 . Umroje 1899. godine. Kada se rodio de Morgan?

A. 1806. B. 1848. C. 1849. D. 1899. E. drugi odgovor

Pitanja za 5 bodova:

17. Simetrala kuta <) C uz osnovicu |BC| jednakokracnog trokuta ABC sijece krakAB u tocki D . Ako je |BC| = |CD| , kolika je velicina kuta <)CDA?

A. 90◦ B. 100◦ C. 108◦ D. 120◦ E. nemoguce je odrediti

18. Drvena kocka 11× 11× 11 nastala je ljepljenjem 113 jedinicnih kocaka. Kolikose najvise jedinicnih kocaka moze vidjeti gledajuci iz iste tocke gledanja?

A. 328 B. 329 C. 330 D. 331 E. 332

19. U “Malim astronomima” djevojke cine vise od 45%, a manje od 50%. Koji jenajmanji moguci broj djevojaka u toj grupi?

A. 3 B. 4 C. 5 D. 6 E. 7

128 Matematicko-fizicki list, LIX 2 (2008. – 2009.)

Page 65: Matematicko-fizicki list Zagreb

20. Djecak uvijek govori istinu cetvrtkom i petkom, uvijek laze utorkom, a ostaledane u tjednu govori istinu ili laze bez pravila. Pitali su ga, sedam dana uzastopno, zanjegovo ime i odgovori prvih 6 dana bili su: Ivan, Branko, Ivan, Branko, Petar, Branko.Sto je djecak odgovorio sedmog dana?

A. Ivan B. Branko C. Petar D. Katarina E. drugi odgovor

21. Martina i Ivica su krenuli planinariti. U selu, u podnozju planine, procitali suoznaku na kojoj pise da do vrha ima 2 sata i 55 minuta (pjesacenja). Napustili suselo u 12 sati. U 13 sati stali su radi kratkog odmora i procitali novu oznaku na kojojpise da do vrha ima samo 1 sat i 15 minuta. Nakon 15 minuta nastavili su put is-tom brzinom kao i prije i nisu se vise zaustavljali do vrha. U koje su vrijeme stigli na vrh?

A. 14 : 30 B. 14 : 00 C. 14 : 55 D. 15 : 10 E. 15 : 20

22. Nazovimo tri prosta broja “specijalnim” ako je njihov umnozak 5 puta veci odnjihovog zbroja. Koliko “specijalnih” trojki prostih brojeva postoji?

A. 0 B. 1 C. 2 D. 4 E. 6

23. Zadana su dva skupa A i B peteroznamenkastih prirodnih brojeva. U skupu Asu brojevi ciji je umnozak svih znamenki 25, a u skupu B brojevi ciji je umnozak svihznamenki 15. Koji skup ima vise brojeva? Koliko puta vise?

A. skup A ,53

puta B. skup A , 2 puta C. skup B ,53

puta

D. skup B , 2 puta E. oba skupa imaju jednaki broj clanova

24. Najveci zajednicki djelitelj dvaju prirodnih brojeva m i n je 12, a njihov najmanji

zajednicki visekratnik je kvadrat. Izme -du 5 brojeva:n3

,m3

,n4

,m4

, m · n koliko njih

su kvadrati?

A. 1 B. 2 C. 3 D. 4 E. nemoguce je odrediti

Zadaci za ucenike 2. i 3. razreda srednje skole (Junior)

Pitanja za 3 boda:1. Dano je 5 kutija, a u svakoj od njih nalaze se kartice sa slovima A , E , I , O , U

kao sto je prikazano na slici. Petar je izbacio neke kartice tako da je na kraju svakakutija sadrzavala jednu karticu, a razlicite kutije sadrzavale su razlicita slova. Koja jekartica ostala u kutiji broj 2?

A. A B. E C. I D. O E. U

Matematicko-fizicki list, LIX 2 (2008. – 2009.) 129

Page 66: Matematicko-fizicki list Zagreb

2. Franjo i Gabrijela natjecali su se u trcanju na 200 metara. Gabrijela je stazupretrcala za pola minute, a Franjo za jednu stotinu sata. Tko je i za koliko sekundi biobrzi?

A. Gabrijela za 36 sekundi B. Franjo za 24 sekunde C. Gabrijela za 6 sekundiD. Franjo za 4 sekunde E. Imaju ista vremena

3. Na proslavi ovogodisnje Nove godine, Borna je obukao majicu s brojem godine ipred ogledalom izveo je stoj na rukama. Koju je sliku u ogledalu vidio njegov prijateljVinko stojeci iza Borne, normalno na nogama?

A. B. C. D. E.

4. Zadani su brojevi a = 2 − (−4) , b = (−2) · (−3) , c = 2 − 8, d = 0 − (−6) ie = (−12) : (−2) . Koliko od njih nije jednako broju 6?

A. 0 B. 1 C. 2 D. 4 E. 5

A

B 5. Kolika je duljina duzine AB , ako je duljinastranice svakog od kvadrata na slici 1 metar?

A. 5 B.√

13 C.√

5 +√

2

D.√

5 E. nijedna od danih

6. Koliko se najmanje slova moze maknuti iz rijeci KANGOUROU tako da supreostala slova abecedno poslagana?

A. 1 B. 2 C. 3 D. 4 E. 5

7. U kutiji je 7 karata. Brojevi od 1 do 7 napisani su na njima i to tocno jedan brojna jednoj karti. Prvi igrac iz kutije na slucajan nacin odabere 3 karte, a drugi igrac uzme2 karte. Tada prvi igrac kaze drugome: “Znam da je zbroj brojeva na tvojim kartamaparan.” Koliki je zbroj brojeva na kartama prvog igraca? (Igraci ne vide preostale karteu kutiji.)

A. 10 B. 12 C. 6 D. 9 E. 15

8. Tom i Jerry dobili su jednake papire u obliku pravokutnika. Svaki od njih je svoj pa-pir razrezao na dva jednaka pravokutnika. Tom je izrezao pravokutnike s opsegom 40 cm,a Jerry dva pravokutnika s opsegom 50 cm. Koliki je bio opseg pocetnog komada papira?

A. 40 cm B. 50 cm C. 60 cm D. 80 cm E. 100 cm

Pitanja za 4 boda:9. Na prvom testu Ana je osvojila 1 bod od mogucih 5. Na sljedecim testovima

osvojila je sve bodove od mogucih 5. Koliko je jos testova trebala pisati da joj prosjecanbroj bodova bude 4 po testu?

A. 2 B. 3 C. 4 D. 5 E. 6

130 Matematicko-fizicki list, LIX 2 (2008. – 2009.)

Page 67: Matematicko-fizicki list Zagreb

10. Marko ima 10 kartica, a na svakoj od njih se nalazi tocno jedan od ovih brojeva:3, 8, 13, 18, 23, 28, 33, 48, 53, 68. Koliko najmanje kartica Marko moze izabrati takoda zbroj brojeva na njima bude jednak 100?

A. 2 B. 3 C. 4 D. 5 E. to je nemoguce uciniti

11. Sedam je patuljaka ro -deno istog dana u 7 uzastopnih godina. Tri najmla -dazajedno imaju 42 godine. Koliko godina zajedno imaju tri najstarija patuljka?

A. 51 B. 54 C. 57 D. 60 E. 63 12. Dva pravilna sesterokuta na slici su jednaka. Ko-

lika je povrsina osjencanog dijela paralelograma?

A.12

B.13

C.14

D.15

E.16

13. Sest prirodnih brojeva oznaceno je slovima na brojevnom pravcu. Poznato je da subarem dva od njih djeljivi s 3 i barem dva od njih djeljivi s 5. Koji su brojevi djeljivi s 15?

A B C E D F

A. A i F B. B i D C. C i E D. svi E. samo jedan od njih

14. Koliko najvise znamenaka treba obrisati iz 1000-znamenkastog broja20082008. . .2008 tako da zbroj preostalih znamenaka bude 2008?

A. 746 B. 510 C. 524 D. 1020 E. 130

15. Na slici je dan jednakokracan trokut u kojem je|AB| = |AC| . Ako je PQ okomito na AB , kut BPC je 120◦ ikut ABP je 50◦ , koliki je kut PBC?

A. 5◦ B. 10◦ C. 15◦ D. 20◦ E. 25◦

16. Koliko postoji parova realnih brojeva ciji su zbroj, produkti kvocijent me -dusobno jednaki?

A. nema takvih parova B. 1 par C. 2 para D. 4 para E. 8 parova

Pitanja za 5 bodova:17. Svaka znamenka pocevsi od trece (brojano slijeva) u sesteroznamenkastom

broju jednaka je zbroju prethodne dvije znamenke (koje se nalaze slijeva). Kolikosesteroznamenkastih brojeva ima to svojstvo?

A. nijedan B. 1 C. 2 D. 4 E. 6

18. Imam drvenu kocku cije tri strane su plave, a tri crvene. Kad je prerezem u3×3×3 = 27 jednakih malih kocaka, koliko od njih ima barem dvije razlicitim bojamaobojane strane

A. 6 B. 12 C. 14 D. 16E. ovisi o tome koje su strane velike kocke bile obojane plavo i crveno

Matematicko-fizicki list, LIX 2 (2008. – 2009.) 131

Page 68: Matematicko-fizicki list Zagreb

19. Oznacimo s n! = 1 · 2 · 3 · . . . · (n − 1) · n . Ako je n! = 215 · 36 · 53 · 72 · 11 · 13,tada je n jednako

A. 13 B. 14 C. 15 D. 16 E. 17

20. Kolika je duljina istaknutog (veceg) luka na kruznicipolumjera 1?

A.5π4

B.5π3

C.π2

D.3π2

E.2π3

21. Kvadrat 4×4 podijeljen je na 16 jedinicnih kvad-ratica. Na -di najveci broj dijagonala koje se mogu povuci utim kvadraticima tako da nikoje dvije nemaju zajednicke tocke.

A. 8 B. 9 C. 10 D. 11 E. 12

22. Kanga uvijek skace po 1 m ili po 3 m. Zeli skociti tocno 10 metara. Na kolikonacina moze Kanga izvesti to skakanje? (Skokove 1+3+3+3 i 3+3+3+1 smatramorazlicitim nacinima.)

A. 28 B. 34 C. 35 D. 55 E. 56

23. Kvadrat ABCD na slici ima stranicu duljine 1, i lu-kovima su spojeni odgovarajuci nasuprotni vrhovi kvadrata.Kolika je udaljenost tocaka P i Q?

A. 2 −√2 B.

34

C.√

5 −√2 D.

√3

2E.

√3 − 1

24. Koliko postoji 2007-teroznamenkastih brojeva s ovim svojstvom: svaki dvozna-menkasti broj sacinjen od dvije uzastopne znamenke pocetnog broja je djeljiv ili sa 17ili s 23?

A. 5 B. 6 C. 7 D. 9 E. vise od 9

Zadaci za za ucenike 4. razreda srednje skole (Student)

Pitanja za 3 boda:1. Brojevi 3, 4 i jos dva nepoznata broja upisani su u polja tablice 2×2. Poznato je

da je zbroj brojeva po retcima jednak 5 i 10, a zbroj po jednom stupcu je 9. Veci od tadva nepoznata broja je

A. 5 B. 6 C. 7 D. 8 E. 3

2. Ako je x + y = 0 i x �= 0, tada jex2008

y2008=

A. −1 B. 0 C. 1 D. 22008 E.xy

132 Matematicko-fizicki list, LIX 2 (2008. – 2009.)

Page 69: Matematicko-fizicki list Zagreb

3. Matrica se sastoji od 21 stupca oznacenih brojevima 1,2,. . . ,21 i 33 retka nume-riranih brojevima 1,2,. . . ,33. Obrisemo one retke ciji redni broj nije visekratnik od 3 isve one stupce ciji redni broj je paran. Koliko je u matrici ostalo polja?

A. 110 B. 121 C. 115.5 D. 119 E. 242

4. Koliko prostih brojeva p ima svojstvo da je i broj p4 + 1 prost broj?

A. nijedan B. 1 C. 2 D. 3 E. beskonacno

18

58

23

A

B

5. Promatramo rijeku od tockeA . U svom toku racva se na dva

rukavca. Prvi rukavac preuzima23

kolicine vode, a drugi ostatak. Nakontoga se prvi rukavac opet racva u tri,pri cemu jedan novi rukavac preuzima

18

vode,

drugi58

, a treci ostatak. Nakon nekog vremena, taj treci se

rukavac spaja s onim rukavcem rijeke nastalim pri prvom racvanju. Mapa rijekeprikazana je na slici. Kolika kolicina vode dolazi u mjesto B?

A.13

B.54

C.29

D.12

E.14

6. U kutiji je 7 karata. Brojevi od 1 do 7 napisani su na njima i to tocno jedan brojna jednoj karti. Prvi igrac iz kutije na slucajan nacin odabere 3 karte, a drugi igrac uzme2 karte. Tada prvi igrac kaze drugome: “Znam da je zbroj brojeva na tvojim kartamaparan.” Koliki je zbroj brojeva na kartama prvog igraca?

A. 10 B. 12 C. 6 D. 9 E. 15

7. Dan je jednakokracan trokut ABC (|CA| = |CB|) .Na osnovici je oznacena tocka D tako da je |AD| = |AC|i |DB| = |DC| . Koliki je kut ACB?

A. 98◦ B. 100◦ C. 104◦ D. 108◦ E. 110◦

8. Najveca vrijednost funkcije f (x) = |5 sin x − 3| u skupu R iznosi:

A. 2 B. 3 C. π D. 5π E. 8

Pitanja za 4 boda:

9. Svaka od kocaka na slici ima bridove duljine 1. Kolika jeudaljenost od tocke A do tocke B?

A.√

17 B. 7 C.√

13 D.√

7 E.√

14

10. Matilda je nacrtala 36 klokana koristeci tri razlicite boje. 25 klokana ima nestoobojano zutom, 28 ih ima nesto sme -de, a za 20 klokana je koristila crnu boju. Samo 5od njih ima na sebi sve tri boje. Koliko klokana imaju na sebi samo jednu boju?

A. 0 B. 4 C. 12 D. 31 E. nemoguce je odrediti

Matematicko-fizicki list, LIX 2 (2008. – 2009.) 133

Page 70: Matematicko-fizicki list Zagreb

11. Tri se kruga diraju u parovima kao na slici.Polumjer svakoga je r . Kolika je povrsina podrucja Aome -denog s ta tri kruga?

A.(√

3 − π2

)r2 B.

(π2−

√3

2

)r2 C.

18πr2

D.(√

3 − 32

)πr2 E.

(π3−

√3

2

)r2

12. Dva pravilna sesterokuta na slici su jednaka. Ko-lika je povrsina osjencanog dijela paralelograma?

A.12

B.13

C.14

D.15

E.16

13. Na natjecanju Matematicki Kup zadano je 5 problema koji svi nose razliciti brojbodova. Damjan je rijesio svih pet i pri tome je osvojio svih 10 bodova koje je bilomoguce osvojiti na dva problema s najmanjim brojem bodova. Osim toga, za dva proble-ma s najvecim brojem bodova dobio je svih 18 bodova. Koliko je bodova osvojio Damjan?

A. 30 B. 32 C. 34 D. 35 E. 40

14. Brojnik i nazivnik razlomka su negativni cijeli brojevi. Brojnik je veci odnazivnika. Koja od sljedecih tvrdnji o tom razlomu je istinita?

A. Razlomak je manji od −1. B. Razlomak je izme -du −1 i 0.C. Razlomak je pozitivan broj manji od 1. D. Razlomak je veci od 1.E. Ne moze mu se odrediti predznak.

� � � �

� � � �

� � � �

15. Tri su tocke izabrane na slucajan nacin iz skupa tocaka naslici. Kolika je vjerojatnost da su te tocke kolinearne?

A.112

B.111

C.116

D.18

E.312

16. Ako je x2yz3 = 73 i xy2 = 79 , tada je xyz =

A. 74 B. 76 C. 78 D. 79 E. 710

Pitanja za 5 bodova:17. Duljine stranica kvadra u centrimetrima su prirodni brojevi koji cine geometrijski

niz s kvocijentom 2. Koji od brojeva mogu biti obujam tog kvadra?

A. 120 cm2 B. 188 cm2 C. 216 cm2 D. 350 cm2 E. 500 cm2

* * * 1 * *

2 2 * *

5 6 * * *

×

+ * * 2

9 0 *

18. U ovom racunu svaka zvjezdica oznacava jednu znamenku.Koliki je zbroj svih znamenaka u rezultatu ovog mnozenja?

A. 16 B. 20 C. 26 D. 30 E. neki drugi odgovor

19. Na -di vrijednost izraza x2 + y2 + z2 , ako je x + y + z = 1 i1x

+1y

+1z

= 0.

A. 0 B. 1 C. 2 D. 3 E. nemoguce je odrediti

134 Matematicko-fizicki list, LIX 2 (2008. – 2009.)

Page 71: Matematicko-fizicki list Zagreb

20. Kruznica je upisana trokutu ABC , pri cemu je |AC| = 5,|AB| = 6 i |BC| = 3. Duzina ED dodiruje kruznicu. Koliki jeopseg trokuta ADE?

A. 7 B. 4 C. 9 D. 6 E. 8

21. Niz je zadan ovako: an+1 = an + (−1)n · n , za n � 1, ia1 = 0. Ako je ak = 2008, koliki je k?

A. 2008 B. 2009 C. 4017 D. 4018 E. neki drugi

22. Broj 332 − 1 ima tocno dva djelitelja koji su veci od 75 i manji od 85. Koliki jeprodukt tih djelitelja?

A. 5852 B. 6560 C. 6804 D. 6888 E. 6972

23. Kvadrat ABCD ima stranice duljine 1. Ako je tocka Mpoloviste stranice AB , kolika je povrsina osjencanog podrucja?

A.124

B.116

C.18

D.112

E.213

24. Ako je sin x + cos x = m , tada je sin4 x + cos4 x =

A. 1 − (1 − m2)2

2B. 1 +

(1 − m2)2

2C.

1 − (1 − m2)2

2D. m4 E. m4 + 1

Rjesenja

Cadet1. C 2. C 3. C 4. D 5. D 6. B 7. C 8. A9. C 10. B 11. C 12. D 13. D 14. D 15. B 16. A17. C 18. D 19. C 20. A 21. B 22. B 23. B 24. B

Junior1. D 2. C 3. B 4. B 5. B 6. D 7. B 8. C9. B 10. D 11. B 12. A 13. A 14. A 15. A 16. B17. D 18. E 19. D 20. D 21. A 22. A 23. E 24. A

Student1. B 2. C 3. B 4. B 5. D 6. B 7. D 8. E9. A 10. B 11. A 12. A 13. D 14. C 15. B 16. A17. C 18. A 19. B 20. E 21. C 22. B 23. D 24. A

� � �

Obavijesti o ovom natjecanju mogu se dobiti na internetskoj stranicihttp://www.math.hr/hmd.

Matematicko-fizicki list, LIX 2 (2008. – 2009.) 135

Page 72: Matematicko-fizicki list Zagreb

Zarko Dadic, Egzaktne znanosti u Hrvata u poslijeprosvjetiteljskom razdoblju(1789.–1835.), edicija: Posebna izdanja, urednik: Milan Miric, Naklada Ljevak, Zagreb,2007., 326 str.

Najnovijom knjigom, o kojoj je ovdje rijec, akademik ZarkoDadic obradovao je sve one koji su ocekivali i napokon docekali dase objelodane rezultati njegovog dugotrajnog ustrajnog i predanogznanstveno-istrazivackog rada. U knjizi se tako objavljuju izvornaautorova istrazivanja koja je on obavljao tijekom vise od dvadesetgodina. Ova knjiga se nadovezuje na knjigu Egzaktne znanostiu Hrvata u doba prosvjetiteljstva koju je godine 2004. objavilaMatica hrvatska a ujedno je jedna od niza knjiga u kojima autorobra -duje cjelokupnu povijest egzaktnih znanosti u Hrvata. Razdobljeod Francuske revolucije do hrvatskog narodnog preporoda, koje jeobra -deno u ovoj knjizi, u pogledu egzaktnih znanosti u Hrvata biloje dosad najmanje poznato. Da bi uzitak pri citanju bio sto veci

pobrinuli su se, uz autora i urednika, recenzenti dr. sc. Mijo Korade i dr. sc. TrpimirMacan. Uz skraceni upadljivi naslov na omotu korica knjige otisnut je fotografskiprikaz u boji armilarnih sfera iz zbirke obitelji Garanjanin u Trogiru. Sfere se cuvaju uGradskom muzeju u Trogiru.

U predgovoru knjige autor je opisao doga -danje u filozofiji, znanosti, knjizevnosti ikulturi nakon Francuske revolucije 1789. pa do 1830.–1840., u Francuskoj, Njemackoji Habsburskoj monarhiji. To razdoblje moglo se s jednog motrista oznaciti kaopostjozefinsko, a drugoga kao pretpreporodno, uz napomenu da hrvatski narodnipreporod pocinje 1835. godine. Autor se stoga odlucio da to razdoblje oznaci kaoposlijeprosvjetiteljsko buduci da je najveci dio knjige posvetio prirodnofilozofskim iprirodoznanstvenim zbivanjima s poslijeprosvjetiteljskog motrista. Jos jedan razlog za toje i cinjenica da se ta knjiga nadovezuje na spomenutu knjigu iz 2004., i u odre -denomsmislu s njom cini cjelinu.

Autor je dao detaljan uvod u tematiku, naime prikazao je osobe vazne za razvojprirodnofilozofskih i prirodnoznanstvenih shvacanja u poslijeprosvjetiteljskom dobu, akoje su djelovale u Francuskoj i Njemackoj. Isto tako naglasena je njihova uloga iobrazovnom sustavu (skole!). Skole u kojima su te osobe djelovale utemeljile su novimodel razvitka znanstvenih istrazivanja. Autor je naglasio i utjecaj tih skola na visokeskole u Habsburskoj monarhiji sto je znacajno za hrvatske znanstvenike koji su djelovaliu zemljama monarhije, a izvan Hrvatske.

U vecem dijelu knjige su analizirana gotovo sva djela hrvatskih znanstvenika kojisu djelovali na sveucilistu u Budimu, odnosno Pesti, te na akademijama u Ma -darskoj iSlovackoj u tom razdoblju. Knjiga pokazuje da je na Filozofskom fakultetu u Budimu,odnosno u Pesti mnogo Hrvata u tom razdoblju predavalo matematiku i fiziku, a da ihje mnogo bilo i na Zvjezdarnici u Budimu. Tako su, izme -du ostalih, na Filozofskomfakultetu predavali Ivan Paskvic, Franjo Josip Domin i Franjo Bruna, a u jednomrazdoblju su na zvjezdarnici djelovali Mirko Danijel Bogdanic, Franjo Bruna i IvanPaskvic. Osim toga je autor vrlo iscrpno prikazao rad Ignjata Martinovica u matematicii fizici, koji je bio najprije u franjevackom redu, a kad je izasao iz njega bio je profesoru Lavovu. Autor temeljito ispravlja neispravne tvrdnje o etnickoj pripadnosti Ignjata

136 Matematicko-fizicki list, LIX 2 (2008. – 2009.)

Page 73: Matematicko-fizicki list Zagreb

Martinovica za kojega dokazuje da je hrvatskog podrijetla. Autor je pronasao mnogedosad nepoznate rukopise spomenutih znanstvenika, posebno u Sveucilisnoj knjizniciu Budimpesti i u knjiznici zvjezdarnice Ma -darske akademije. Na temelju tiskane irukopisne gra -de prikazan je cjelokupni rad spomenutih hrvatskih znanstvenika u tomrazdoblju te je pokazana geneza njihova doprinosa. Na temelju njihove korespondencijesu prikazani njihovi kontakti sa znanstvenicima u velikim znanstvenim sredistima uEuropi. Osobito je na temelju izvora po prvi put opsezno razjasnjem sukob hrvatskogastronoma Paskvica koji je pocetkom 19. stoljeca bio ravnatelj zvjezdarnice u Budimu snjegovim asistentom Danielom Kmethom, pri cemu su najveci astronomi toga vremena,primjerice Gauss, dokazali da je Paskvica neosnovano napao Kmeth i da ga je htioznanstveno diskreditirati. Koristeni su gotovo svi astronomski casopisi u kojima su seobjavljivali rezultati motrenja hrvatskih astronoma na zvjezdarnici u Budimu u to doba.U knjizi je obra -dena i znanstvena situacija u Hrvatskoj u to doba, posebno na Kraljevskojakademiji u Zagrebu. Obra -deni su rukopisi sacuvani u franjevackim knjiznicama te jeanalizirano skolstvo u hrvatskim krajevima. Posebna je pozornost posvecena promjeniprograma nastave u skolama u Dalmaciji nakon sto su je okupirali Francuzi, a onda inakon Austrijske okupacije kad je skolstvo u Dalmaciji ujednaceno s programima kojeje vrijedilo za sjevernu Hrvatsku.

U pogovoru naglasava se znacenje 1835. godine na svjetskoj pozornici znanstvenihzbivanja kada prirodne znanosti odbacuju metafiziku, a unutar njih pojavljuju sespecijalnosti. Autor je, uz to, istaknuo da i u matematici od tog vremena pocinje jednosasvim novo razdoblje. Godina 1835. znacajna je i za Hrvatsku, jer se usporednos politickim djelovanjem pojavljuju i nastojanja za uspostavu nacionalnih kulturnihi znanstvenih ustanova. Od tada hrvatski znanstvenici uglavnom djeluju u domovini,osnivaju domace znanstvene ustanove i drustva, i zapocinju sustavni i moderni znanstvenirad u Hrvatskoj – zakljucio je autor.

Na kraju valja istaci da u pogledu sustavnosti, detaljnosti i cjelokupnosti ovaknjiga nadilazi povijesno-znanstveni i povijesno-filozofski pristup. To je vrhunski prikaznastanka egzaktne novovjeke znanosti u Hrvata u spomenutom razdoblju. Zbog toga jeknjiga ne samo nezaobilazna literatura u proucavanju te tematike, nego je jedinstvena upogledu sintetickog pristupa. Nakladniku pripada zasluga sto je izdavanjem ove knjigeuvelike obogatio istrazivanje prave povijesne istine naseg naroda.

Branko Hanzek, Zagreb

� � �

PAZNJA! — STARI BROJEVI — U nasem skladistu ima starih brojeva, i to: god.XVI, br. 4; god. XXXII, br. 3; god. XXXIII, br. 4; god. XXXIV, br. 3, 4; god. XXXV,br. 3; god. XXXVI, br. 1, 2, 3, 4; god. XXXVII, br. 1, 4; god. XXXIX, br. 1, 2, 3, 4;god. XL, br. 2, 3, 4; god. XLI, br. 1, 2, 3, 4; god. XLII, br. 3-4; god. XLIV, br. 1, 2, 3,4; god. XLV, br. 1, 2, 3, 4; god. XLVI, br. 1, 2, 3, 4; god. XLVII, br. 1, 2, 3, 4; god.XLVIII, br. 1, 2, 3, 4; god. XLIX, br. 1, 2, 3, 4; god. L, br. 1, 2, 3, 4; god. LI, br. 1, 2,3, 4; god. LII, br. 1, 2, 3, 4; god. LIII, br. 1, 2, 3, 4; god. LIV, br. 1, 2, 3, 4; god. LV,br. 1, 2, 3, 4; god. LVI, br. 1, 2, 3, 4; god. LVII, br. 1, 2, 3, 4; god. LVIII, br. 1, 2, 3, 4.

Cijena pojedinog broja je 20 kuna.Izvanredni broj (E) – zadaci iz matematike (cijena 20 kn); Izvanredni broj (F) –

Rjecnik matematickih naziva – hrvatski, engleski, njemacki (cijena 30 kn); Izvanrednibroj (H) – zadaci iz matematike (cijena 20 kn).

Matematicko-fizicki list, LIX 2 (2008. – 2009.) 137

Page 74: Matematicko-fizicki list Zagreb

Razredbeni ispit na FOI-u iz matematike 2008. g.

Donosimo zadatke s razredbenog ispita na Sveucilistu u Zagrebu, Fakultet organizacijei informatike, Varazdin, 2008. g. Do 2008. godine bilo je po 20 zadataka iz matematike,informatike i hrvatskog jezika, a od 2009. godine ce se sastojati od 30 zadataka izmatematike i 30 zadataka iz informatike. Svaki tocno rijesen zadatak nosi 10 bodovai nema negativnih bodova za netocno rijesene zadatke. U svakom zadatku ima 4ponu -dena odgovora od kojih je jedan tocan. Za prijelaz praga treba skupiti najmanje200 bodova, ali mora biti ispunjen i uvjet da je 100 bodova minimalno iz matematikei 100 iz informatike. Dakle, ako bi pristupnik imao samo 200 bodova i to samo izinformatike, on ne bi presao prag. Vise informacija o upisima na FOI, mozete naci nahttp://www.foi.hr/upisi/. Ovdje donosimo 60 zadataka iz matematike, koji su bilina razredbenom ispitu 2008. godine.

Grupa A

1. Algebarski razlomak

(x +

xy + y2

x + y

)·(

1 + a3

x2 − y2:

1 − a + a2

x − y

)za |x| �= |y| je

jednak

A. 1 + a B. x + y C.1 − ax + y

D.axy

2. Ostatak pri dijeljenju polinoma p(x) = x10 + 2x5 + 1 s polinomom q(x) = x + 1jednak jeA. x B. 2 C. 0 D. −1

3. Apsolutna vrijednost kompleksnog broja2 − i3 − 2i

jednaka je

A.

√65

13B. 4 C.

√13 D.

√134

4. Rjesenje nejednadzbe log (x + 2) � 1 jeA. [−2, 8] B. 〈−∞,−2] C. 〈−∞, 8] D. 〈−2, 8]

5. Ako je g(x) = 2x − 8 i h(x) = x2 − 2, tada je g(h(2)) jednakoA. −4 B. 4 C. x D. 2

6. Neka je f (x) = |x + 3| − 4. Minimalna vrijednost funkcije f jeA. 2 B. −4 C. 3 D. −3

7. Koliko realnih rjesenja ima jednadzba x3 + 3x2 − x − 3 = 0?A. 1 B. 2 C. 3 D. 4

8. Rjesenje kvadratne nejednadzbe −x2 + 5x + 6 � 0 jeA. [−1, 6] B. 〈−∞, 6] C. [−1, +∞〉 D. 〈 0, +∞〉

9. Vrhovi cetverokuta su redom tocke A(3, 2) , B(1,−2) , C(2,−5) , D(4,−1) . Tajcetverokut jeA. kvadrat B. pravokutnik C. paralelogram D. romb

138 Matematicko-fizicki list, LIX 2 (2008. – 2009.)

Page 75: Matematicko-fizicki list Zagreb

10. Ako se radijus sfere udvostruci, sto ce se dogoditi s oplosjem sfere?A. porast ce za faktor 2 B. porast ce za faktor 4

C. porast ce za faktor 6 D. porast ce za faktor 8

11. Volumen uspravnog stosca je 12π , a polumjer baze r = 3. Visina stosca jeA. 1 B. 2 C. 3 D. 4

12. Mate i Sanja su udaljeni 2000m i oboje gledaju balon. U isto vrijeme, Sanja vidibalon pod kutem od 42◦ , a Mate pod kutem od 70◦ . Sanja je udaljena od balonaA. 2026.98 m B. 1984.37 m C. 1443.36 m D. 2086.95 m

13. Opseg paralelograma na slici je 80 cm. Povrsina tog paralelograma je

A. 276 cm2 B. 144 cm2 C. 138 cm2 D. 84 cm2

14. Zbroj rjesenja jednadzbe 22x + 2 · 2x − 8 = 0 je

A. −2 B. 1 C. 4 D.23

15. Krivulje y = x2 + 2 i y = 4−x se sijeku uA. I. kvadrantu B. II. kvadrantu C. III. kvadrantu D. IV. kvadrantu

16. Srediste i radijus kruznice x2 + x + y2 − 2y =34

je

A. S(1

2, 1)

, r = 2 B. S(−1

2, 1)

, r =√

2

C. S(−1

2,−1

), r =

34

D. S(−1

2, 1)

, r = 2

17. Domena funkcije f (x) =√

ax − 1 za 0 < a < 1 jeA. 〈−∞, 0] B. [0, +∞〉 C. 〈 1, +∞〉 D. 〈−∞,−1]

18. Cetvrti clan niza an =n · cos

(n · π

2

)n!

jednak je

A.12

B.14

C.16

D.18

19. Odredite realni parametar a tako da su zadani pravci ax + (a − 1)y = 5 ix + ay = 0 okomiti.A. 0 B. −2 C. −3 D. 1

20. Omjer povrsina baza krnje piramide jednak jeA. omjeru visine krnje piramide i visine cijele piramideB. volumenu krnje piramideC. omjeru kvadrata njihovih udaljenosti od vrha potpune piramideD. omjeru njihovih udaljenosti od vrha potpune piramide

Matematicko-fizicki list, LIX 2 (2008. – 2009.) 139

Page 76: Matematicko-fizicki list Zagreb

Grupa B

1. Algebarski izraza3 − 8b3

a2 − 4b2:

a2 + 2ab + 4b2

a2 + 2abje jednak

A. 1 B. a C. a + 2b D. 1 − ab

2. Izraz 6√

3x2y : 3√

9xy2 ·√

27x4y3 je jednakA. 3x2y B. 6

√3x2y3 C. 3

√xy D. 9xy2

3. Imaginarni dio kompleksnog broja1

−1 + 2ije jednak

A. 0 B. −12

C. −25

D.52

4. Kakav treba biti realni parametar a tako da kvadratna jednadzba −3x2−ax−4 = 0nema realnih rjesenja?A. a ∈ 〈−4

√3, 4

√3〉 B. a ∈ 〈−∞, 4

√3〉 C. a ∈ Re D. a ∈ [−√

48, 0]5. Luka je ispucao loptu. Visina u metrima h(t) koju dostize lopta nakon t sekundi

je h(t) = −16t2 + 48t . Lopta ce udariti u tlo nakon

A. 1.5 s B. 2 s C.258

s D. 3 s

6. Ako je f (x) = 3x−2 i g(x) = x2 − x , tada je g(f (3)) jednakoA. 6 B. 3 C. 2 D. −4

7. Rjesenje nejednadzbex2 − 7x + 10

x − 4> 0 je

A. x ∈ 〈 2, 4〉 ∪ 〈 5,∞〉 B. x ∈ 〈−∞, 5〉 C. x ∈ 〈 4, 5〉 D. x ∈ [5, +∞〉8. Rjesenje jednadzbe log5 (x − 4) + log5 (x + 2) = log5 16 pripada intervalu

A. [0, 4] B. 〈 4, 10] C. 〈−∞, 0〉 D. 〈 10, +∞〉9. Cijena iznajmljivanja bicikla najprije je povecana 25%, a zatim snizena 22%. Sto

treba napraviti s trenutnom cijenom da opet bude jednaka pocetnoj?A. Povecati je 3% B. Sniziti je 3% C. Povecati je 2.56% D. Sniziti je 2.56%

10. Domena funkcije f (x) = log2x2

x + 1je

A. 〈−∞,−1〉 ∪ 〈 1, +∞〉 B. 〈−1, 0〉 C. 〈−∞,−1] D. 〈−1, +∞〉 \ {0}11. Na -dite povrsinu osjencanog kruznog odsjecka.

A. 17.01 cm2 B. 18.27 cm2 C. 18.53 cm2 D. 19.34 cm2

12. Odredite koordinate sredista S i duljinu polumjera r kruznice x2 + y2 + 8x +2y + 13 = 0.A. S(−4,−1) , r = 2 B. S(4, 1) , r = 2 C. S(−4, 1) , r = 4 D. S(8, 2) , r = 13

140 Matematicko-fizicki list, LIX 2 (2008. – 2009.)

Page 77: Matematicko-fizicki list Zagreb

13. Polinom drugog stupnja koji prolazi tockama A(−1,−6) , B(0,−3) i C(1,−4)je polinomA. f (x) = −6x2 − 3x − 4 B. f (x) = 5x2 + 3x + 5

C. f (x) = −2x2 + x − 3 D. f (x) = −3x2 + x − 2

14. Kakav trokut odre -duju pravci y = 3x − 1, x − 7y − 7 = 0 i x + y − 7 = 0?A. pravokutan B. tupokutan

C. jednakokracan D. trokut sa sve tri stranice razlicitih duljina

15. Ako je oplosje valjka 8π cm2 , a polumjer baze jednak visini valjka, tada jevolumen valjka jednakA. 2 cm3 B.

√2π cm3 C. 8π cm3 D. 2π

√2 cm3

16. Formula za volumen V = B · v , gdje je B povrsina baze, a v visina tijela, vrijedizaA. prizmu i valjak B. stozac i kvadar

C. piramidu i kuglu D. paralelepiped i krnju piramidu

17. Na koliko se nacina 5 ljudi moze posjesti oko okruglog stola?A. 120 B. 24 C. 20 D. 48

18. Izrazcos x

1 + sin x+ tg x je jednak

A. tg x B. cos x C.1

cos xD. ctg x

19. U kakvom su odnosu pravac y = −2x + 9 i parabola y2 = 3x?A. pravac je normala parabole B. pravac je tangenta parabole

C. pravac ne sijece parabolu D. pravac sijece parabolu u ishodistu

20. Treci clan geometrijskog niza kojemu su prva dva clana√

2 i 4 je jednakA. 8 B. 4

√2 C. 6 D. 8

√2

Grupa C

1. Izraza(a + 2)

(a + 1)4 − 1je jednak

A.a + 2a − 2

B.1

a2 + 2C. 1 D.

1a2 + 2a + 2

2. Racionalizacijom nazivnika u izrazu3 + 2

√3√

3 + 2dobije se

A.√

3 B. −√3 C. 2

√3 D.

√3 + 2

3. Broj rjesenja nejednadzbe x2 − 5x + 6 � 0 u skupu Z jeA. 1 B. 2 C. 3 D. 4

4. Otopina soli A mijesa se sa 16% otopinom soli B u omjeru 3 : 4 i dobije se 22%otopina. Postotak soli u otopini A iznosiA. 20% B. 24% C. 28% D. 30%

5. Nakon sto se podijele potencije (−21x3a3b)7 : (7xa2b5)9 dobije se

A. −37x12a3

72b38B. −214x6a

9b16C.

217

abx D. −73x12a37b28

Matematicko-fizicki list, LIX 2 (2008. – 2009.) 141

Page 78: Matematicko-fizicki list Zagreb

6. Polinom p(x) = x4 + 3x3 + 4x2 + x + a je djeljiv s q(x) = x2 + x − 1 ako je ajednakoA. 3 B. −3 C. 4 D. −4

7. Imaginarni dio kompleksnog broja(1 + i

1 − i

)4je jednak

A. −1 B. 1 C. 2 D. 08. Ordinata tocke na osi y koja je jednako udaljena od tocaka A(1,−5) i B(7, 3) je

jednakaA. 2 B. 3 C. 0 D. −2

9. Skup svih rjesenja nejednadzbex + 1x − 1

� 2 je

A. 〈 1, 3] B. [1, +∞〉 C. 〈−∞, 1〉 ∪ [3, +∞〉 D. R

10. Broj realnih rjesenja jednadzbe log x4 + log (x + 4)4 = 4 log 3 jeA. 1 B. 2 C. 3 D. 4

11. Napisite preslikavanje u obliku F(x, y) = (?, ?) tako da opisuje transformacijukoja tocku (x, y) osno simetricno preslikava s obzirom na os y .A. F(x, y) = (−x, y) B. F(x, y) = (y, x) C. F(x, y) = (x,−y) D. F(x, y) = (−x,−y)

12. Na slici je graf funkcije f (x) = logb x . Odredite b .

A. b = 2 B. b = 4 C. b = 8 D. b = 1613. Radijus kruznice x2 + 2x + y2 − 6y = 0 je

A. 10 B.√

10 C. 6 D.√

214. Opseg baze pravilne uspravne cetverostrane piramide je 24 cm, a povrsina

dijagonalnog presjeka je 3√

14 cm2 . Volumen piramide jeA. 72 cm3 B. 36 cm3 C. 12

√7 cm3 D. 4

√14 cm3

15. Hipotenuza pravokutnog trokuta iznosi c = 13 cm, a kut pri vrhu A je α = 30◦ .Povrsina trokuta je

A.169

√3

8cm2 B. 169 cm2 C. 13

√3 cm2 D. 144

√3 cm2

16. Tocka presjeka pravaca 2x + 4y = 3 i 3x − y = 0 se nalazi naA. osi x B. pravcu 7x + 7y− 6 = 0 C. kruznici x2 + y2 = 1 D. osi y

17. Zajednicka tetiva krivulja y2 = 8x i x2 − y2 = 9 ima duljinuA. 4 B. 3

√6 C. 12 D. 12

√2

18. Rjesenje jednadzbe 3 · 2x − 2x−1 = 20 zadovoljava uvjet

A. x < 0 B. 0 < x � 52

C.52

< x � 92

D. x >92

19. Odredite istinitu tvrdnju.A. Funkcija f (x) = log x poprima samo pozitivne realne vrijednosti.B. Funkcije g(x) = log2 x i h(x) = 2x su me -dusobno inverzne.C. Jednadzba ax = 0 (a > 0) ima uvijek dva realna rjesenja.D. Os x je asimptota funkcije f (x) = log x .

142 Matematicko-fizicki list, LIX 2 (2008. – 2009.)

Page 79: Matematicko-fizicki list Zagreb

20. Ako je cos x =√

24

, cos 2x jednako je

A. −34

B.45

C.43

D. −45

Rjesenja zadataka

Grupa A

1. A 2. C 3. A 4. D 5. A 6. B 7. C 8. A 9. C 10. B11. D 12. A 13. A 14. B 15. B 16. B 17. A 18. C 19. A 20. C

Grupa B

1. B 2. A 3. C 4. A 5. D 6. A 7. A 8. B 9. C 10. D11. B 12. A 13. C 14. C 15. D 16. A 17. B 18. C 19. A 20. D

Grupa C

1. D 2. A 3. B 4. D 5. A 6. B 7. D 8. A 9. C 10. D11. A 12. A 13. B 14. C 15. A 16. B 17. D 18. C 19. B 20. A

Priredili Blazenka Divjak i Damir Horvat

SVIM SURADNICIMA

U Matematicko–fizickom listu objavljuju se clanci iz matematike, fizike i informatike, smalim prilogom iz astronomije, zadaci i rjesenja, prikazi natjecanja i ljetnih skola iz matematikei fizike, zanimljivosti u obliku clanaka i zadataka od ucenika, profesora i ostalih matematicara,novosti iz znanosti, zadaci s razredbenih (kvalifikacijskih) ispita, zabavna matematika i nagradninatjecaj.

Prilozi trebaju biti napisani racunalom (Word, Tex, Latex) ili pisacim strojem sa sirokimproredom na formatu A-4. Uz kopiju posaljite i disketu.

Slike trebaju biti jasno nacrtane na posebnom papiru i pogodne za presnimavanje. Slike crtaneracunalom (eps, tif, gif, jpg i sl.) posaljite i na disketi.

Clanci neka ne budu dulji od osam stranica, a ako je to potrebno neka budu napisani unastavcima.

Pozivaju se ucenici da posalju clanak o nekoj od spomenutih tema, originalne zadatke srjesenjima ili prikaze nekih manifestacija (ljetne skole, susreti ucenika, rad skolske grupe).

Kako se rukopisi ne vracaju, sacuvajte original a posaljite kopiju na papiru formata A-4.

Svi rukopisi podlijezu recenziji redakcije ili neke strucne osobe za odre -deno podrucje.

Prilozi se salju na adresu ovog casopisa koja je na prvoj stranici lista.

RJESAVATELJIMA ZADATAKA

Svako rjesenje neka bude napisano na posebnom papiru (formata A-4 ili A-5) i to samona jednoj strani papira. Uz svako rjesenje na vrhu papira treba potpuno ispisati tekst zadatka.Svako rjesenje treba citljivo potpisati (ime i prezime), naznaciti razred, skolu i mjesto.

Matematicko-fizicki list, LIX 2 (2008. – 2009.) 143

Page 80: Matematicko-fizicki list Zagreb

Rjesenje nagradnog natjecaja br. 183

Rjesenje. Koristeci binomnu formulu dobivamo

(1 + 2)100 =

1000

!+ 2

1001

!+ 22

1002

!+ . . . + 2100

100100

!,

tj.xy = 3100 = (32)50 = (34)25 = (35)20 = (325)4 = (310)10 = (320)5 = (350)2,

odakle je (x, y) ∈ {(3, 100), (32, 50), (34, 25), (35, 20), (325, 4), (310, 10), (320, 5), (350, 2)} .

Knjigom su nagra -deni rjesavatelji:

1. Edin Ajanovic (3), Prva bosnjacka gimnazija, Sarajevo, BiH;

2. Kristijan Kvaternik (1), V. gimnazija, Zagreb.

Rijesili zadatke iz br. 4/232

(Broj u zagradi oznacava razred–godiste srednje–osnovne skole.)

a) Iz matematike: Edin Ajanovic (3), I. bosnjacka gimnazija, Sarajevo, BiH, 3105–3108,3110–3113, 3115, 3117, 3118; Davor Devald (1), I. prirodoslovno-matematicka gimnazija, SSIzidora Krsnjavoga, Nasice, 3105–3108, 3110, 3116; Kristijan Kvaternik (1), V. gimnazija,Zagreb, 3105–3108; Vanja Ubovic (2), Gimnazija Petra Preradovica, Virovitica, 3106–3108,3112, 3115.

b) Iz fizike: Ana Lucija Alviz (7), OS Fausta Vrancica, Sibenik, 278; Filip Dunaj (8),OS Augusta Cesarca, Krapina 278, 280; Arijan Golub (8), OS Augusta Cesarca, Krapina 278,280; Juraj Krsnik (8), OS Augusta Cesarca, Krapina 278, 280; Dorian Lacko (8), OS AugustaCesarca, Krapina 278, 280; Ana Paic (7), OS Fausta Vrancica, Sibenik 278–281; Karlo Radecic(7), OS Fausta Vrancica, Sibenik, 278, 280; Kristijan Kvaternik (1), V. gimnazija, Zagreb, 1392,1395, 1396.

Nagradni natjecaj br. 185

Dokazi da je broj11 . . . 111︸ ︷︷ ︸

2008

22 . . . 222︸ ︷︷ ︸2009

5

potpun kvadrat.

144 Matematicko-fizicki list, LIX 2 (2008. – 2009.)